1. Trang chủ
  2. » Ngoại ngữ

Tạp chí Epsilon số 16

172 10 0

Đang tải... (xem toàn văn)

Tài liệu hạn chế xem trước, để xem đầy đủ mời bạn chọn Tải xuống

THÔNG TIN TÀI LIỆU

Ta chọn trên mỗi một đường thẳng như vậy (số đường thẳng bằng n D .q m 1/=.q 1/; vì chúng chỉ có một điểm chung là gốc tọa độ) một điểm bất kỳ khác với gốc tọa độ, ví dụ có thể chọn điểm[r]

(1)

TỐN HỌC VÀ BĨNG ĐÁ Trần Nam Dũng

SỐ CHÍNH PHƯƠNG TRONG BIỂU DIỄN CÁC SỐ NGUYÊN

Nguyễn Quang Minh

ĐẲNG THỨC TỔ HỢP QUA CÁC BÀI TOÁN OLYMPIC Trịnh Đào Chiến

VÀ CÁC CHUYÊN MỤC KHÁC

NO

“Hãy hình dung buồng gương, phịng với tường gương Nếu vào phịng với vài nến hệ phản chiếu

trên tường gương có cảm giác lọt vào khơng gian vô tận.” TOPO CỦA VŨ TRỤ

Cao Chi “Một kỳ thi giản dị, dân chủ trung thực.”

(2)(3)

Epsilon 16 lên trang xuất xưởng ngày người hâm mộ nước phát cuồng hai chức vơ địch SEA Games hai đội tuyển bóng đá U22 nam nữ Epsilon, với tinh thần theo sát nhịp thở thời đại, có cách thể niềm hân hoan theo cách riêng

Và để kỷ niệm chiến thắng 3-0 đội tuyển nam trước U22 Indonesia trận chung kết, BBT Epsilon cam kết tiếp tục xuất Epsilon đến số 30 Thú vị theo tiến độ nay, số 30 phát hành vào năm tổng biên tập tròn 60 tuổi Quá phù hợp cho thay đổi chuyển giao hệ

Epsilon, hẳn nhiên tờ báo đặc biệt Theo ngôn ngữ thời đại tờ báo 4.0: khơng tồ soạn, khơng nhân viên, khơng quan chủ quản, khơng kinh phí Báo không bán người viết không nhận nhuận bút

Vậy mà số báo đặn Lần xuất thứ hai vào ngày thứ sáu mười ba không ngoại lệ1.

Trân trọng gửi đến độc giả Epsilon 16 với 15 tác giả, 14 viết, xuất ngày 13 tháng 12!

(4)

Cao Chi

Topo Vũ trụ 5 Nguyễn Lê Anh

Sự tích "Trâu Vàng", "Cáo Chín Đi" Hồ Tây 13 Trần Nam Dũng

Tốn học Bóng đá 21 Lý Ngọc Tuệ

Tính tốn với dấu chấm động máy tính - Phần 26 Nguyễn Tuấn Anh

Đơn đồ thị vô hướng đề chọn đội tuyển Tỉnh - Thành Phố 2019 - 2020 32 Trịnh Đào Chiến

Đẳng thức tổ hợp qua toán Olympic 48 Nguyễn Tất Thu

Định hướng giải số toán cực trị tổ hợp dành cho THCS 71 Nguyễn Quang Minh

Số phương biểu diễn số nguyên 80 Ngơ Hồng Anh

Những tốn sơ cấp kỳ thi IMC 100 Trần Quang Hùng

Một số bất đẳng thức diện tích tam giác 140 Lê Xuân Hoàng

Một số vấn đề đường tròn Mixtillinear Thébault 148 Nguyễn Minh Hà Lê Viết Ân

Một mở rộng đường thẳng Simson 153 S.B.Gashkov

Mã kỳ thi Olympic toán (II) 159 Nguyễn Hùng Sơn

(5)

TOPO CỦA VŨ TRỤ

Cao Chi

GIỚI THIỆU

Vấn đề nghiên cứu topo vũ trụ gây ý nhiều nhà vũ trụ học Nhiều vũ trụ xét mặt hình học vũ trụ vơ hạn (infinite) song ý đến topo (tức đến tồn cục) lại vũ trụ hữu hạn (finite) Nhiều quan sát CMB (Cosmic Microwave Background - Bức xạ Phông Vũ trụ) xác hố thêm nhờ chi tiết phát sinh từ hệ topo Bài viết nhằm mục đích giới thiệu vài nét tổng quát ảnh hưởng topo đến vũ trụ học

1 Topo (topology) gì?

Topo mơn học nghiên cứu hình dạng khơng gian mặt tính liên thơng (connectedness), tính liên tục (continuity) biên (boundary) Những tính bất biến phép biến đổi liên tục gồm biến đổi kéo dài, uốn cong mà không bao gồm biến đổi xé rách (tearing) dán dính (gluing)

Xem hình ta thấy cốc, hình xuyến hình xuyến xoắn tương đương topo với

Hình 1:Cái cốc, hình xuyến hình xuyến xoắn tương đương topo với nhau

(6)

Hình 2:Biến hình xuyến thành cốc nhờ phép biến đổi liên tục topo 2 Hình học topo

Cần phân biệt hình học (độ cong phẳng, dương hay âm?) topo (dạng nào, liên thông nào?) Xét mặt hình học tồn loại vũ trụ: Vũ trụ phẳng (Euclidean, độ cong khơng), vũ trụ cầu (đóng, hữu hạn độ cong dương) vũ trụ hyperbolic (mở, vô tận độ cong âm), xem hình số

Khi nói đến độ cong âm người ta thường nghĩ đến khơng gian vơ hạn Song có nhiều độ cong topo, ví dụ hình xuyến (torus) có độ cong âm mặt (inside edge) topo hữu hạn (finite)

Thuyết tương đối tổng quát (với phương trình Einstein vốn phương trình vi phân) nói lên tính định xứ (local) mà khơng thể xác định tính tồn cục (global) tức topo vũ trụ Thuyết tương đối tổng quát không bất biến biến đổi đồng phơi (homeomorphism - xem thích) mà bất biến biến đổi vi phôi (diffeomorphism) tức biến đổi toạ độ

Với vũ trụ đồng đẳng hướng ta có lời giải metric Friedmann-Lemaitre-Robertson-Walker (FLRW)

d s2Dgd xd x D d t2Ca2./

d r2 kr2 Cr

2 d 2

Csin2d ˆ2

:

trong đóalà thừa số kích thước vũ trụ, độ congk D0;C1; 1tương ứng với độ cong

không, độ cong dương độ cong âm (xem hình vẽ 3)

Giữa bán kính độ cong R mật độ trung bình vật chất vũ trụ có mối quan hệ R D a=jkj12 D 1=Hj 1j

1

2;trong đóH số Hubble, D

c;cònc mật độ ứng với vũ

trụ phẳng Ta có vũ trụ với độ cong không, độ cong dương độ cong âm tương ứng với mật độD1; > 1và< 1:

(7)

Một yếu tố metric định xứ cho trước tương ứng với tập lớn mơ hình vũ trụ khác nhau về mặt topo.

Tồn vô số (đếm được) dạng không gian với độ cong dương, tất khơng gian đóng vơ số khơng gian với độ cong âm số khơng gian đóng (hữu hạn) một số không gian mở (vô tận).

Thuyết tương đối mơ tả hình xuyến mặt phẳng với phương trình hình xuyến hữu hạn mặt phẳng vô hạn Để xác định topo vũ trụ cần hiểu biết vật lý nằm lý thuyết tương đối (như CMB -Cosmic Microwave Background - Bức xạ Phông Vũ trụ)

Hãy hình dung buồng gương, phòng với bức tường (gồm trần nền) là gương Nếu vào phòng với vài nến hệ phản chiếu các tường gương có cảm giác lọt vào không gian vô tận.

Giống buồng gương (xem hình 4) vơ tận vũ trụ ảo tưởng Vũ trụ hữu hạn thực tế Ảo tưởng vô tận phát sinh từ tượng tia sáng thực quỹ đạo chạy quanh không gian nhiều lần (khi vũ trụ hữu hạn) tạo nên hình ảnh đa bội thiên hà

Hình 4:Buồng gương gây hệ vơ tận đối tượng hữu hạn

Nếu buồng kính ta có3 bóng tranh tạo nên hình ảnh vơ số bóng

như hình vẽ số5:

(8)

Hình 5:Ba bóng buồng kính tạo nên hình ảnh vơ số bóng 3 Tạo hình

Topo giúp ta tạo hình xuyến (hoặc dải Moebius) từ mảnh phẳng không thời gian cách đồng đường mép (edge) đối diện mảnh phẳng (hình6)

Nói chung người ta biểu diễn phần đa diện (polyhedron) với mặt đối đồng đơi

Trên hình6ta thấy dán2mép hình vng ta có hình trụ, dán mép (tức

đồng mépavớia; bvớib) hình vng ta có hình xuyến, dán mép

của hình bát giác (đồng nhấtavớia; bvớib; c vớic; : : :) ta có hình xuyến với2lỗ Ta gọi

số lỗ genus ví dụ hình xuyến2lỗ có genusD2:

Hình 6: Tạo hình trụ, hình xuyến1lỗ và2 lỗ cách dán mép đối (tức đồng nhất mép)

Chú ý hình xuyếnS1S1 có nhờ đồng mép đối diện hình vng

(9)

Hình trụ, hình xuyến lỗ, hai lỗ khơng gian topođa liên thơng (multiplyconnected) Trong hình7ta có topođơn liên thông(cột một) hai topođa liên thông(cột2và3)

Hình 7: Đây vài ví dụ đa tạp đồng phôi (homeomorphic), số hàng dưới số lỗ topo chúng

Phần đa diện10mặt ngũ giác mà đôi mặt ngũ giác đồng với

nhau khơng gian đóng với độ cong âm (compact hyperbolic space) Vậy vấn đề nghiên cứu topo vũ trụ nằm câu hỏi sau

Vũ trụ đóng hay mở? Vũ trụ có lỗ (hay tay quai-handle) không? Vũ trụ liên thông hay đa liên thông? Những câu hỏi topo thường bị bỏ quên nhà vũ trụ học Trong mô hình đầy đủ phải kể đến câu hỏi topo Vũ trụ thực sân khấu ảo tưởng quang học khổng lồ phát sinh hiệu ứng thấu kính topo (topological lens)

Nghiên cứu vũ trụ ta phải ý hai mặt: Hình học topo Về mặt hình học ta có: Khơng gian Euclide (độ cong không), không gian cầu (độ cong dương) không gian hyperbolic (độ cong âm).

Không gian cầu trường hợp hữu hạn Đối với hai loại khơng gian cịn lại tính hữu hạn vô tận lại phụ thuộc vào topo Nếu topo đơn liên thơng (simply-connected) thì chúng vơ tận.

Song topo đa liên thơng (như hình xuyến với lỗ hay hai lỗ) có khả năng xét mơ hình vũ trụ khơng gian hữu hạn độ cong cả lúc mật độ vật chất số vũ trụ thấp (mà xét hình học ta phải có khơng gian phẳng hyperbolic vô tận).

Như khơng gian với độ cong âm hữu hạn topo đa liên thông.

4 Một vài chi tiết tốn học

Trong khn khổ vũ trụ học chuẩn vũ trụ mơ tả đa tạp không thời gian

M4 D R M cộng với metric FLRW Trong M D E3 (Euclidean), S3 (cầu) H3

(10)

Độ cong củaM tất điều cần thiết để xác định xem không gian3chiều hữu hạn (finite)

hay vơ hạn (infinite) Bởi khơng gianM đa tạp thương (quotient

manifold) dĩM D Mc

G :Trong đóMc D.E

3; S3; H3/là không gian gọi không gian phủ

tổng quát (universal covering) Không gianM không gian đa liên thông Phép G cho phép

phủ Mc tế bào gọi đa diện (fundamental polyhedron-FP) thực

những phép tịnh tiến gắn liền với việc đồng mép (xem hình8)

Ví dụ hình xuyếnT2 D EG2 đóFP hình chữ nhật với mép đồng

hình vẽ số6:

Trong không gian đa liên thông hai điểm nối liền nhiều đường trắc địa hệ vũ trụ hữu hạn ánh sáng từ đối tượng đến với quan sát viên theo nhiều quỹ đạo khác – bầu trời ta có nhiều hình ảnh nguồn xạ

Hình 8:Cư dân trongFDvà khơng gian phủ

Ta lát không gian phủ tổng quát (universal covering) với nhiều FD cách đồng

mặt mép

Một cư dân hình xuyến nhìn phía trước thấy phía sau nhìn thấy khơng gian phủ tổng qt mạng hình ảnh (xem hình8)

Với khơng gian đa liên thơng (có số lỗ) số lượngN copy củaFDtrong

tranh quan sát vũ trụ đánh giá công thức: N D V =VFD:Trong đóV thể tích

vùng vũ trụ quan sát cịnVFD thể tích củaFD: 5 Topo CMB

Trong vũ trụ học CMB xạ nhiệt tàn dư từ lúc Big Bang (xem hình9)

(11)

Hình 9:Bản đồ Bức xạ Phơng Vũ trụ CMB bầu trời

Tính đa liên thơng khai thác để tìm bất thường phổ lượng hệ topo Một điểm xạ theo nhiều hướng khác ánh sáng sau vòng quanh vũ trụ nhiều lần đến người quan sát cho nhiều hình ảnh đa bội điểm xạ

Nhiều khía cạnh CMB bị thay đổi topo vũ trụ đa liên thông (hữu hạn):

(1) Topo đa liên thơng phá huỷ tính đẳng hướng tính đồng toàn cục, xuất CMB phân bố bất đẳng hướng bất đồng ứng với hình ảnh ảo (ghost) nhiều điểm

(2) Phổ loại thăng giáng gián đoạn, điều phản ánh không gian hữu hạn (tương tự Cơ học lượng tử thể tích hữu hạn cho phổ lượng gián đoạn)

(3) Bức tranh thăng giáng nhiệt độ phản ánh xuất lặp lại điểm nóng lạnh hệ thấu kính topo

6 Kết luận

Topo vũ trụ lĩnh vực quan trọng phối hợp lý thuyết toán học với định luật vật lý Chúng ta có khả quan sát topo vũ trụ cách gián tiếp nhờ quan sát CMB Hiểu topo vũ trụ nghiên cứu xác ảnh hưởng - lượng vật chất lên không thời gian Trong vũ trụ học đại nhà khoa học sử dụng toàn diện vật lý, hình học lẫn topo

(12)

Tài liệu

[1] Mark Baltovic,The topology of the Universe

[2] M.J.Reboucas, G.I.Gomero, Cosmic topology: A brief Overview, Brazilian Journal of Physics, vol.34, no.4A, December,2004:

[3] Jean-Pierre Luminet,Is the Universe crumpled?

[4] Janna Levin,Topology and the Cosmic Microwave Background, arXiv:gr-qc/0108043v2 20 Aug2001, Physics Reports 365(2002) 251-333

(13)

SỰ TÍCH "TRÂU VÀNG", "CÁO CHÍN ĐI" VÀ

HỒ TÂY

Nguyễn Lê Anh

GIỚI THIỆU

"Để hiểu lịch sử Việt Nam cần phải tìm hiểu tình hình địa chất theo thời gian."Đó nhận định tác giả "ruột" Epsilon - Nguyễn Lê Anh Nếu năm 2018 năm "đi tìm máy bay" năm 2019 năm mà ơng cịn xa nhiều: tìm lại nguồn cội dân tộc! Vẫn ln góc nhìn lạ với chuỗi lập luận logic dựa nghiên cứu dày công, từ khảo cứu tài liệu đến thực địa địa phương, Nguyễn Lê Anh đưa hàng loạt kết nghiên cứu lịch sử, khơng kết đánh đổ nhận định vốn tồn hàng trăm năm Ở Epsilon số 16 này, trân trọng giới thiệu tới độc giả phần loạt lịch sử với góc nhìn thú vị lạ Hồ Tây tích "trâu vàng" "cáo chín đi"

1 Vị trí đặc biệt hồ Tây?

Cấu tạo tầng đất từ thềm lục địa, tức từ đáy biển xưa trở lên, gồm đá cát đất thịt Các vật liệu sông Hồng mang tới, xếp lại theo định luật Acximet, "vật chất có khối lượng riêng lớn dưới, vật chất có khối lượng riêng nhỏ Các hịn đá hộc bán kính cỡ vài mét nằm cùng, tới cuội sỏi cát, tầng sét Nếu quan sát thấy xuất lớp có khối lượng nhỏ dưới, ví dụ lớp sét lớp cuội sỏi, tức có thay đổi chế độ bồi lắng Ví dụ lớp hình thành biển dâng, lớp biển rút

Dòng nước mang phù sa chảy chậm lại phù sa lắng đọng Nơi dịng nước chảy chậm thường nơi bị đẩy cho chảy xốy vịng ngược trở lại – vận tốc mà bị triệt tiêu Phù sa đọng dần thành gị mơ hình dạng dịng nước xốy, độ cao gị ngun tắc khơng cao q mực nước sơng cao Hay nói xác mức cao dịng nước cửa sơng Đối với gị lớn, gió thổi bụi tấp vào gốc khiến cho gị cao dần nhờ gió Khi chưa có đê bao nước sông chảy tự do, mùa mưa hàng năm phù sa tráng thêm lớp lên toàn đồng

(14)

dâng sông Hồng mang phù sa bồi Tuy nhiên, hạt bùn nhỏ phù sa bị dòng hải lưu biển rửa trôi nên lớp cát lắng biển tiến

Lượng phù sa hàng năm sông Hồng khoảng 100 triệu tấn, tương đương 70 triệu m3 đất.

Đồng Bắc Bộ tam giác cân, đáy 150km hai cạnh bên 200km Nó nghiêng từ Phú Thọ, độ cao 30m tới biển Thời gian để có lượng phù sa cần thiết lấp đầy khối tứ diện vuông Phú Thọ, cao 30m, đáy tam giác cân nói 2000 năm Như 6000 năm trước đồng Bắc Bộ vịnh sâu khoảng 20m

Tất gò đất vùng đồng Sông Hồng cao không 30m so với mực nước biển – độ cao cửa sông Hồng Phú Thọ Khu vực Cổ Loa, Ba Đình, Gị Đống Đa, khu vực Xuân La gò cao khoảng 15m so với mực nước biển, cao khoảng 5m so với mặt chung khu vực Những gò đất cao khu vực Hương Canh, Phú Thọ tạo từ kỷ băng hà trước, khoảng 200 nghìn năm trước, mực nước biển cao vài chục mét

Đồng sông Hồng phù sa tạo nhiều năm Như sông Hồng đổ phù sa biển, bờ biển lùi dần Nói cách đơn giản đoạn cuối sông đoạn sông chảy nước biển trước bị phù sa làm cho nâng lên Dịng chảy sơng biển bị dòng Hải Lưu đẩy ngược trở lại, tạo xốy vun phù sa thành gị Con sơng Hồng chảy ngoằn nghèo rẽ nhánh quanh gò tạo hệ thống sông chảy đồng Bắc Bộ Con sơng Hồng có độ sâu trung bình khoảng 20m chảy đất cát phù sa từ hàng chục nghìn năm trước tạo Do độ dày tầng bồi lắng tới thềm lục địa vào khoảng 100 mét, thể hình dáng sơng Hồng hệ lụy lực tự nhiên, yếu tố lồi lõm thềm lục địa độ sâu trăm mét phía bên tầng đất phù sa khơng ảnh hưởng tới định hình dịng sơng Tất gị đất chắn thẳng dịng nơi sơng hồng chảy tới bị bào mòn theo thời gian

Xét tổng thể có lực tự nhiên tác động lên dịng nước sơng Hồng, lực thứ nghiêng khối lục địa, lực thứ hai lực Coriolis đẩy dòng nước chảy theo hướng từ Tây sang Đông Lực Coriolis xuất dòng nước chảy xa khỏi tâm trái đất, tức chảy phía xích đạo Khi chảy song song với vĩ độ lực khơng cịn (chảy từ vĩ độ thấp tới vĩ độ cao, sông Kỳ Cùng, hướng chảy ngược lại, từ Đơng sang Tây) Do độ nghiêng thềm lục địa mà lực chảy nghiêng lúc có mạnh nhiều lần lực Coriolis Quy luật dịng chảy có phù sa cửa sơng biển ln hình thành xốy, xốy hình thành gị đất Lực đẩy dịng nước phía bên gị đất lực Coriolis, yếu dần theo thời gian Tất sức mạnh dịng nước sơng Hồng chuyển xuống nhánh gị

Có hai sơng tác động đến hướng chảy sông Hồng sông Đà sông Lô Sức chảy sông Đà mạnh gấp nhiều lần sức chảy sơng Hồng sơng Lơ định dịng nước sơng Hồng

Vào khoảng 6000 năm trước bờ biển vào khoảng Phú Thọ Khoảng 4000 năm trước bờ biển xưa chạy dọc theo QL18 tiếp đến QL2A, theo QL2C, theo QL21A

(15)

Hình 1: Sơng hồng chảy từ Mê Linh qua khu vực Vân Trì phía Chí Linh Cái dịng sơng bị hiệu ứng gị chia đơi, nhánh bên bị chẹn khiến nhỏ đi, dịch chuyển song song vị trí sơng Đáy ngày Nhánh bên chảy mạnh sông Hồng ngày Đúng sông Hồng chảy theo hướng Tây Bắc sang Đông Nam, có lẽ bị cố hố sụt lún Hồ Tây khiến cho dòng nước bị bẻ quặt góc 45 độ

sơng Lơ định, đẩy dịng chảy sơng Hồng phía Nam thành sông Đáy Như sông Đà sông Lô để lại dấu vết sông Phan sông Đáy Các gị đất xuất khiến sơng Phan trở thành nhánh phụ chảy lực Coriolis dần phía đầm Vạc Sự hình thành sơng Cà Lồ, sơng Đuống, sơng Thái Bình theo ngun lý dựa vào lực Coriolis Như phù sa bị đưa theo dịng nước, từ Tây Bắc tới Đơng Nam, mà lịng sơng Hồng dịch chuyển song song dần theo hướng Đơng Bắc xuống Tây Nam Nhìn đồ thấy bãi Phúc Xá lớn dần Con sông Đuống khu vực Mê Linh chảy phía Luy Lâu Gị Cổ Loa chia sông thành hai nhánh, nhánh bên sông Đuống, nhánh sông Hồng ngày Nhánh bị phù sa làm cho nghẽn nhỏ lại Nó tịnh tiến song song phía Nam vị trí sơng Đuống

(16)

ngay hồ Tây thấy thay đổi hướng dòng chảy tới gần 45 độ Rõ ràng lệch hướng kết việc bị lún cục làm thay đổi dịng chảy Rõ ràng sơng Hồng ln chảy phía bên Gia Lâm, dịng sơng có su dịch chuyển song song phía hồ Tây, có lẽ chưa chảy qua hồ Tây

Phía bên Gia Lâm vùng đất ruộng phù sa đất cát đọng lại mà thành, khơng có cấu trúc cứng để tạo phản lực đẩy dịng nước sơng Hồng chảy từ bên Vân Trì sang tới Hồ Tây Sông hồng chảy từ Mê Linh qua khu vực Vân Trì phía Chí Linh Cái dịng sơng bị hiệu ứng gị chia đơi, nhánh bên bị chẹn khiến nhỏ đi, dịch chuyển song song vị trí sơng Đáy ngày Nhánh bên chảy mạnh sông Hồng ngày Đúng sông Hồng chảy theo hướng Tây Bắc sang Đơng Nam, có lẽ bị cố hố sụt lún Hồ Tây khiến cho dịng nước bị bẻ quặt góc 45 độ nói

2 Trận động đất năm 1016

Để làm rõ chi tiết trận động đất tạo hồ Tây, cần xét thêm ghi chép lịch sử

Vào năm 1016 trận động đất lớn thềm lục địa bị bẻ gãy độ sâu khoảng 20km phía Hồ Tây Sụt lún dần vòng 1000 năm qua tạo vùng trũng Hồ Tây ngày

Trang 84 Đại Việt Sử Ký (Bản in Nội Các Quan Bản Mộc khắc năm Chính Hịa thứ 18 (1697)) chép kiện năm 1016 sau:

"Thiên hạ mờ tối, trung thần giấu tính danh, giữa trời nhật nguyệt sáng, ai chẳng thấy dáng hình".

Nguyên văn:"Thiên hạ tao mơng muội, trung thần nặc tính danh,

trung thiên minh nhật nguyệt, thục bất kiến kỳ hình".

Bài thơ cịn có dị Việt điện u linh, Sơn Tây tỉnh chí Như kiện tạo ấn tượng rất ghê gớm có thật

Đại Việt Sử Ký 1697, Lê Văn Hưu, Phan Phu Tiên, Ngô Sĩ Liên, biên soạn Sự kiện vào năm 1016 Đại Việt Sử Ký có chép kiện động đất Đây chắn trận động đất mạnh, gần 700 năm sau cịn nhớ để đưa vào sử

Lẽ đương nhiên kiện thần thánh hóa, sức mạnh trận động đất so sánh với đại chiến đánh thắng qn Ngun Mơng

(17)

Bính Thìn, [Thuận Thiên] năm thứ [1016], (Tống Đại Trung Tường Phù năm thứ 9) Mùa xuân, tháng 3, lại lập hoàng hậu: Tá Quốc hoàng hậu, Lập Nguyên hoàng hậu, Lập Giáo hồng hậu, Độ cho nghìn người kinh sư làm tăng đạo Dựng hai chùa Thiên Quang, Thiên Đức tô bốn tượng Thiên Đế.

Động đất.

Làm lễ tế vong danh sơn Vua nhân xem núi sơng, đến bến đị Cổ Sở (Cổ Sở: bến Cổ Sở -tên nôm bến Giá, xã Yên Sở, huyện Hoài Đức, tỉnh Hà Tây), thấy khí tốt núi sơng, tâm thần cảm động, làm lễ rưới rượu xuống đất, khấn rằng: "Trẫm xem địa phương này, núi lạ sông đẹp, có nhân kiệt địa linh hưởng lễ".

Đêm ấy, vua chiêm bao thấy có dị nhân đến cúi đầu lạy hai lạy, nói: "Thần người làng này, họ Lý tên Phục man, làm tướng giúp Nam Đế, có tiếng người trung liệt, giao trông coi hai dải sông núi Đỗ Động Đường Lâm, bọn Di Lão không dám xâm phạm biên giới, một phương yên bình Đến chết, thượng đế khen trung trực, sắc cho giữ chức cũ Cho nên phàm giặc Man Di đến cướp chống giữ Nay may bệ hạ thương đến, biết cho thần giữ chức lâu rồi".

Rồi thung dung nói: "Thiên hạ mờ tối, trung thần giấu tính danh, giữa trời nhật nguyệt sáng, ai chẳng thấy dáng hình".

Nguyên văn "Thiên hạ tao mơng muội, trung thần nặc tính danh,

trung thiên minh nhật nguyệt, thục bất kiến kỳ hình".

Vua thức dậy nói việc với Ngự sử đại phu Lương Nhậm Văn rằng: " Đó ý thần muốn tạc tượng" Vua sai bói xin âm dương, nhiên Bèn sai người châu [8a] lập đền đắp tượng hình dạng người chiêm bao, tuế thời cúng tế Khoảng niên hiệu Nguyên Phong [1251-1258] đời Trần, người Thát Đát (Tartar hay Tatar, tộc Mông Cổ, đây chỉ quân Nguyên - Mông) vào cướp, đến địa phương này, ngựa khuỵu chân không được, người thôn dẫn dân chúng chống đánh, chém đầu giặc, giặc chạy tan Khoảng năm Trùng Hưng [1285-1293], [Thát Đát] lại vào cướp, đến đâu đốt phá, mà ấp vẫn như che chở, không bị xâm phạm mảy may, lời thần nói.

- Nhà Tống phong vua làm Nam Bình Vương - Năm mùa to, 30 bó lúa giá 70 tiền Cho thiên hạ năm nộp tô thuế.

Đinh Tỵ, [Thuận Thiên] năm thứ [1017], (Tống Thiên Hy năm thứ 1) - Mùa xuân, tháng 3, cho Trần Văn Tú làm Thái phó Xuống chiếu xá tơ ruộng cho thiên hạ - Điện Càn Nguyên bị sét đánh, vua coi chầu điện phía đơng.

(18)

thần linh mạnh tới mức sức mạnh hủy diệt quân Nguyên - Mông Tuy nhiên trận động đất không làm thay đổi diện mạo Hà Nội

Điện Càn Nguyên, tức khu vực Hoàng Thành bị sét đánh, vị trí tương đối cao Ngày biết vùng đất cổ có độ cao khoảng 16m, độ cao với đình Quán La (Phía Tây hồ Tây, Hà Nội) Như Gị đất cổ xưa dài 3km cao 16m chạy dài từ dọc Hồ Tây đến Hồng Thành bị sụt Có lẽ vùng sụt lún hình trịn, phù sa sơng Hồng bồi dần nửa

Hồn tồn khơng thấy nói tới Hồ Tây, Đại Việt Sử Ký ghi nhận Vua đến vùng Yên Sở, huyện Hồi Đức, tỉnh Hà Tây Như hiểu trận động đất 1016 phá hủy hoàn toàn vùng Hà Nội Hà Tây, tượng sụt lún tạo Hồ Tây (hồ Dâm Đàm) xảy từ từ Hồ Tây xưa hồ khơng lớn ngày Điều giải thích ngun nhân tích Trâu Vàng Hồ Tây tới sau xuất

Lần Đại Việt Sử Ký Toàn Thư chép hồ Tây năm 1044 Đại Việt Sử Ký chép"Tháng 9 Vua sai đặt cũi lớn Dâm Đàm lấy voi nhà Chiêm Thành làm mồi dử voi rừng vào trong ấy, Vua thân đến bắt".

Sự tích kể rằng: "Thiền sư Minh Khơng cho đúc tượng Phật cao trượng, chóp đỉnh tháp Báo Thiên chín tầng, đỉnh đồng có đường kính 10 sải tay chuông đồng cực lớn Chuông đúc xong, Vua sai Minh Không đánh hồi chuông dài Nghe tiếng chuông trâu bằng vàng to lớn nằm trước kho đồng bên Tàu tự dưng bừng tỉnh "Đồng đen mẹ vàng" ngỡ là tiếng mẹ gọi liền vươn phóng thẳng xuống phương Nam tìm đến chng khổng lồ, quần xung quanh Trâu vàng quần quanh mà không thấy, khiến cho vùng đất lớn quanh chuông sụt xuống thành vùng hố sâu Quả chuông sau hồi đổ sụp xuống hố sâu Trâu vàng theo nhảy xuống nằm bên cạnh, chẳng sau vùng đất bị trâu vàng dẫm sụt, nước tràn đầy trải rộng thành hồ nước mênh mông Thiền sư Minh Không sau thợ đúc đồng vùng Ngũ Xá (nay Đông Nam hồ Trúc Bạch) thờ làm tổ sư nghề đúc đồng."

Như vùng phía nam hồ Trúc Bạch nay, tức gần với Hoàng Thành làng nghề đúc đồng Động đất gây sụt lún nhấn chìm lị đúc đồng tạo hồ nước, hồ Tây ngày

Ngày nước cạn nhìn thấy vài ngơi mộ cổ phía xa lịng hồ Tây Vậy ngồi vị trí bị sụt sâu năm 1016, phần lớn hồ Tây bị chìm dần suốt 1000 năm qua Qua ước tính hồ Tây bị lún khoảng 2m vòng 1000 năm qua

Do từ năm 1016 đến khơng có trận động đất lớn ghi nhận, có lẽ trận động đất năm 1016 gắn với tích Trâu Vàng Hồ Tây, nói lên kiện sụt lún trận động đất mà tạo Hồ Tây Xét đồ 1910 phần lớn Hồ Tây ruộng lúa nước lúc lúc chìm, ngày chìm hẳn Như hồ Tây bị sụt tới tiếp tục sụt lún Điều cho thấy trận động đất sinh Hồ Tây chưa phải xẩy lâu Trong “Tây Hồ bát cảnh” thời Lê cho biết rừng gỗ tầm bán đảo hồ Tây

(19)

Hình 2: Dựa đồ 1885 1910, ước lượng hai đảo nhỏ chiếm diện tích khoảng 1/50 hồ Tây hồ Tây bị biến sau khoảng thời gian cỡ 20 năm

So sánh đồ 1885 1910 sau 25 năm hai mơ đất có diện tích khoảng 1/50 mặt hồ bị biến Như thời gian hồ Tây bị sụt lún khoảng 25x50 năm, tức ứng với khoảng 1000 năm trước Rất hệ lụy trận động đất vào năm 1016 nhắc tới Đại Việt Sử Ký Toàn Thư

3 Tiếp tục khảo sát

Những mộ cổ với nghĩa địa chưa chìm hẳn cịn thấy hồ Tây cho thấy trình lún từ từ Theo ước tính ngơi mộ beton lún sâu xuống khoảng 1m Vật liệu xây dựng beton có từ Pháp sang Việt Nam, tức sau khoảng 100 năm Hồ Tây bị lún khoảng 1m Tức nơi lún sâu Hồ Tây vào khoảng 10m Theo kết điện trở [1] Nguyễn Văn Giảng, Noboru Hida, Maksim Bano dọc theo hai tuyến, tuyến thứ SH1 cắt ngang sông Hồng, tuyến thứ hai SH2 theo bờ sơng Hồng phía Hồ Tây nhận thấy phân lớp điện trở rõ ràng, theo tuyến SH2 hồn tồn khơng có xáo trộn lớp Như khơng thể có đổi dịng chảy sông Hồng Theo tuyến SH2 thấy di chuyển từ Đơng Ngạc sang phía Nhật Tân thấy lớp đồng điện trở võng dần xuống khoảng 10m chiều dài thẳng 3km Đó hệ lụy bị lún cục Hồ Tây gây Như lún cục Hồ Tây diễn với mức độ khoảng 2m 1km chiều dài Bán kính Hồ Tây khoảng 2.5km, vùng hố lún phải lớn hơn, ước lượng khoảng 3km nơi lún sâu khoảng 10m

(20)

Hình 3: Sơ đồ vị trí tuyến đo địa vật lý dải ven sơng Hồng

Tài liệu trích dẫn

[1] Van Giang, N., Hida, N and Bano, M., 2012 The characteristics of shallow geological structure for Red River side-Tay Ho-Hanoi area by geophysical data VIETNAM JOURNAL OF EARTH SCIENCES, 34(2)

(21)

TỐN HỌC VÀ BĨNG ĐÁ

Trần Nam Dũng

(Trường Đại học KHTN, ĐHQG thành phố Hồ Chí Minh) GIỚI THIỆU

Bóng đá môn thể thao nhiều người yêu thích Các kỳ World Cup, Euro, Asian Cup, AFF Cup hay Seagames nhận ủng hộ cuồng nhiệt tín đồ túc cầu giáo Rất thú vị bóng đá liên quan nhiều đến tốn học, từ vấn đề thống kê, xác suất, đến toán lập lịch, đến quỹ đạo bóng bóng Bài viết giới thiệu số toán liên quan đến bóng đá, gồm vấn đề sân có ngòai sân cỏ Bài viết lấy cảm hứng từ chức vô địch Seagames30của hai đội tuyển U22 nam nữ

1 Các toán

Các tốn đây, dù số liệu lấy từ thực tế, toán giả định Tất kiện lấy từ đề bài, không lấy thêm từ thơng tin bên ngồi

Bài tốn Trước trận chung kết bóng đá nam Seagames30giữa Việt Nam Indonesia, nhà

cái Smartbets kèo cho kết (của90phút thi đấu thức) là 1:75–3:5–5:1;tương

ứng với Việt Nam thắng – Hòa – Indonesia thắng.

a) Hãy cho biết nhà tính tốn xác suất Việt Nam thắng – Hòa – Indonesia thắng trong

90phút thức để đưa tỷ lệ trên?

b) Do trận chung kết khơng có kết hòa nên hòa trong 90phút thi đấu hai đội sẽ

đấu tiếp hiệp phụ Nếu hai hiệp phụ hịa đá ln lưu11m Giả định rằng

nếu đá phạt đền xác suất thắng hai đội ngang Hãy nêu đề xuất hợp lý cho việc tính xác suất thắng – hòa – thua hai đội hai hiệp phụ (nếu hai đội hịa hai hiệp chính), từ tính xác suất Việt Nam đoạt chức vơ địch.

Bài toán Trước trận lượt cuối bảngB;đội Việt Nam được12điểm, ghi17bàn để lọt

lưới4bàn, đội Indonesia được9điểm, ghi13bàn để lọt lưới2bàn, đội Thái Lan được9điểm,

ghi 12bàn để lọt lưới2 bàn Ở lượt trận cuối, Indonesia gặp Lào, Việt Nam gặp Thái

Lan Đây là trận đấu định vé vào bán kết Chỉ có3 đội Việt Nam, Indonesia

(22)

Hãy tính xác suất Việt Nam đoạt vé vào bán kết (hai đội xếp đầu bảng đoạt vé) với giả định sau: Xác suất Indonesia thắng Lào là 90%:Xác suất Thái Lan thắng Việt Nam là33%;trong

đó xác suất Thái Lan thắng Việt Nam với2bàn cách biệt là10%:

Bài toán Ở giải Champions League mùa bóng2008ở tứ kết có8đội, có4đội của

Anh Đức, Ý, Tây Ban Nha, Thổ Nhĩ Kỳ nước có1đại diện.8đội bóng bốc thăm ngẫu nhiên

đấu loại trực tiếp Nếu giả định rằng8đội bóng ngang ngửa (xác suất thắng trận

được chia cho hai đội) xác suất có trận chung kết tồn Anh bao nhiêu? (Thực tế thì đã xảy vậy, với trận chung kết MU Chelsea sân Moscow).

Bài tốn Ở Seagames30;bảngB 6đội Việt Nam, Thái Lan, Indonesia, Singapore, Lào,

Brunei lịch thi đấu là:

25=11WViệt Nam vs Brunei, Indonesia vs Thái Lan, Lào vs Singapore.

28=11WViệt Nam vs Lào, Indonesia vs Singapore, Thái Lan vs Brunei.

1=12WViệt Nam vs Indonesia, Thái Lan vs Singapore, Lào vs Brunei.

3=12WViệt Nam vs Singapore, Thái Lan vs Lào, Indonesia vs Brunei.

5=12WViệt Nam vs Thái Lan, Indonesia vs Lào, Singapore vs Brunei.

Đây hình mẫu cho lịch thi đấu giải đấu vòng tròn gồm6đội Tất có5

vịng đấu, vịng có3trận.

Hãy xếp lịch thi đấu cho giải đấu gồm10đội thi đấu vòng tròn lượt, gồm9vòng, mỗi

vòng5trận Cùng tốn cho giải đấu gồm2nđội thi đấu vịng trịn lượt.

Bài tốn Ở giải bóng đá có6đội thi đấu vịng trịn1lượt Trong trận đấu, đội thắng

được3điểm, đội thua được0điểm Nếu hai đội hịa đội được1điểm Khi kết thúc

giải người ta thấy điểm đội là số nguyên liên tiếp Hỏi đội vô địch mấy

điểm?

Nếu thay6đội bóng n đội bóng, hỏi điều xảy khơng? Giải thích rõ câu

trả lời.

Bài tốn Kể từ năm2008;kích thước sân bóng đá chuẩn dài105m rộng68m Khung thành

có chiều ngang7m32, cao2m44:Vịng trịn sân có bán kính9m15:Các khu vực5m50 16m50được tính theo khoảng cách đến cầu mơn.

a) Nếu có sút phạt góc bên phải vịng16m50thì khoảng cách từ đó

đến tâm cầu môn bao nhiêu?

b) Một cầu thủ di chuyển từ góc bên trái vịng 16m50về hướng góc bên

phải vịng Hỏi vị trí đường thẳng mà di chuyển anh có góc sút lớn nhất?

c) Theo thống kê chuyên gia, cầu thủ trận chạy trung bình khoảng

10km Với số liệu này, cho biết cầu thủ lên công thủ Cafu, Roberto Carlos,

(23)

Bài toán Kể từ World Cup 1970tại Mexico bóng đấu thức thường được

may từ miếng da hình ngũ giác lục giác có cạnh Ở năm1970thì ngũ giác

sẽ có màu đen, lục giác có màu trắng Sau có nhiều biến thể với thiết kế mỹ thuật khác nhau.

a) Dựa vào thơng tin trên, cho biết có miếng da hình ngũ giác có bao nhiêu miếng da hình lục giác?

b) Cho biết bóng có đường kính25cm, ước lượng tổng chiều dài đường khâu.

c) Nếu miếng da (ngũ giác lục giác đều) có cạnh bằng 4:5cm, ước lượng

(24)

2 Hướng dẫn, lời giải, đáp số

1 a) Xác suất tỷ lệ nghịch với tỷ lệ cược Xác suất cao, tỷ lệ cược thấp Nếu lấy nghịch đảo số1:75; 3:5; 5:1rồi cộng lại ta được1:053221lớn hơn1một chút Đây

chất tỷ lệ lời nhà (nếu 1thì trị chơi có kỳ vọng0) Ta lấy

số để điều chỉnh lại xác suất, từ tính xác suất tương ứng là54:3%; 27:1%

và18:6%:

b) Đáp án câu hoàn toàn dựa vào giả định Có thể có lập luận hội hai đội hiệp phụ ngang nhau, áp dụng tỷ lệ cũ cho hiệp phụ Cuối cùng, hiệp phụ kéo dài30 phút nên khả hòa cao hơn, ta nâng xác suất hịa lên

giảm xác suất thắng/thua xuống (vẫn theo tỷ lệ)

Ví dụ ta cho kết hịa là54:2% (gấp đơi so với hai hiệp chính) Số phần trăm lại

chia cho hai đội theo tỷ lệ hiệp là34:1% và11:7%:

Nếu theo phương án xác suất Việt Nam vô địch bằng54:3%C27:1%.34:1%C27:1%/D

70:9%:

2 Việt Nam đoạt vé không thua Thái Lan với cách biệt2bàn trở lên Trong trường hợp

Việt Nam thua Thái Lan với bàn cách biệt trở lên Việt Nam đoạt vé Indonesia

khơng thắng Lào Vì xác suất Việt Nam đoạt vé bán kết bằng90%C10%:10%D91%:

3 Do đội ngang ngửa bắt thăm ngẫu nhiên nên xác suất lọt vào trận chung kết hai đội Do cóC82 D28cặp đấu xuất chung kết,

cóC42D6cặp đấu tồn Anh nên xác suất để có trận chung kết tồn nước Anh 28 D

3 14:

4 Ta xét hình9cạnh

Đánh số tâm số10;đỉnh là1; 2; 3; : : : ; 9:Lần lượt xếp lịch thi đấu sau: Ở lượt

(25)

Ví dụ lượt1sẽ là1 10; 9; 8; 7; 6;lượt2sẽ là2 10; 1; 9; 8; 7:

Lịch đấu cho giải gồm 2n đội lập hoàn toàn tương tự Chú ý ngồi phương pháp xây dựng ý tưởng hình học độc đáo trên, ta cịn có cách xây dựng theo kiểu quy nạp

5 Ta có6đội bóng thi đấu tất cả15trận Mỗi trận đem lại nhiều là3điểm 2điểm Khơng thể xảy trường hợp tất trận phân định thắng/thua (lúc điểm số

mỗi đội bội của3;khơng thể số nguyên liên tiếp) tất trận hịa (lúc

điểm đội nhau) Vậy tổng điểm đội< 45 và> 30 Gọi a điểm đội xếp cuối

điểm đội làa; aC11; : : : ; aC5:Suy tổng điểm đội là6aC15:Từ lập luận

ở trên, suy raaD3hoặcaD4:

Nếua D 3, tổng điểm đội là33; suy có12 trận hịa,3trận phân định thắng - thua Ba

đội xếp đầu được6; 7; 8điểm đội phải thắng nhất1trận, suy đội thắng đúng1

trận Nhưng đội vô địch thắng1trận khơng thể được8điểm, mâu thuẫn Vậya

không thể bằng3:

NếuaD4;tổng điểm đội là39, có6trận hịa,9trận phân định thắng thua điểm đội

lần lượt là4; 5; 6; 7; 8; 9:Ta xếp để có giải đấu Xin dành tập

cho bạn đọc Hơn nữa, ta chứng minh mệnh đề tổng quát sau (bằng quy nạp)

Với mọi n > 4;tồn giải đấu gồmn đội thi đấu bóng đá vòng tròn lượt cho ở

cuối giải điểm số đội làn 2; n 1; : : : ; 2n 3:

6 Bài tốn tính tốn đại số đơn giản Riêng câu góc sút lớn gợi ý điểm có góc sút lớn đường trịn qua điểm chân hai cầu mơn tiếp xúc với đường thẳng quỹ đạo chạy cầu thủ

7 GọiP số ngũ giác,H số lục giác,V số đỉnh,S số cạnh Ta thấy góc đỉnh

của lục giác bằng120ı;cịn góc đỉnh ngũ giác bằng108ı;cho nên đỉnh có3

đa giác kết nối với nhau, phải là1ngũ giác kết nối với2lục giác Mỗi cạnh cạnh

của hai đa giác Do ta có phương trình sau: Tính số lượt đỉnh:5P C6H D3V:

Tính số lượt cạnh:5P C6H D2S:

Một đỉnh có ngũ giác kết nối lục giác:5P D 6H;tức là5P D 3H:Ngoài ra, theo

cơng thức Euler thìV S C.P CH /D2:

Từ ta dễ dàng tính đượcP D12; H D20; V D60; S D90:

Mơ hình hóa bóng hình nhị thập diện cụt, ta ước lượng tổng chiều dài đường khâu (bằng90lần cạnh ngũ giác lục giác) là476:1cm

Nếu cạnh ngũ giác lục giác 4:5 cách tính theo tỷ lệ thuận, ta ước lượng

đường kính bóng là21:41cm

(26)

TÍNH TỐN VỚI DẤU CHẤM ĐỘNG TRONG MÁY TÍNH - PHẦN 1

Lý Ngọc Tuệ

GIỚI THIỆU

Khi làm việc với máy tính hay điện thoại, khơng nhiều phải tính tốn với giá trị không nguyên Và hầu hết thiết bị, giá trị biểu diễn, lưu trữ tính tốn dạngdấu chấm độngatheo tiêu chuẩn IEEE 754 Trong này, tìm câu trả lời cho câu hỏi sau:

1 Dấu chấm động gì?

2 Tiêu chuẩn IEEE 754 định nghĩa dạng liệu dấu chấm động máy tính nào?

3 Máy tính thực phép tính cộng, trừ, nhân, chia với dấu chấm động nào?

afloating-point numbers

1 Dấu chấm động gì?

Trong tính tốn hay ước lượng sống, giá trị mà quan tâm lớn, nhỏ, độ xác mà ta cần dùng thường cố định, thay đổi Vì thay ghi nhiều chữ số123456789101112000000000, hay0,000000123456, có

thể đưa chúng dạng chuẩn hóa:109·1,23456789101112≈109·1,234568và10−7·1,23456

Khi đấy, độ lớn nhỏ giá trị ước lượng số mũ số 10 (9hay−7) độ

chính xác cần thiết thấy thơng qua số chữ số phần định trị1,234568

Cho số thựcxvà sốb >1cố định Chúng ta biểu diễnxdưới dạng chuẩn hóa với

cơ sốbnhư sau:

x= (−1)s·be·c,

trong

• s= 0,1xác định dấu củax,

(27)

• clà phầnđịnh trị1 củaxthỏa mãn điều kiện1≤c < b

Mỗi số thực x tương ứng với ba giá trị (s, e, c) với sốb xác định trước

chính điều kiện ≤ c < b Vì vậy, ta muốn lưu trữ giá trị củax,

dùng ba(s, e, c)tương ứng vớix Cách lưu trữ giá trị củaxnày máy tính gọi dấu chấm động(dấu phẩy động)

Từ cơng thức dạng chuẩn hóa, ta có |e−logb(x)| < Vì thế, số lượng chữ số cần

thiết để lưu trữ ba(s, e, c)nhỏ lưu trữ chữ số củaxrất nhiều khixcó thể lớn

hoặc bé, độ xác cần thiết khơng q lớn Ví dụ cần phải lưu trữ hai số123456789101112000000000và0,000000123456với độ xác 6-7 chữ số Nếu

như sử dụng dấu chấm tĩnh, cần đến 24 + 12 = 36 chữ số cho phần nguyên phần thập

phân Trong đấy, sử dụng dấu chấm động (với số 10), cần 1-2 chữ số cho số mũ, chữ số cho phần định trị, nhiều so với 36 Hay nói cách khác, với dung lương lưu trữ, dấu chấm động cho phép biểu diễn số khoảng rộng nhiều với độ xác xác định trước

Điểm yếu lớn cách lưu trữ số dạng dấu chấm động phép toán trở nên phức tạp nhiều so với số nguyên dấu chấm tĩnh, kể phép tính cộng trừ Điều khiến cho việc tính tốn với dấu chấm động chậm nhiều khơng có hỗ trợ tăng tốc từ phần cứng

2 Tiêu chuẩn IEEE 754 cho dấu chấm động

Vào năm 1960-70, việc tính tốn máy tính bất đầu trở nên phổ biến, nhiều công ty tham gia thiết kế sản xuất máy chủ lớn, kèm theo phần cứng hỗ trợ tính tốn dấu chấm động Tuy nhiên, công ty lại theo đuổi định dạng dấu chấm động riêng mình, đơi dòng máy khác hệ dùng định dạng dấu chấm động khác Điều dẫn đến mã chương trình tính tốn dấu chấm động khơng thể dùng lại cho máy tính khác nhau, việc chia sẻ liệu dấu chấm động máy phải qua chương trình chuyển đổi phần mềm chậm chạp Hơn nữa, định dạng dấu chấm động khác lại có khoảng biểu diễn khác nhau, dẫn đến sai số liệu với chuyển đổi

Vì việc chuẩn hóa định dạng dấu chấm động máy tính trở nên cần thiết để giảm bớt ràng buộc vào phần cứng phần mềm liệu, qua giúp việc trao đổi liệu tái sử dụng mã chương trình trở nên hiệu Nhu cầu dấn đến đời tiêu chuẩn IEEE 754 vào năm 1985 (còn viết IEEE 754-1985) quy định định dạng phép tính tốn với dấu chấm động máy tính Tiêu chuẩn sau mở rộng cập nhật thêm hai lần vào năm 2008 (IEEE 754-2008) gần 2019 (IEEE 754-2019) Tiêu chuẩn IEEE 754 chuẩn hóa định dạng dấu chấm động cho số (binary) 10 (decimal) Trong tập trung vào dấu chấm động số 2, dạng hỗ trợ phổ biến hầu hết máy tính Định dạng dấu chấm động vớixx-bit thường đặt tên

thức làbinaryxxsử dụng

(28)

• bit làm bit dấus,

• le bit làm bit số mũe,

• lcbit cịn lại làm bit cho phần định trịc

Nếu xem bit số mũe (vàc) số nguyên dương,

khơng thể biểu diễn số nhỏ 1và gần với0 Vì để biểu diễn

số nhỏ lẫn lớn, số mũ thật củaxở dạng chuẩn hóa quy định làe−biasvới số

hiệu dịchbiasđược xác định trước

Các giá trị tham số cho số dấu chấm động16,32,64, và128-bit theo tiêu chuẩn IEEE 754

là sau:

Số bit sử dụng Tên thức Tên thường gặp le lc bias

16 binary16 half precision, half 10 15 = 24−1

32 binary32 single precision, single, float 23 127 = 27−1

64 binary64 double precision, double 11 52 1023 = 210−1

128 binary128 quadruple precision, quad 15 112 16383 = 214−1

Khi bit củaekhông phải tồn0hay tồn1, ba(s, e, c)là dạng chuẩn hóa giá trị thực

là:

x= (−1)s·2e−bias· + 2−lc ·c.

Lưu ý đến phần1+ở phần định trị Vì viết số nhị phân dạng chuẩn hóa, phần định trị

ln bắt đầu bằng1.xxxxx, nên lưu trữ số nhị phân dạng chuẩn hóa, bit

ẩn để tiềt kiệm nhớ, cho phép lưu trữ phần định trị xác thêm bit đằng sau

Ví dụ ba(0,128,4194304)viết dạng nhị phân (32-bit): 0|1000 0000|100 0000 0000 0000 0000 0000

sẽ có giá trị thật là:

x= (−1)0·2128−127· + 2−23·4194304=

Trường hợp đặc biệt bit củaetoàn là1được chia làm trường hợp con:

• Nếu nhưc= 0, ba(s, e,0)sẽ biểu diễn giá trị âm dương vô (±∞)

tùy vào dấus

• Nếu nhưc6= 0, ba(s, e, c)được gọi NaN (Not-A-Number, số)

Khi bit củaetoàn là0cũng chia làm trường hợp con:

• Nếu c = 0: ba (0,0,0) biểu diễn giá trị +0, ba (1,0,0) biểu diễn giá

trị −0 Về mặt giá trị +0 −0là nhau, nghĩa tiêu chuẩn quy định phép so

sánh+0 = −0trả giá trị đúng, và+0 6= −0trả giá trị sai Tuy nhiên kết

(29)

• Nếu nhưc6= 0: ba(s,0, c)được gọi số chuẩn2và có giá trị thực tính theo

công thức:

x= (−1)s·21−bias−lc·c.

Với số thực khơng thể biểu diễn xác với định dạng dấu chấm động lựa chọn, tiêu chuẩn IEEE 754 quy định phương thức làm tròn sau:

• Làm trịn - loại bỏ tồn chữ số thừa • Làm trịn lên (về+∞)

• Làm trịn xuống (về−∞)

• Làm trịn đến số gần - trường hợp giữa, làm tròn số định trị chẵn gần

nhất

Trong bốn phương thức làm tròn này, phương thức làm trịn gần nhất, làm tròn định trị chẵn gần (RTNE3) phương thức làm trịn mặc định, hầu hết tính

tốn máy tính dùng phương pháp làm trịn

Ví dụ muốn tìm cách biểu diễn sốx= 4098dưới dạng binary16

• Đầu tiên ta chuẩn hóax= 212· + 2−11

• Phần số mũe = 12 +bias = 12 + 15 = 27, biểu diễn dạng nhị phân với bit 11011

• Phần định trị viết dạng nhị phân là1,000 000 000 01sẽ làm tròn về1,000 000 000

• Như vậy, sốx= 4098sẽ lưu trữ dạng binary16 thành 0|11011|0000000000

và đổi ngược lại thành giá trị4096

Tương tự vậy, sốx= 4100sẽ biểu diễn dạng binary16 thành 0|11011|0000000010

có giá trị là4102

2subnormal/denormal numbers

(30)

3 Tính tốn với dấu chấm động - Cộng Trừ

Vậy làm để cộng (hay trừ) hai số biểu diễn dạng dấu chấm động IEEE 754? Vớix= (s1, e1, c1)vày= (s2, e2, c2), xét đến trường

hợp đặc biệt:

• Nếu nhưxhoặcylà NaN, kết củax+yvàx−yđược quy định làN aN

• Các trường hợp(+∞) + (−∞),(+∞)−(+∞),(−∞)−(−∞)cũng quy định cho

ra kết NaN

• Nếu nhưx=y=±0,x+y= (−1)min{s1,s2}·0; hay nói cách khác,−0 +−0 =−0,

lại kết là+0

Với cảx yđều dạng chuẩn, biến đổi x vày số mũ,

cộng trừ phần định trị sau biến đổi Giả sử nhưe1 ≥ e2, phần định trị1 + 2−lc ·c1

chúng ta viết gọn lại thành1, c1 Khi ta có được: x+y= (−1)s1 ·2e1−bias·1, c

1+ (−1)s2 ·2e2−bias·1, c2 = 2e1−bias· (−1)s1 ·1, c

1 + (−1)s2 ·2e2−e1 ·1, c2

Hay nói cách khác, sau xếp bit củaxvày thẳng hàng, việc cộng,

trừ phần định trị, thực làm tròn chuẩn hóa lại kết

Về mặt bản, cách mà số dấu chấm động dạng binary32 binary64 cộng trừ bên CPU máy tính Tuy nhiên thuật tốn tính cộng vấp phải vấn đề là, để xếp bit củaxvàythẳng hàng, phải dùng nhiều bit

Như với trường hợpxvà yđều binary64, phải cần đến 2000 bits, trở

lại vấn đề ban đầu việc sử dụng dấu chấm tĩnh

Để giải vấn đề này, có hai điều đáng ý sau giúp giảm bớt số bit cần dùng cho phép cộng trừ:

• Nếu

2 ≤f racxy ≤2, cần nhiều làlc+ 3bit để tính tổng phần

định trị sau biến đổi số mũ

• Nếu |x| > 2· |y| |y| > 2· |x|, kết

2max{|x|,|y|} < |x+y| <

2max{|x|,|y|}

Trong trường hợp thứ 2, kết x+y nằm quanh quẩn số có trị tuyệt đối lớn

giữaxvày Giả sử số có trị tuyệt đối lớn làx Để làm tròn kết x+y, 1-2 bit thêm vào saux, bit nhỏ theo sau quan trọng tất

(31)

chúng ta tính trực tiếp bit củaysẽ chuyển sau xhơn bit mà không

cần phải dịch chuyểny Sau dịch chuyểny, cần quan tâm đến tối đa bit

saux Vì vậy, tổng số bit mà cần để thực phép cộng trừ định trị sau

biến đổi vào khoảnglc+ 4, nhỏ nhiều so với tồn khoảng vài nghìn bit trước Bài tập 1. Giả sử muôn làm tròn theo phương pháp mặc định sốx, chúng

ta có bit cuối vị trí cần làm trịn củaxlàx0, bit sau làx−1, vàz =

nếu bit lại theo saux−1 0, khơng thìz = Xét tất trường hợp

củax0, x−1 vàzvà tìm kết làm trịn tất trường hợp

Bài tập 2. Dùng tập đoạn thảo luận trước để hoàn thiện thuật tốn tính tổng

x+ycho số dấu chấm động dương dạng chuẩnxvày

Bài tập 3. Mở rộng thuật toán để xét trường hợpxvà ytrái dấu, hay nói cách khác, tính

trừ

Bài tập 4. Lập trình thuật tốn ngơn ngữ ưa thích bạn kiểm tra thử kết lại với kết CPU

Tài liệu

[1] ttps://en.wikipedia.org/wiki/IEEE_754 [2] ttps://ieeexplore.ieee.org/document/4610935

(32)

ĐƠN ĐỒ THỊ VÔ HƯỚNG TRONG ĐỀ CHỌN ĐỘI

TUYỂN CÁC TỈNH/THÀNH PHỐ

2019 - 2020 Nguyễn Tuấn Anh

(Trường THPT chuyên Nguyễn Quang Diêu - Đồng Tháp)

GIỚI THIỆU

Những ý tưởng ban đầu lý thuyết đồ thị nhà Toán học Leonhard Euler trình bày vào kỷ XVIII báo v bi toỏn By chic cu Kăonigsberg

vo năm1973:aĐến thời điểm tại, Lý thuyết đồ thị ứng dụng vào nhiều lĩnh vực

khác nhaub, Tốn điều thấy rõ ràng Nhằm góp phần thể vai

trị viết đến với số toán chọn đội dự thi VMO năm học2019 2020với lời giải dựa vào đồ thị (chính xác đơn đồ thị vơ hướng) Có hai

điều sau xin lưu ý bạn đọc

Trước hết, khái niệm liên quan đếnLý thuyết đồ thịsẽ có đơi chút khác biệt tùy vào mục đích lĩnh vực áp dụng Vì vậy, để bạn đọc dễ theo dõi người viết chọn khái niệm tài liệuTheory and Application of Graphs [1] khơng trình bày lại Ban đọc cần thiết tra cứu lại

Vì tốn giải quan điểm đồ thị nên có lời giải tương đối khơng gọn thay cách phương pháp giải khác:Sử dụng song ánh, đếm hai cách, Như bạn đọc nên tìm lời giải khác, đường học hiệu

Mọi đóng góp cho viết bạn đọc gửi địa chỉ:anh110004@gmail.com aNhưng đến kỷ XIX người ta quay lại với chủ đề với nghiên cứu

về mạng điện, mơ hình tinh thể cấu trúc phân tử chất Sự phát triển logic hình thức đẩy đến việc nghiên cứu quan hệ hai ngơi dạng đồ thị Sau nhiều toán khác phát triển ngơn ngữ đồ thị

bChính điều mà chương trình đổi giáo dục nội dungLý thuyết đồ thịđược đưa

(33)

1 Một số tốn

Ví dụ (Khánh Hịa - 2019, ngày 1) Một nhóm phượt cónthành viên Năm 2018 họ thực hiện 6chuyến du lịch mà chuyến có đúng5thành viên tham dự Biết hai chuyến du lịch bất kỳ có tối đa hai thành viên chung Tìm giá trị nhỏ củan

Lời giải Gọinthành viên tham dự làX1; X2; : : : ; XnvàY1; Y2; : : : ; Y6là sáu chuyến phượt

Xét đồ thị hai phe với tập đỉnh làX[Y đó:

X D fX1; X2; : : : ; XngvàY D fY1; Y2; : : : ; Y6g:

Hai đỉnhXi; Yj nối cạnh với ngườiXi tham gia chuyến phượtYj

Theo giả thiết đồ thị có65D30cạnh GọiS tập cặp hai cạnh có đỉnh thuộc X Ta đếmjSj

Đếm theo đỉnh tậpY Theo giả thiết với hai đỉnh thuộcY nối cạnh chung tối đa2

đỉnh thuộcX Do đójSj62C2

Đếm theo đỉnh thuộc tậpX Gọidi Ddeg.Xi/(khi ta cóPdi D30) Khi với

đỉnhXi ta cóCd2

i cách chọn ra2đỉnh

1 thuộcY cùng liên thuộc cạnh vớiX i

Suy

jSj DXCdi2 DXd

i di

2 :

Áp dụng bất đẳng thức Cauchy ta có

.d1Cd2C Cdn/2

n d1Cd2C Cdn/

2

Xdi2 di

2 D30:

Từ ta n > 10:Để khẳng địnhn D 10là giá trị nhỏ ta cần với10người có

thể xếp6chuyến du lịch thỏa giả thiết Xem lại cách chứng minh, thấy đẳng thức xảy

và khidi D3với mọii D1; 2; : : : ; 10 Như ta cần xây dựng đồ thị hai phe có 30cạnh, đỉnh củaXcó bậc là3, đỉnh củaY có bậc là5(xem hình vẽ)

1Ta quy ướcC2

(34)

X1 X2

X3 X4 X5 X6 X7 x8 X9 X10

Y1 Y2 Y3 Y4 Y5 Y6

Lời giải hoàn tất

Nhận xét Cùng với ý tưởng xây dựng đồ thị hai phe đếm số cặp hai cạnh chung đỉnh, ta có tốn sau (mặc dù mặt diễn đạt chúng khác nhau)

Ví dụ (Lào Cai - 2019, ngày 1) Trên mặt phẳng cho tậpAgồmnđiểm phân biệt (vớinlà số nguyên dương) vàB là tập gồm14đường thẳng phân biệt Biết đường thẳng tậpB đi qua đúng14điểm tậpA

a) Gọi tất điểm tậpAP1; P2; : : : ; Pn Với mỗiPi gọiai là số đường thẳng của tậpBđi quaPi Chứng minh rằng

n X

iD1

ai D196:

b) Chứng minh rằngn>102:

Lời giải Gọil1; l2; : : : ; l14là14đường thẳng thuộc tậpB Xét đồ thị hai phe với tập đỉnh

phân chia thành hai tập làAvàB Một đỉnhPi nối cạnh vớilj đường thẳng lj qua điểmPi

a) Theo giả thiết tốn đồ thị có1414D196cạnh Từ suy n

X

iD1 D

n X

iD1

deg.Pi/D

1962

2 D196:

n X

iD1

deg.Pi/D 14 X

iD1

deg.li/; n X

iD1

deg.Pi/C 14 X

iD1

deg.li/D2196:

(35)

Đếm theo đỉnh A Vì đỉnhPi cóCa2i cách chọn 2đường thẳng nhận làm

đỉnh.2Do đó:

jSj D n X

iD1 Cai2 D

n X

iD1

a2i :

Đếm theo đỉnh củaB Theo giả thiết ta có hai đỉnh củaB có tối đa1đỉnh chung (vì

có từ hai điểm chung trở lên hai đường thẳng trùng nhau) Do đójSj6C142:

Như ta có bất đẳng thức

n X

iD1

ai2 6C

2 14:

Áp dụng bất đẳng thức Cauchy, ta có

.a1Ca2C Can/2

n a1Ca2C Can/

2

n X

iD1

a2i 6C

2 14:

Theo câu.a/thìa1Ca2C CanD196nên ta 1962

n 196

2 6C

2

14 D91;

hayn>102(vìnnguyên dương) Bài toán chứng minh

Nhận xét

Khác với toán thứ nhất, toán ta nhận đẳng thức khơng thể xảy

trong đánh giá n > 102 Một vấn đề đặt nhỏ để tồn tạinđiểm thỏa mãn

bao nhiêu? Phần mời bạn đọc khai khác

Một cách đếm đơn giản khác (thu đánh giá tốt hơn): Đánh ký hiệu14đường thẳng

làl1; l2; : : : ; l14 Khi xét đường thẳngl1, đường thẳng cần có14điểm

Xét đường thẳng l2 đường có tối đa điểm chung với l1 nên ta cần có thêm 13

điểm

Xét tiếp đường thẳngl3, đường thẳng có điểm chung với2đường trước tối đa

điểm ta cần có thêm14 3C1D12điểm

Tiếp tục q trình trên, ta cần nhất14C13C C2C1 D 105điểm để thỏa

mãn, tứcn>105

Ưu điểm lời giải tốn Tổ hợp cách sử dụng đồ thị (vơ hướng có

hướng) khơng nằm chỗ có nhiều định lý, tính chất áp dụng mà hết thấy hình dáng chung nhiều tốn dù có cách phát biểu bề ngồi khác Các tốn tương tự thể điều đó, mời bạn đọc làm thử

2Ta quy ướcC2

(36)

Bài tập (Chuyên ĐH Vinh - 2019) 16 học sinh tham gia làm thi trắc nghiệm Đề thi chung cho tất học sinh có ncâu hỏi, câu hỏi có 4phương án trả lời Sau thi xong, thầy giáo nhận thấy với câu hỏi, học sinh chọn đúng 1phương án trả lời hai học sinh có nhiều nhất 1câu hỏi có phương án trả lời giống nhau.

a) VớinD2hãy ví dụ phương án trả lời câu hỏi của16học sinh. b) Chứng minh rằngn65

Bài tập (Cần Thơ - 2019, IMO Shortlist - 2004) Trong trung tâm văn hóa tỉnh có

501học sinh tổ chức CLB (một học sinh tham gia nhiều CLB) Các CLB phối hợp với để tổ chức hoạt động xã hội Biết cókhoạt động xã hội thỏa mãn các điều kiện:

i) Mỗi cặp học sinh thuộc đúng1CLB.

ii) Với học sinh hoạt động xã hội, học sinh thuộc đúng CLB trong hoạt động xã hội tương ứng.

iii) Mỗi CLB có số lẻ thành viên số thành viên là2mC1thì số hoạt động xã hội làm

Tính tất giá trị có củak

Bài tập (Trung Quốc - 1996, ngày 2) 8ca sĩ tham gia chương trình văn nghệ vớimbuổi hịa nhạc Trong buổi hịa nhạc, có bốn ca sĩ tham gia số lần tham gia của cặp ca sĩ bằng n(nlà số nguyên dương) Tính giá trị nhỏ nhất củam

Bài tập (VMO - 2005) Cho bát giác lồiA1A2 A8khơng có ba đường chéo đồng quy Giao hai đường chéo tùy ý gọi nút Xét tất tứ giác lồi được tạo thành từ bốn đỉnh bát giác cho tứ giác gọi tứ giác Hãy xác định số nguyên dương nnhỏ cho tơ màunnút để với mọii; k phân biệt thuộc f1; 2; : : : ; 8g thì số S.i; k/ bằng nhau, đó S.i; k/ký hiệu tứ giác nhận Ai; Ak làm đỉnh giao hai đường chéo nút tơ màu.

Ví dụ (PTNK - 2019) Một trường phổ thơng cónhọc sinh Các học sinh tham gia vào nhiều câu lạc khác có tất cảm câu lạc Giả sử câu lạc có đúng4học sinh và với hai học sinh tham gia chung với tối đa câu lạc Chứng minh rằng

m n.n 1/ 12 :

Lời giải GọiAlà tập học sinh,B tập câu lạc Xét đồ thị hai phe có tập đỉnh

phân chia làAvà B Hai đỉnhAi AvàBj 2B nối cạnh với nếuAi

một học sinh tham gia câu lạc bộBj 2B

Để chứng minh bất đẳng thức toán, ta đếm số cạnh đồ thị

(37)

Ta chứng minh kết sau:

GọiA12 Alà đỉnh có nối cạnh sangB với số lượng cạnh nhiều (nếu có nhiều đỉnh như vậy, ta chọn số chúng) số cạnh nhậnA1là đỉnh khơng vượt q n 13 .

Thật số lượng cạnh nhiều n

3 Giả sử tập đỉnh mà A1 nối cạnh B0 D fB1; B2; : : : ; Bkgvớik > n 13 vàA2; A3; A4là ba đỉnh khác nối cạnh vớiA1

đếnB1:Khi đó, với đỉnhB2; B3; : : : ; Bk

Ta nối cạnh thêm với đỉnh khác vớiA2; A3; A4 (vì thân đỉnh B2; B3; : : : ; Bk nối cạnh vớiA1) Ta gọi gọiA0DAnfA1; A2; A3; A4g

Nếu đỉnh thuộcA0nối vớiBi B0 khơng nối với phần tử

cịn lại củaB0.

Do số đỉnh phân biệt thuộcA0cần để nối cạnh đến đỉnh thuộcB0thỏa giả thiết 3.k 1/ >

n 1

3

Dn 4:

Điều số đỉnh tậpAsẽ lớn hơnn Chứng minh hoàn tất

Từ kết trên, số cạnh đồ thị không vượt n.n 1/

3 :

Do vậym6 n.n 1/12 Bài toán chứng minh

Nhận xét Bài toán sử dụng đồ thị đếm cặp cạnh theo hai cách ví dụ

1; Phần xin dành cho bạn đọc

Ví dụ (Phú Thọ - 2019, ngày 2) Có nhóm người mà đó, cặp khơng quen nhau

có hai người quen chung, cặp quen khơng có người quen chung Chứng minh số người quen người nhau.

Lời giải Giả sử nhóm cónngười làA1; A2; : : : ; An Xét đồ thị có tập đỉnh tậpnngười

trong nhóm Hai đỉnh nối cạnh với hai người tương ứng hai đỉnh quen

Xét hai đỉnhA1; A2 thuộc đồ thị GọiN.A1/; N.A2/là hai tập đỉnh liền kề vớiA1; A2

Ta chứng minhjN.A1/j D jN.A2/j Ta có hai trường hợp sau:

Trường hợp 1:A1và A2là hai đỉnh liền kề Khi đóN.A1/nfA2gvàN.A2/nfA1glà hai tập rời

nhau (hai người quen khơng có người quen chung)

Trường hợp 1.1: Nếu N.A1/nfA2g D ; N.A2/nfA1g D ; ngược lại giả sử C1 N.A2/nfA1g C1; A1 hai người khơng quen có A2

người quen chung

(38)

Trường hp 1.2: Nu N.A1/nfA2g Ô ;, gi s N.A1/nfA2g D fB1; B2; : : : ; Blg (với l N) Khi đóN.A2/nfA1gcũng khác ; (vì N.A2/nfA1g D ;thì theo lập luận

trênN.A1/nfA2g D ;), giả sửN.A2/nfA1g D fC1; C2; : : : ; Ckgvớik2N

XétB1, vìB1vàA2là không liền kề Suy tồn hai đỉnh khác nối cạnh với B1 vàA2(trong có sẵngA1) Như tồn nhấtCi N.A2/nfA1gsao cho B1nối cạnh vớiCi Thực tương tự với cácBi khác, chỳ ý rng khụng th cúBi vBj

(vii Ôj) nối sang đỉnh củaN.A2/nfA1g Từ ta đượcjN.A1/j6jN.A2/j

A1 A2

B1

B2 B l

Ck

C2

C1

Áp dụng cách lập luận cách đổi vai trò hai tậpN.A1/vàN.A2/ta jN.A2/j6jN.A1/j VậyjN.A1/j D jN.A2/j

Như vậy, trường hợp ta đượcjN.A1/j D jN.A2/j

Trường hợp 2: A1 vàA2 khơng nối cạnh Khi tồn đỉnh, giả sử A3 nối cạnh

với hai đỉnh Theo trường hợp1số đỉnh liền kề vớiA1; A2bằng với số đỉnh liền kề vớiA3

A1 A2

A3

Tóm lại, số đỉnh liền kề hai đỉnh tùy ý thuộc đồ thị nhau, hay nói cách khác hai người tùy ý nhóm có số lượng người quen

Nhận xét

Trường hợpjN.A1/j D jN.A2/j D1thật xảy nhóm có hai người

và hai người quen Và trường hợp nhiều học sinh lập luận bỏ qua, chưa đảm bảo tính chặt chẽ

Cách lập luận chứng minh jN.A1/j D jN.A2/jbản chất xây dựng song ánh hai

(39)

Trường hợp2có thể lập luận trực tiếp hình vẽ sau (dành cho bạn đọc)

A1 A2

B1

B2 B l

Ck

C2

C1

A3 A4

Một toán tương tự: (APMO - 1990) Trong đồ thị không chứa tam giác và không có đỉnh nối cạnh với tất các đỉnh khác Biết hai đỉnh bất kỳA; B nếu khơng liền kề có đỉnhC sao choC liền kề với cảAB:Chứng minh rằng tất đỉnh có bậc nhau.

Ví dụ (Hưng Yên - 2019, ngày 1) Cho tậpX khác rỗng chia thành tập đôi một không giao nhauA1; A2; : : : ; An và đồng thời chia thành tập đôi một không giao nhau B1; B2; : : : ; Bn Biết hợp hai tập hợp không giao nhauAi; Bj (với 6i; j 6n) có khơng hơnnphần tử Chứng minh số phần tử củaX khơng hơn n22. Khi số phần tử củaX n22 hãy cách chia tập hợp thỏa mãn toán.

Lời giải Giả sửX D fx1; x2; : : : ; xmgvà gọiAD fA1; A2; : : : ; Ang Xét đồ thị hai phe có

tập đỉnh phân chia X; A Mỗi đỉnhxi X nối cạnh với đỉnh Aj Anếu xi Aj Dễ dàng nhận thấy số cạnh đồ thị bằngjXj D m(đếm theo đỉnh X)

Ta đếm số cạnh theo đỉnh thuộcA Trước hết ta giả sử hai điều sau:

jB1j DminfjA1j;jA2j; : : : ;jAnj;jB1j;jB2j; : : : ;jBnjg(nếu tập thỏa mãn làAi

đó ta gọi lại tậpAD fB1; B2; : : : ; Bng)

jB1j Dp n2 Vì ngược lại tốn hiển nhiên

Khi ta phân chia tậpAthành hai tập đỉnh là:

A0 tập đỉnh mà thân tập có phần tử chung vớiB1 Dễ thấy số đỉnh

vậy không vượt quáp n

A00là tập đỉnh mà thân tập khơng có phần tử chung vớiB1

Giả sử

A0 D fAi1; Ai2; : : : ; AilgvàA

00 D fA

ilC1; AllC2; : : : ; Aing

(40)

B1

A0

A00

>lpcạnh

>.n l/.n p/cạnh x1

x2

x3

xp

xpC1 xpC2 xm xm xm

Ai1

Ai2

Ai3

Ail

AilC1 AilC2 Ain Ain

Số cạnh đồ thị có đỉnh thuộcA0ít làlpvì X

Ai2A0

deg.Ai/>lp:

Theo giả thiết hai tậpAi; Bj khơng giao có nhấtnphần tử Áp dụng điều

choAij (vớij DlC1; lC2; : : : ; n) vàB1 Suy deg.Aij/>n p Hay nói cách khác

số cạnh đồ thị có đỉnh thuộcA00 là.n l/.n p/

Vậy số cạnh đồ thị là.n l/.n p/Clp Như ta được: mD jXj >.n l/.n p/Clp

>n2 n.l Cp/C2lp D n

2

2 C2 n

2 l n p

> n2

2

Đẳng thức xảy khipD n2 hoặcl D n

2 (trường hợp kéo theop D n 2) Tóm

lại đẳng thức xảy khip D n2 Dựa vào ta xây dựng đồ thị hai phe

thỏa yêu cầu lại toán Phần xin dành cho bạn đọc

Ví dụ (Quảng Bình - 2019, ngày 2) Cho số nguyên dương n > 2, xét2n điểm phân biệt trong mặt phẳng cho khơng có3điểm thẳng hàng Trong số điểm ta nốin2C1 cặp điểm lại với Chứng minh rằng:

(41)

Lời giải Xét đồ thị có2nđỉnh là2nđiểm cho Hai đỉnh nối cạnh với hai

điểm nối đoạn thẳng sốn2C1đoạn nối

a) Theo định lý Mantel, số cạnh tối đa đồ thị không chứa tam giác làh.2n/2

i

Dn2 Do

đó vớin2C1cạnh đồ thị chứa tam giác

b)Cách 1:Ta chứng minh quy nạp

Với n D 2dễ dàng kiểm tra Giả sử khảng định vớin, ta chứng minh với nC1 Xét đồ thị2.nC1/đỉnh với nC1/2 C1 cạnh Theo định lý Mantel tồn tam

giác, giả sử ba đỉnh tam giác A; B; C GọiN.A/; N.B/; N.C / tập chứa đỉnh

lần lượt liền kề vớiA; B; C tập2n 1đỉnh cịn lại Khi số tam giác tạo thành T D jN A/\N B/j C jN B/\N C /j C jN A/\N C /j:

Theo nguyên lý bù trừ thì:

T D jN A/j C jN B/j C jN C /j C jN A/\N B/\N C /j jN A/[N B/[N C /j

>jN A/j C jN B/j C jN C /j jN A/[N B/[N C /j

>jN A/j C jN B/j C jN C /j 2nC1:

VìjN A/[N B/[N C /j62n Ta có hai trường hợp sau:

Trường hợp 1: VớijN A/j C jN B/j C jN C /j 2nC1>nthì khẳng định

với nC1vì ta cóT > nkết hợp thêm ABC tam giác ta có nC1tam giác

Trường hợp 2: VớijN A/j C jN B/j C jN C /j 2nC1 < n;hay jN A/j C jN B/j C jN C /j< 3n 1:

Vì vai trị ba số jN A/j;jN B/j;jN C /j bình đẳng Do ta giả sử jN A/j C jN B/j< 2n Suy rajN A/j C jN B/j 62n 2:

Xét đồ thị thu từ đồ thị ban đầu cách bỏ hai đỉnhA; B cạnh nhận A; B đỉnh (tính cảAB; BC; CA) Khi đồ thị thu có2nđỉnh với số cạnh

nhất

(42)

N.B/

N.A/

N.C /

C

B

A

Theo giả thiết quy nạp đồ thị chứa nhấtntam giác Suy đồ thị ban đầu có

ít nhấtnC1tam giác

Theo ngun lý quy nạp ta điều phải chứng minh

Cách 2:Ta chứng minh quy nạp kết tổng quát sau:

Với đồ thị cójVj D 2nCx vớin>2ngun vàx f0; 1gvà cójEj D n.nCx/C1thì đồ thị có nhấtntam giác.

Trước tiên ta thấy đồ thị lúc ln có đỉnh có bậc khơng vượt qn Thật

vậy, đỉnh có bậc từnC1trở lên ta có: nC1/.2nCx/D.nC1/jVj6 X

X2V

deg.X /D2jEj D 2n.nCx/C2:

Điều kéo theo

.nC1/.2nCx/62n.nCx/C2,.n 1/ x 2/>0:

Bất đẳng thức khơng vìn>2; x 61:

Với mỗin>2là số nguyên dương vàx f0; 1gta gọiP n; x/là mệnh đề có dạng:

Với đồ thịG.V; E/jVj D2nCx vớin> 2ngun cójEj D n.nCx/C1thì đồ thị có nhấtntam giác.

Trường hợp 1: VớixD1, ta giả sửP n; 0/là ĐặtX đỉnh có bậc khơng vượt q n(theo nhận xét trênX tồn tại) Xét đồ thịG0thu từGbằng cách bỏ điX

cạnh liên thuộc vớiX Khi đồ thịG0có2nđỉnh có số cạnh n.nC1/C1 degG.X />n.nC1/C1 nDn.nC0/C1:

(43)

Trường hợp 2: Với x D 0, ta giả sửP n 1; 1/đúng ĐặtX đỉnh có bậc khơng vượt

qn(theo nhận xét trênX tồn tại) Xét đồ thịG0thu từG cách bỏ điX

các cạnh liên thuộc vớiX Khi đồ thịG0có2n 1D2.n 1/C1đỉnh có số cạnh là:

nnC1 degG.X />nnC1 nD.n 1/.n 1C1/C1:

VìP n 1; 1/đúng nên đồ thịG0chứa nhấtn 1tam giác

Ta lại có:

Trường hợp 2.1: Nếu degG.X / < nthì số cạnh củaG0là lớn hẳn n 1/.n 1C1/C1cạnh

VìP n 1; 1/đúng nênG0chứa nhấtn >1tam giác GọiABC tam

giác

Xét đồ thị mớiG00 củaG0thu cách xóa bớt cạnh AB Khi đóG00

vẫn có2.n 1/C1đỉnh có nhất.n 1/.n 1C1/C1cạnh Do đóG00có

nhấtn 1tam giác Hiển nhiên tam giác tam giác thuộc đồ thịG0

Kết hợp thêm tam giácABC ta đượcG0 thật có làntam giác Suy G

có nhấtntam giác

Trường hợp 2.2: Nếu degG.X /Dn, ta gọiN.X /là tập đỉnh liền kề vớiX GvàN.X /là tậpn 1đỉnh lại

Nếu có hai đỉnh trongN.X /là liền kề hai đỉnh kết hợp với X tạo thành tam giác (đương nhiên tam giác không trùng với

tam giác trongn 1tam giác củaG0) Do đóG có nhấtntam giác Ngược lại, N.X / khơng có hai đỉnh liền kề Khi đỉnh Y

N.X /thì degG0.Y /6n

Nếu có đỉnhZ N.X /sao cho degG0.Z/ 6n 2thì degG.Z/6n

Khi ta thay thếX choZ Áp dụng trường hợp 2.1 ta đượcGchứa n

tam giác

Ngược lại, đỉnhY 2N.X /thì degG0.Y /Dn Như số cạnh đồ thịG0có đỉnh thuộcN.X /và đỉnh cịn lại thuộcN.X /chỉ là

n.n 1/ < n.n 1/C1:

(ta ý thêm hai đỉnh thuộcN.X /không nối cạnh trường hợp

này) Khi tồn cạnh nối hai đỉnh củaN.X / Cạnh kết hợp

tất cạnh nối từ đỉnh thuộcN.X /ta đượcntam giác.3

Tóm lạiP n; 0/đúng

Để hoàn tất chứng minh, ta kết nối lại bước chứng minh sau (một kiểu quy nạp theo hai biến):

3Đồ thị thu từG0 bằng cách bỏ tất cạnh có hai đỉnh thuộcN.X /là đồ thị hai pheK

n 1;n

(44)

P 2; 0/đúng

P 2; 1/đúng

P 3; 0/đúng

P 3; 1/đúng

P k; 0/đúng

P k; 1/đúng

P kC1; 0/đúng

P kC1; 1/đúng

Bước sở ban đầu tứcP 2; 0/có hay không quan trọng Kiểm tra trực tiếp đồ thị 4đỉnh có nhất5cạnh, ta thấyP 2; 0/đúng Chứng minh hoàn tất

Nhận xét

Một toán với giả thiết tương tự (ta sử dụng đồ thị kết hợp với quy nạp)

Bài tập (China TST - 1987) Cho đồ thị có2nđỉnh có đúngn2C1cạnh Chứng minh đồ thị chứa hai tam giác có chung cạnh.

Một đánh giá khác số tam giác có đồ có n đỉnh k cạnh (bạn đọc

chuyển đổi toán ngơn ngữ đồ thị để thấy điều đó)

Bài tập (APMO - 1989) ChoS là tập gồmm cặp.a; b/ với a; b là số nguyên dương thỏa16a < b 6n Chứng minh có nhất4m

m n24

3n bộ ba.a; b; c/sao cho.a; b/; a; c/.b; c/thuộcS.

Thay sử dụng đồ thị cách định lý liên quan để giải tốn Tổ hợp, ta cịn sử dụng kết định lý (không nên khó) đồ thị, sau diễn đạt theo cách tốn Tổ hợp thu hấp dẫn Vấn đề khó định lý chọn tương đối vừa phải, chuyển đổi toán phải tự nhiên, thực tiễn đặc biệt phải ẩn định lý chọn khéo léo tốt Để kết lại viết, ta đến với toán mang ý tưởng

Ví dụ (Trường đông BTB - 2019, ngày 1) Trong kỳ thi vấn đáp, có 64thí sinh và 6vị giám khảo Mỗi thí sinh phải trả lời với giám khảo nhận hai kết quả: “đạt – trượt” Biết với hai thí sinh bất kỳ, ln có vị giám khảo đánh giá thí sinh này đạt, cịn thí sinh trượt Sau kỳ thi, hai thí sinh có kết khác giám khảo kết bạn với (giả sử trước kỳ thi, chưa có bạn bè cả). a) Hỏi có tất cặp bạn bè nhau?

b) Chứng minh xếp tất thí sinh ngồi lên bàn trịn mà hai thí sinh ngồi cạnh nhau bạn bè nhau.

Lời giải Ta gọi6 vị giám khảo làX1; X2; : : : ; X6 xếp hàng ngang cố định

Với thí sinhYi (vớii D1; 2; : : : ; 64) ta ghi kết mà6vị giám chấm cho thí sinhYi theo

quy ước sau:

(45)

Ngược lại ta ghi số0:

Dễ dàng thấy kết của64thí sinh đầy đủ64xâu nhị phân có6ký tự

Gọi tập xâu nhị phân làA D fA1; A2; : : : ; A64g Khi ta xét đồ thị có tập đỉnh làA,

hai đỉnh Ai; Aj nối cạnh với hai xâuAi Aj sai khác

một vị trí Rõ ràng định nghĩa đồ thị lập phươngQ6 Một cách tổng quát, đồ thịQn4

có tính chất sau (bạn đọc tự chứng minh)

Số đỉnh là2n Số cạnh là2n 1n:

Đồ thịQncó chu trình Hamilton

Áp dụng kết ta lời giải cho toán

2 Bài tập tự luyện

Trong mục số tốn sử dụng đơn đồ thị vơ hướng để tìm lời giải (hiển nhiên khơng hướng tiếp cận nhất) Một số khơng ghi nguồn tốn diễn đạt lại số toán cũ người viết xin khơng trích nguồn, cốt yếu để bạn đọc (đặc biệt học sinh) luyện tập

Bài tập Cho6đường thẳng không gian cho không gian cho khơng có ba đường nào đồng thẳng Chứng minh tồn ba đường thẳng cho điều kiện sau xảy ra:

Hai đường ba đường hai đường thẳng chéo nhau. Ba đường thẳng đôi song song.

Ba đường thẳng đồng quy.

Bài tập Tìm số nguyên dương nhỏ nhấtnsao cho với tập gồmnsố vơ tỉ có ba số vơ tỉ mà tổng hai số ba số số vô tỉ.

Bài tập Chứng minh nhóm có9người (mỗi người nhóm quen nhau không quen nhau) tồn ba người quen tồn tại 4người đôi một không quen nhau.

Bài tập 10 Trong buỗi khiêu vũ, người khiêu vũ với người khác (chỉ một

lượt) khơng khiêu vũ (vì lý sức khỏe chẳng hạn), khiêu vũ nhiều lượt với những người khác Chứng minh với nhóm gồmnngười (n2ZC) ta ln xếp

một buổi khiêu vũ cho nhóm cho khơng có ba người có số lượt khiêu vũ.

(46)

Bài tập 11 (Trường Đông TP.HCM - 2018) Sau học trường mẫu giáo,20em nhỏ chờ ông nội ông ngoại đến đón Biết rằng 2em có chung người ơng Chứng minh có1ơng ơng nhất14em nhỏ.

Bài tập 12 (Vũ Thế Khôi, Trường đông BTB - 2019) Một tậpS gồm xâu nhị phân có độ dàin Hai xâu nhị phân gọi gần hai xâu sai khác một vị trí Giả sử xâu thuộcS gần với đúngkxâu thuộcS.

a) Chứng minh rằngjSjlà số chẵn. b) Chứng minh rằngjSj>2k.

Bài tập 13 (Mock Test - 2019) Với số nguyên dươngm; n, xét bảng ô vuôngmnđược phân chia thành ô vuông Giả sử tơ màu trăng - đen bảng cho với ơ vng đơn vị số vng màu có điểm chung với (khơng tính vng đó) số lẻ.

a) Chứng minh rằngmnlà số chẵn.

b) Giả sửmnlà số chẵn, chứng minh tơ màu bảng thỏa điều kiện đề bài.

Bài tập 14 (Hải Phòng - 2019) ChoXlà bát giác tâmO GọiAlà tập tất đỉnh củaX và giao điểm hai đường chéo củaX GọiY là tập8điểm thuộcAkhơng trùng O và gần O nhất (hình vẽ) Gọi Y là tập tất cạnh của X và đoạn thẳng nối hai điểm thuộcAkề đường chéo củaX Mỗi điểm thuộcAđược tơ một trong hai màu xanh đỏ Có tất cả26điểm đỏ Biết rằngO được tô đỏ, hai số những điểm đỏ đỉnh củaX, ba số điểm đỏ thuộcB

O

Các đoạn thẳng thuộc Y được tô màu theo quy tắc: đoạn thẳng có hai đầu mút đỏ nó được tơ màu đỏ, hai đầu mút xanh tơ màu xanh, đầu mút đỏ đầu mút xanh tơ màu vàng Biết có20đoạn thẳng trongY màu vàng Hỏi có bao nhiêu đoạn thẳng trongY màu xanh?

(47)

Bài tập 16 9giáo viên dẫn đoàn tham dự Olympic Trong thời gian chờ học sinh tham gia kỳ thi họ bắt đầu trò chuyện làm quen với Tuy nhiên, thời gian chờ học sinh tương đối ngắn nên người làm quen hết với Nếu hai người làm quen với nhau và họ thích chủ đề Tốn cuối buổi trị chuyện họ trao đổi email cho nhau (để sau trao đổi tài liệu chẳng hạn) Ngược lại trị chuyện làm quen kết thúc mà khơng có trao đổi email Hỏi cần có làm quen chắn rằng ln có3người làm quen với cuối buổi trị chuyện ba người khơng trao đổi email cho ba đôi trao đổi email cho nhau.

Tài liệu

[1] Junming Xu,Theory and Application of Graphs Springer US, 2003

[2] https://artofproblemsolving.com/community

(48)

ĐẲNG THỨC TỔ HỢP

QUA CÁC BÀI TOÁN OLYMPIC Trịnh Đào Chiến

GIỚI THIỆU

Trong số tốn Olympic, đơi việc biểu diễn tính tốn liên quan đến số tổ hợp đóng vai trị quan trọng để tiếp tục hoàn thiện lời giải, sau có định hướng Nhân việc trả lời thắc mắc số bạn học sinh vài đoạn biến đổi đẳng thức tổ hợp tài liệu [1] giới thiệu, viết tự nhiên hình thành Nội dung chủ yếu đề cập đến vai trò đẳng thức tổ hợp qua số toán Olympic, tiếp nối để chi tiết hóa nội dung tài liệu nêu mà tơi thích hệ thống tốn phong phú Cơng thức nội suy Lagrange

1 Các biểu diễn tính tốn liên quan đến số tổ hợp 1.1 Biểu diễn tích qua số tổ hợp

Bài toán 1.Giả sửnlà số nguyên dương,n ≥2, xác định trước Với mỗik ∈ {1,2, , n}, hãy biểu diễn tích sau qua số tổ hợp

n

Y

i=1,i6=k k−i

Giải.Với mỗik ∈ {1,2, , n},ta có n

Y

i=1,i6=k k−i =

1

(k−1) (k−2) 2.1.(−1) (−2) (−(n−k−1)).(−(n−k))

=

(k−1)!(−1)n−k(n−k)! = (−1) n−k

(k−1)! (n−k)! = (−1) n−k

k.n!

k.n! (k−1)! (n−k)! =

n!.(−1) n−k

.k n!

k! (n−k)! = n!.(−1)

n−k k.Cnk

(49)

Đẳng thức 1.

n

Y

i=1,i6=k k−i =

1 n!.(−1)

n−k k.Cnk

Bài toán 2.Giả sử nlà số nguyên dương,n ≥ 1, xác định trước Với mỗik ∈ {0,1,2, , n}, hãy biểu diễn tích sau qua số tổ hợp

n

Y

i=0,i6=k k−i

Giải.Với mỗik ∈ {0,1,2, , n}, ta có n

Y

i=0,i6=k k−i =

1

k(k−1) (k−2) 2.1.(−1) (−2) (−(n−k−1)).(−(n−k))

=

k!(−1)n−k(n−k)! = (−1) n−k

k! (n−k)! =

n!.(−1) n−k

n!

k! (n−k)! = n!.(−1)

n−k Cnk

Ta có biểu diễn sau

Đẳng thức 2.

n

Y

i=0,i6=k k−i =

1 n!.(−1)

n−k Cnk

Bài toán 3.Giả sửnlà số nguyên dương,n ≥2, xác định trước Với mỗik ∈ {1,2, , n}, hãy biểu diễn tích sau qua số tổ hợp

n

Y

i=1,i6=k

(n+ 1)−i k−i

Giải.Với mỗik ∈ {1,2, , n}, ta có n

Y

i=1,i6=k

(n+ 1)−i k−i =

n(n−1) (n−k+ 2).(n−k) 2.1

(k−1) (k−2) 2.1.(−1) (−2) (−(n−k−1)).(−(n−k))

=

n! n−k+1

(k−1)!(−1)n−k(n−k)! = (−1)

n−k n!

(k−1)! (n−k)! (n−k+ 1) = (−1)n−k n!

(k−1)! (n−k+ 1)! = (−1) n−k

Cnk−1

Ta có biểu diễn sau

Đẳng thức 3.

n

Y

i=1,i6=k

(n+ 1)−i

k−i = (−1) n−k

(50)

Bài toán 4.Giả sửnlà số nguyên dương xác định trước Với mỗik ∈ {0,1,2, , n}, biểu diễn tích sau qua số tổ hợp

n

Y

i=0,i6=k

(n+ 1)−i k−i

Giải.Với mỗik ∈ {0,1,2, , n}, ta có n

Y

i=0,i6=k

(n+ 1)−i k−i =

(n+ 1)n (n−k+ 2).(n−k) 2.1 k(k−1) 1.(−1) (−2) (−(n−k))

=

(n+1)!

n−k+1

k!(−1)n−k(n−k)! = (−1)

n−k (n+ 1)!

k! (n−k)! (n−k+ 1) = (−1)n−k (n+ 1)!

k! (n−k+ 1)! = (−1) n−k

Cn+1k

Ta có biểu diễn sau

Đẳng thức 4.

n

Y

i=0,i6=k

(n+ 1)−i

k−i = (−1) n−k

Ck n+1

1.2 Khai triển nhị thức Newton hệ quả

Ta biết đẳng thức khai triển nhị thức Newton sau

Đẳng thức 5.

(a+b)n = n

X

k=0

Cnk.an−k.bk

Vớia= 1, ta có hệ sau Đẳng thức 6.

n

X

k=0

Cnk.bk = (1 +b)n

Đẳng thức sở để giải số tốn tính tổng sau

Bài tốn 5.Tính

n

X

k=0

Cn+1k bk Giải.

n+1X k=0

Ck

n+1.bk= n

X

k=0 Ck

n+1.bk+Cn+1n+1.bn+1

⇒(1 +b)n+1 = n

X

k=0

(51)

n

X

k=0

Cn+1k bk= (1 +b) n+1

−bn+1

Ta có kết sau

Đẳng thức 7.

n

X

k=0

Cn+1k bk = (1 +b)n+1−bn+1

Bài tốn 6.Tính

n

X

k=0

Cn+2k bk Giải.

n+2X k=0

Cn+2k bk = n

X

k=0

Cn+2k bk+Cn+2n+1.bn+1+Cn+2n+2.bn+2

⇒(1 +b)n+2 = n

X

k=0

Cn+2k bk+ (n+ 2).bn+1+ 1.bn+2

n

X

k=0

Cn+2k bk= (1 +b)n+2−bn+2−(n+ 2)bn+1

Ta có kết sau

Đẳng thức 8.

n

X

k=0

Cn+2k bk = (1 +b)n+2−bn+2−(n+ 2)bn+1

Bài tốn 7.Tính

n

X

k=0

Cn+1k+1.bk Giải.Vớib6= 0, ta có

n

X

k=0

Cn+1k+1.bk= b

n

X

k=0

Cn+1k+1.bk+1 =

b

Cn+11 b1+Cn+12 b2+ +Cn+1n bn+Cn+1n+1.bn+1 =

b

Cn+10 b0+Cn+11 b1+Cn+12 b2+ +Cn+1n bn+Cn+1n+1.bn+1−Cn+10 b0

= b

C0

n+1.b0+Cn+11 b1+Cn+12 b2+ +Cn+1n bn

+Cn+1

n+1.bn+1−Cn+10 b0

= b n X k=0

Cn+1k bk+1.bn+1−1.1

!

= b

(52)

= b

(1 +b)n+1−1

Ta có kết sau

Đẳng thức 9.

n

X

k=0

Cn+1k+1.bk = b

(1 +b)n+1−1

Bài tốn 8.Tính

n+1X k=1

Cn+1k−1.bk Giải.

n+1X k=1

Cn+1k−1.bk=Cn+10 b+Cn+11 b2+ +Cn+1n−1.bn+Cn+1n bn+1 =bCn+10 b0+Cn+11 b1+ +Cn+1n−1.bn−1+Cn+1n bn

=bCn+10 b0+Cn+11 b1+ +Cn+1n−1.bn−1+Cn+1n bn+Cn+1n+1.bn+1−Cn+1n+1.bn+1

=b n+1X k=0

Cn+1k bk−1.bn+1

!

=b(1 +b)n+1−bn+1

Ta có kết sau

Đẳng thức 10.

n+1X k=1

Cn+1k−1.bk =b(1 +b)n+1−bn+1

Bài tốn 9.Tính

n+1X k=1

Cn+2k bk Giải.

n+1X k=1

Cn+2k bk =Cn+21 b1+Cn+22 b2+ +Cn+2n bn+Cn+2n+1.bn+1 =Cn+20 b0+Cn+21 b1+Cn+22 b2+ +Cn+2n bn+Cn+2n+1.bn+1+Cn+2n+2.bn+2

−Cn+2n+2.bn+2−Cn+20 b0 = (1 +b)n+2−bn+2−1

Ta có kết sau

Đẳng thức 11.

n+1X k=1

(53)

Bài tốn 10.Tính

n

X

k=1

Cn+2k bk Giải.Bởi Đẳng thức 11, ta có

(1 +b)n+2−bn+2−1 = n+1X k=1

Cn+2k bk

= n

X

k=1 Ck

n+2.bk+Cn+2n+1.bn+1 = n

X

k=1 Ck

n+2.bk+ (n+ 2).bn+1

n

X

k=1

Cn+2k bk = (1 +b)n+2−bn+2−(n+ 2).bn+1−1

Ta có kết sau

Đẳng thức 12.

n

X

k=1

Cn+2k bk= (1 +b)n+2−bn+2−(n+ 2).bn+1−1

Bài tốn 11.Tính

n

X

k=0

(−1)n−k.Cnk.kn+1

Giải.Sử dụng Công thức nội suy Lagrange cho đa thức

P(x) = xn+1−x(x−1) (x−2) (x−n)

có bậc khơng vượt qn, với mốc nội suyx0 = 0,x1 = 1,x2 = 2, ,xn =n, ta P (x) =

n

X

k=0

P (xk) n

Y

i=0,i6=k x−i k−i

  = n X k=0  P (k)

n

Y

i=0,i6=k x−i k−i

 = n X k=0  kn+1

n

Y

i=0,i6=k x−i k−i

 

Hệ số củaxnở vế trái là

n(n+ 1)

Hệ số củaxnở vế phải là n X k=0 kn+1 n Q

i=0,i6=k(k−i) =

n

X

k=0

kn+1

k(k−1) 2.1.(−1) (−2) (−(n−k))

= n

X

k=0

kn+1

k!.(−1)n−k.(n−k)! = n!

n

X

k=0

(−1)n−k n! k! (n−k)!.k

n+1

(54)

= n!

X

k=0

(−1)n−k.Cnk.kn+1

Đồng hệ số củaxnở hai vế, ta có kết sau Đẳng thức 13.

n

X

k=0

(−1)n−k.Cnk.kn+1= n.(n+ 1)!

2

Bài toán 12. Cho số nguyên dươngnvà số thựca Tính n

X

k=0

(−1)k.Cnk.(a−k)n

Giải.Áp dụng Công thức nội suy Lagrange cho đa thứcP(x) = xnvớin+ 1mốc nội suy xk =a−k, k = 0,1,2, , n,

ta có

xn=Xn k=0

P (xk) n

Y

i=0,i6=k

x−xi xk−xi

 = n X k=0  (xk)n

n

Y

i=0,i6=k

x−xi xk−xi

  = n X k=0

(a−k)n n

Y

i=0,i6=k

x−(a−i) a−k−(a−i) =

n

X

k=0

(a−k)n n

Y

i=0,i6=k

x−a+i

−(k−i)

 

Đồng hệ số củaxnở hai vế, ta được =

n

X

k=0

(a−k)n n

Y

i=0,i6=k

−(k−i)

  = n X k=0

(a−k)n

(−k) (−(k−1)) (−2) (−1).1.2 (n−k−1) (n−k)

!

= n

X

k=0

(a−k)n

(−1)kk! (n−k)!

!

= n

X

k=0

(−1)k

k! (n−k)!.(a−k) n ! = n X k=0

(−1)k.1 n!

n!

k! (n−k)!.(a−k) n

!

= n!

n

X

k=0

(−1)k.Ck

n.(a−k) n Suy n X k=0

(−1)k.Cnk.(a−k)n=n!

Vậy, ta có kết sau

Đẳng thức 14.

n

X

k=0

(55)

Chẳng hạn:

- Vớin= 2, ta có đồng thức sau

X

k=0

(−1)k.C2k.(a−k)2= 2!

hay

C20.a2−C21.(a−1)2+C22.(a−2)2 =

Vậy

a2−2(a−1)2+ (a−2)2 =

- Vớin= 3, ta có đồng thức sau

X

k=0

(−1)k.C3k.(a−k)3= 3!

hay

C30.a3−C31.(a−1)3+C32.(a−2)3−C33.(a−3)3 =

Vậy

a3−3(a−1)3

+ 3(a−2)3−(a−3)3 =

Bài toán 13. Giả sử n là số nguyên dương xác định trước vàm là số nguyên tùy ý thỏa mãn 1≤m≤n Đặt

Rmn := m

X

k=0

(m−k)n.(−1)k.Cn+1k Chứng minh rằng

Rnn−m+1 =Rmn Giải. Vì biểu diễnRm

n Rnn−m+1 phức tạp, nên để dễ hiểu nội dung toán, ta thử

minh họa Chẳng hạn vớim = 3,n = 4, ta có Rmn =R34 =

3

X

k=0

(3−k)4.(−1)k.C5k= 34.C50−24.C51+ 14.C52 = 81.1−16.5 + 1.10 = 11

Rnn−m+1 =R24 =

X

k=0

(2−k)4.(−1)k.C5k= 24.C50−14.C51 = 16.1−1.5 = 11

Vậy

Rnn−m+1 =Rmn

Bây giờ, ta trở lại toán

Rnn−m+1 = n−Xm+1

k=0

(56)

= n−Xm+1

k=0

((n+ 1−k)−m)n.(−1)k.Cn+1n+1−k

Đặti=n+ 1−k Vớik= 0, thìi=n+ Vớik =n−m+ 1thìi=m < n+ Khi

Rnn−m+1 =−

n+1X i=m

(i−m)n.(−1)n+1−i.Cn+1i

=− n+1X i=m

(−(m−i))n.(−1)n+1−i.Cn+1i =− n+1X i=m

(−1)n.(m−i)n.(−1)n+1−i.Cn+1i

=− n+1X i=m

(m−i)n.(−1)2n+1−i.Ci

n+1 =− n+1X i=m

(m−i)n.(−1)2n+1.(−1)−i.Ci n+1

= n+1X i=m

(m−i)n.(−1)−i.Cn+1i = n+1X i=m

(m−i)n.(−1)i.Cn+1i

Thay sốibởi sốk, ta có Rn−m+1

n =

n+1X k=m

(m−k)n.(−1)k.Ck n+1

Do

Rnn−m+1 =Rmn ⇔

n+1X k=m

(m−k)n.(−1)k.Cn+1k = m

X

k=0

(m−k)n.(−1)k.Cn+1k

m

X

k=0

(m−k)n.(−1)k.Cn+1k −

n+1X k=m

(m−k)n.(−1)k.Cn+1k =

m

X

k=0

(m−k)n.(−1)k.Cn+1k − n+1X k=m

(m−k)n.(−1)k.Cn+1k =

m

X

k=0

(m−k)n.(−1)k.Cn+1k + n+1X k=m

(m−k)n.(−1)k+1.Cn+1k =

n+1X k=0

(m−k)n.(−1)k+1.Cn+1k =

Ta cần chứng minh đẳng thức

Thật vậy, vớim tùy ý,1 ≤m ≤n, áp dụng Công thức nội suy Lagrange cho đa thứcP(x) = (m−x)n, với mốc nội suyxj =j,1≤j ≤n+ 1,ta có

(m−x)n= n+1X k=1

(m−k)n n+1Y j=1,j6=k

x−j k−j

(57)

Bởi đồng thức trên, chox= 0, ta có mn =

n+1X k=1

(m−k)n n+1Y j=1,j6=k

−j k−j

 

Với mỗi1≤j ≤n+ 1,j 6=k, ta có n+1Y

j=1,j6=k

(−j) = (−1) (−2) (−(k−1)).(−(k+ 1)) (−n) (−(n+ 1))

= (−1) n+1

.(n+ 1)!

−k = (−1) n

.(n+ 1)! k

và, Đẳng thức 1, ta có

n+1Y j=1,j6=k

1 k−j =

1

(n+ 1)!.(−1) n+1−k

.k.Cn+1k

Vậy

mn= n+1X k=1

(m−k)n.(−1)n.(n+ 1)! k

1

(n+ 1)!.(−1) n+1−k

.k.Ck n+1

!

hay

mn = n+1X k=1

(m−k)n(−1)n.(−1)n+1−k.Cn+1k

= n+1X k=1

(m−k)n.(−1)2n+1−k.Cn+1k = n+1X k=1

(m−k)n.(−1)k+1.Cn+1k

Suy

−mn+ n+1X k=1

(m−k)n.(−1)k+1.Cn+1k =

hay

n+1X k=0

(m−k)n.(−1)k+1.Cn+1k =

Ta có điều phải chứng minh Vậy, với cách xác địnhRm

n tốn trên, ta có kết sau Đẳng thức 15.

(58)

2 Đẳng thức tổ hợp “ẩn” toán Olympic Bài toán 14 (USA - 1975)Cho đa thứcP (x)bậcn ≥1, thỏa mãn

P (k) = Ck

n+1

,∀k = 1,2, , n

TínhP (n+ 1)

Giải. Áp dụng Cơng thức nội suy Lagrange cho đa thứcP (x), với bậcn, tạin+ 1điểm khác

nhau đôi mộtx0 = 0, x1 = 1, , xk =k, , xn=n, ta có P(x) =

n

X

k=0

 P (xk)

n

Y

i=0,i6=k

x−xi xk−xi

  = n X k=0  P (k)

n

Y

i=0,i6=k x−i k−i

 = n X k=0   Ck n+1 n Y

i=0,i6=k x−i k−i

 

Do đó, Đẳng thức 4, ta có

P (n+ 1) = n X k=0   Ck n+1 n Y

i=0,i6=k

(n+ 1)−i k−i

  = n X k=0 Ck n+1

.(−1)n−k.Cn+1k

!

= n

X

k=0

(−1)n−k

VậyP (n+ 1) = 1khinchẵn vàP (n+ 1) = 0khinlẻ Bài tốn 15.Cho đa thứcP (x)có bậcn, thỏa mãn

P (k) = k, với mỗik∈ {1,2, ,n, n+ 1} Tính giá trị củaP(n+ 2)

Giải. Sử dụng Công thức nội suy Lagrange với mốc nội suyx1 = 1, x2 = 2, ,xn = n, xn+1 =n+ 1, ta

P(x) = n+1X k=1

P(xk) n+1Y i=1,i6=k

x−i k−i

  = n+1X k=1  P(k)

n+1Y i=1,i6=k

x−i k−i

 = n+1X k=1  1 k n+1Y i=1,i6=k

x−i k−i

 

Do

P (n+ 2) = n+1X k=1  1 k n+1Y i=1,i6=k

n−i+ k−i

 = n+1X k=1  1 k n+1Y i=1,i6=k

(n+ 1) + 1−i k−i

(59)

ĐặtN =n+ 1, Đẳng thức Đẳng thức 11, ta có P (n+ 2) =

n+1X k=1  1 k n+1Y i=1,i6=k

(n+ 1) + 1−i k−i

 = N X k=1  1 k N Y

i=1,i6=k

(N + 1)−i k−i

  = N X k=1 1

k.(−1) N−k

.Ck−1 N = n+1X k=1 1

k.(−1) n+1−k

.Ck−1 n+1 = n+1X k=1 k.(−1)

n−k+1

(n+ 1)! (k−1)! (n−k+ 2)!

!

= n+1X k=1

(−1)n−k+1 (n+ 1)!

k(k−1)! (n−k+ 2)!

!

= n+1X k=1

(−1)n−k+1 (n+ 1)! k! (n−k+ 2)!

!

= n+1X k=1

(−1)n−k+1 n+

(n+ 2)! k! (n−k+ 2)!

!

= n+

n+1X k=1

(−1)n−k+1.Cn+2k

=

n+ 2.(−1) n+1

n+1X k=1

Cn+2k (−1)−k=

n+ 2.(−1) n+1

n+1X k=1

Cn+2k (−1)k

=

n+ 2.(−1) n+1

.(1−1)n+2−(−1)n+2−1=

n+ 2.(−1) n+1

.−(−1)n+2−1

= n+

−(−1)2n+3−(−1)n+1= n+

1−(−1)n+1= 1−(−1) n+1

n+

Vậy

P (n+ 2) = n+ 2,

khinchẵn vàP(n+ 2) = 0, khinlẻ

Bài tốn 16 (USAMO 1975)Cho đa thứcP (x)có bậcn, thỏa mãn P (k) = k

k+ 1, với mỗik∈ {0,1,2, ,n} Tính giá trị củaP (n+ 1)

Giải.Sử dụng Công thức nội suy Lagrange với mốc nội suyx1 = 0,x1 = 1, ,xn=n, ta

được

P (x) = n

X

k=0

P (xk) n

Y

i=0,i6=k x−i k−i

  = n X k=0  P (k)

n

Y

i=0,i6=k x−i k−i

 = n X k=0   k

k+ n+1Y i=1,i6=k

x−i k−i

 

Do đó, Đẳng thức 4, ta có

P (n+ 1) = n

X

k=0

  k

k+ n

Y

i=0,i6=k

n−i+ k−i

 = n X k=0   k

k+ n

Y

i=0,i6=k

(n+ 1)−i k−i

(60)

= n

X

k=0 k

k+ 1.(−1) n−k

.Cn+1k = n

X

k=0 k

k+ 1.(−1) n−k

(n+ 1)! k! (n−k+ 1)!

= n+

n

X

k=0

k.(−1)n−k (n+ 2)! (k+ 1)! (n−k+ 1)!

!

= n+

n

X

k=0

k.(−1)n−k.Cn+2k+1

= n+

n

X

k=0

(k+ 1−1).(−1)n−k.Cn+2k+1

= n+

n

X

k=0

(k+ 1) (−1)n−k.Cn+2k+1+ n

X

k=0

(−1)n−k+1.Cn+2k+1

!

Hơn nữa, Đẳng thức 7, ta có

n

X

k=0

(k+ 1).(−1)n−k.Cn+2k+1

= n

X

k=0

(k+ 1).(−1)n−k (n+ 2)! (k+ 1)! (n−k+ 1)

!

= (n+ 2) n

X

k=0

(−1)n−k (n+ 1)! k! (n−k+ 1)

!

= (n+ 2) n

X

k=0

(−1)n−k.Cn+1k = (−1)n.(n+ 2) n

X

k=0

(−1)−k.Cn+1k

= (−1)n.(n+ 2) n

X

k=0

(−1)k.Cn+1k = (−1)n.(n+ 2).(1−1)n+1−(−1)n+1

= (−1)n.(n+ 2).(−1)n+2 = (−1)2n+2.(n+ 2) =n+

Ngoài ra, đặtK =k+ 1, Đẳng thức 11, ta có n+1X

k=0

(−1)n−k+1.Ck+1 n+2 = n+1X K=1

(−1)n−K+2.CK n+2

= (−1)n+2 n+1X K=1

(−1)−K.Cn+2K = (−1)n+2 n+1X K=1

(−1)K.Cn+2K

= (−1)n+2.(1−1)n+2−(−1)n+2−1= (−1)n+2.−(−1)n+2−1=−1 + (−1)n+1

Do

P (n+ 1) = n+

n+ 2−1 + (−1)n+1= n+ + (−1) n+1

n+

Vậy

P (n+ 1) = n+ 1−1 n+ =

n n+ 2,

khinchẵn

P (n+ 1) = n+ + n+ = 1,

(61)

Bài tốn 17.Cho đa thứcP (x)có bậcn, thỏa mãn P (k) = 2k, với mỗik∈ {1,2, ,n, n+ 1} Tính giá trị củaP(n+ 2)

Giải. Sử dụng Công thức nội suy Lagrange với mốc nội suyx1 = 1, x2 = 2, ,xn = n, xn+1 =n+ 1, ta

P(x) = n+1X k=1

P(xk) n+1Y i=1,i6=k

x−i k−i

  = n+1X k=1  P (k)

n+1Y i=1,i6=k

x−i k−i

 =

n+1X k=1

 2k

n+1Y i=1,i6=k

x−i k−i

 

Do đó, Đẳng thức 10, ta có

P (n+ 2) = n+1X k=1

 2k

n+1Y i=1,i6=k

n−i+ k−i

 

= n+1X k=1

2k. (n+ 1)n (n−k+ 3).(n−k+ 1) 2.1

(k−1) (k−2) 1.(−1) (−(n−k)).(−(n−k+ 1))

!

= n+1X k=1

 2k

(n+1)!

n−k+2

(k−1) (k−2) 1.(−1) (−(n−k)).(−(n−k+ 1))

  = n+1X k=1  2k

(n+1)!

n−k+2

(k−1) (k−2) 1.(−1)n−k+1.1 (n−k).(n−k+ 1)

 

= n+1X k=1

(−1)n−k+1.2k (n+ 1)!

(k−1) (k−2) 1.1 (n−k) (n−k+ 1) (n−k+ 2)

!

= n+1X k=1

(−1)n−k+1.2k (n+ 1)! (k−1)! (n−k+ 2)!

!

= n+1X k=1

(−1)n−k+1.Cn+1k−1.2k

= (−1)n+1 n+1X k=1

(−1)−k.Cn+1k−1.2k= (−1)

n+1n+1X k=1

(−1)k.Cn+1k−1.2k = (−1)

n+1n+1X k=1

Cn+1k−1.(−2) k

= (−1)n+1.(−2).(1−2)n+1−(−2)n+1= (−1)n+1.(−2).(−1)n+1−(−2)n+1 = (−1)n+1.(−1)n+1.(−2)−(−1)n+1.(−2)n+1.(−2) = 1.(−2)−2n+1.(−2)

= 22n+1−1

Vậy

P (n+ 2) = 22n+1−1

(62)

Bài toán 18 (United Kingdom) Cho số nguyên dương k và số nguyên đôi phân biệt bất kìa1, a2, , an Chứng minh tổng

n X i=1 ak i n Q

j=1,j6=i(ai−aj) luôn nhận giá trị số nguyên.

Giải. Xét đa thức P(x) = xk và Q(x) = (x−a

1) (x−a2) (x−an) Chia đa thức P (x)cho đa thứcQ(x), ta thấy tồn đa thứcS(x)vàR(x)với hệ số nguyên (trong 0≤degR(x)<degQ(x) =n) cho

P(x) = Q(x)S(x) +R(x)

Từ đây, ta cóR(ai) =aki, vớii= 1,2, , n

Sử dụng Công thức nội suy Lagrange cho đa thứcR(x)với mốc nội suy a1, a2, , an, ta

được

R(x) = n

X

i=1

R(ai) n

Y

j=1,j6=i

x−aj ai−aj

 = n X i=1  aik

n

Y

j=1,j6=i

x−aj ai−aj

 

Hệ số xn−1 của đa thứcR(x)ở vế trái số nguyên Hệ số củaxn−1 của đa thứcR(x)

ở vế phải số

n

X

i=1

aik n

Q

j=1,j6=i(ai−aj)

Đồng hệ số củaxn−1ở vế, ta thu kết cần chứng minh.

Chẳng hạn, với số nguyên đôi phân biệt bất kìa1, a2, a3, số sau số nguyên a15

(a1−a2) (a1−a3)

+ a2

5

(a2−a1) (a2−a3)

+ a3

5

(a3−a1) (a3−a2)

Bài toán 19 (Việt Nam - 1977)Chon+ 1số nguyênx0 < x1 < < xn Chứng minh trong giá trị đa thức

P (x) = xn+a1xn−1+a2xn−2+ +an

tại điểmx0, x1, , xn, ln tìm số mà giá trị tuyệt đối khơng bé 2n!n

Giải.Áp dụng Công thức nội suy Lagrange cho đa thứcP (x), bậcn, tạin+ 1điểm khác

từng đơi mộtx0 < x1 < < xn, ta có P (x) =

n

X

j=0

P(xj) n

Y

i=0,i6=j

x−xi xj −xi

 

Hệ số củaxnở vế trái Hệ số củaxnở vế phải bằng n

X

j=0

P(xj) n

Y

i=0,i6=j xj −xi

(63)

Do

n

X

j=0

P (xj) n

Y

i=0,i6=j xj −xi

 =

Giả sử

|P (xj)|< n!

2n,∀j = 0,1, , n

Khi đó, ta có

1 = n X j=0 

P (xj) n

Y

i=0,i6=j xj−xi

  ≤ n X j=0 

|P (xj)| n

Y

i=0,i6=j

|xj −xi|

 

Lưu ý rằng, với số nguyênx0 < x1 < < xn, ta ln có

|xj−xi| ≥ |j−i|,∀i, j = 0,1, , n

Khi đó, Đẳng thức 2, ta có

1< n

X

j=0

 n!

2n n

Y

i=0,i6=j

|j−i|

 = n X j=0 n! 2n

1 n!.C

j n

!

= 2n

n X j=0 Cj n= 2n.2

n = 1,

mâu thuẫn Ta có điều phải chứng minh

Bài toán 20 (Iran - 2011)Cho số nguyênn≥2và đa thức f(x) =xn+a

n−2xn−2+an−3xn−3+ +a1x+a0 có hệ số số thực Chứng minh tồn tạik∈ {1,2, , n}sao cho

|f(k)| ≥ n! Ck

n

Giải.Giả sử với mọik∈ {1,2, , n}, ta có

|f(k)|< n! Ck n

Xét đa thức

g(x) = f(x)−xn=an−2xn−2+an−3xn−3 + +a1x+a0

Ta códegg ≤n−2 Sử dụng Cơng thức nội suy Lagrange với mốc nội suy làx1 = 1, x2 = 2, , xn =n, ta có

g(x) = n

X

k=1

g(xk) n

Y

i=1,i6=k

x−xi xk−xi

 = n X k=1  g(k)

n

Y

i=1,i6=k x−i k−i

(64)

Trong đẳng thức trên, hệ số củaxn−1 ở vế trái 0, hệ số củaxn−1ở vế phải bằng n

X

k=1

 g(k)

n

Y

i=1,i6=k k−i

  Vậy = n X k=1  g(k)

n

Y

i=1,i6=k k−i

 = n X k=1 

(f(k)−kn) n

Y

i=1,i6=k k−i

  = n X k=1 

(f(k)−kn) n

Y

i=1,i6=k k−i

 

Do đó, Đẳng thức 1, ta có

0 = n

X

k=1

(f(k)−kn).1 n!.(−1)

n−k k.Ck

n

= n!

n

X

k=1

f(k).(−1)n−k.k.Cnk

− n X k=1

kn.(−1)n−k.k.Cnk

! hay n X k=1

f(k).(−1)n−k.k.Cnk= n

X

k=1

(−1)n−k.Cnk.kn+1

Bởi Đẳng thức 13, suy

n

X

k=1

f(k).(−1)n−k.k.Cnk= n.(n+ 1)!

2

Do

n.(n+ 1)!

2 =

n.(n+ 1)! ≤ n X k=1

|f(k)|.k.Cnk

< n X k=1 n! Ck n

.k.Cnk

!

=n! n

X

k=1

k =n!n(n+ 1)

2 =

n.(n+ 1)!

2 ,

mâu thuẫn Ta có điều phải chứng minh

Bài toán 21.Giả sửnlà số nguyên dương cho trước vàP(x)là đa thức với bậc không lớn hơn 2n, thỏa mãn điều kiện

|P (k)| ≤1,∀k ∈ {−n, −(n−1), ,0, ,n−1, n} Chứng minh rằng

|P (x)| ≤4n

Giải. Áp dụng Công thức nội suy Lagrange cho đa thức P (x), với bậc không lớn hơn2n, 2n+ 1điểm nguyên khác đôi

(65)

ta có

P (x) = n

X

k=−n

P (xk) n

Y

i=−n,i6=k

x−xi xk−xi

  =

n

X

k=−n

 P(k)

n

Y

i=−n,i6=k x−i k−i

 

Do

|P (x)| ≤ n

X

k=−n

|P (k)| n

Y

i=−n,i6=k

|x−i|

|k−i|

 

Với mỗik ∈ {−n, −(n−1), ,0, ,n−1, n}, ta có

n

Y

i=−n,i6=k

|x−i|=|x−(−n)|.|x−(−(n−1))| |x−n|

≤ |n−(−n)|.|n−(−(n−1))| |n−(n−1)|

= 2n.(2n−1) = (2n)!, ∀x∈[−n;n]

Ngoài ra, với mỗik∈ {−n, −(n−1), ,0, , n−1, n}, ta có n

Y

i=−n,i6=k

(k−i) = (k+n) (k+n−1) 1.(−1) (−2) (−(n−k))

= (−1)n−k.(n+k) ! (n−k) !

Vậy, với mọix∈[−n, n], ta có

|P (x)| ≤

n

X

k=−n

(2n)!

(n+k)! (n−k)! = 2n

X

l=0

(2n)! l! (2n−l)! =

2n

X

l=0

C2nl = 22n = 4n

Bài toán 22.Giả sửnlà số nguyên dương cho trước Xác định tất đa thứcP (x)có bậc nhỏ hơnnvà thỏa mãn điều kiện

n

X

k=0

(−1)n−k+1.P(k).Cnk =

Giải. Áp dụng Công thức nội suy Lagrange cho đa thức P(x), với bậc nhỏ hơnn, tạin điểm

nguyên khác đôi mộtx0 = 0, x1 = 1, , xk =k, , xn−1 =n−1, ta có

P (x) = nX−1 k=0

P (xk) nY−1 i=0,i6=k

x−xi xk−xi

  =

nX−1 k=0

 P (k)

nY−1 i=0,i6=k

x−i k−i

 

Suy

P(n) = nX−1 k=0

 P (k)

nY−1 i=0,i6=k

n−i k−i

(66)

Hơn nữa, với mỗik ∈ {0,1, , n−1}, Đẳng thức 4, ta có nY−1

i=0,i6=k n−i

k−i = (−1) n−k−1

Cnk = (−1) n−k+1

Cnk

Do

P (n) = nX−1 k=0

P (k).(−1)n−k+1Cnk

hay

nX−1 k=0

P (k).(−1)n−k+1Cnk−P (n) =

Đẳng thức viết lại sau

nX−1 k=0

(−1)n−k+1P (k)Cnk+ (−1)n−n+1P (n)Cnn=

hay

n

X

k=0

(−1)n−k+1P(k)Ck n

=

Do đó, điều kiện tốn thỏa mãn Tóm lại, đa thức cần tìm có dạng

P (x) = nX−1 k=0

 P (k)

nY−1 i=0,i6=k

x−i k−i

 

trong đóP (0), P (1), , P(n−1)là giá trị tùy ý

Bài toán 23 (IMO Shortlist - 1983)Giả sử(Fn)n≥1là dãy Fibonacci F1 =F2 = 1, Fn+2 =Fn+1+Fn(n ≥1) P (x)là đa thức có bậc 990, thỏa mãn

P (k) = Fk, k∈ {992,993, ,1982} Chứng minh rằngP (1983) =F1983−1

Giải.Chú ý rằngFncó cơng thức tổng qt Fn=

1

5(α n

(67)

với mọinnguyên dương,

α= +

5 , β=

1−√5

Dễ thấy rằngα+β = 1, α−β =√5, αβ =−1

Áp dụng Công thức nội suy Lagrange với 991 mốc nội suy

xk =k, k∈ {992,993, ,1982},

ta có

P (x) = 1982X k=992

P (xk) 1982Y i=992,i6=k

x−i k−i

  = 1982X k=992  P(k)

1982Y i=992,i6=k

x−i k−i

 =

1982X k=992

 Fk

1982Y i=992,i6=k

x−i k−i

  = 1982X k=992  √1

5

αk−βk 1982Y i=992,i6=k

x−i k−i

 = √1

5 1982X k=992

αk−βk 1982Y i=992,i6=k

x−i k−i

 

Do đó, Đẳng thức Đẳng thức 7, ta có

P (1983) = √1

5 1982X k=992

αk−βk 1982Y i=992,i6=k

1983−i k−i

 

= √1

5 990

X

k=0

αk+992−βk+992 990

Y

i=0,i6=k

991−i k−i

 

= √1

5 990

X

k=0

αk+992−βk+992.(−1)990−k.C991k

= √1

5 990

X

k=0

αk+992−βk+992.(−1)990

.(−1)−k.Ck 991

= √1

5 990

X

k=0

αk+992−βk+992.(−1)−k.C991k

= √1

5 990

X

k=0

αk+992−βk+992.(−1)k.C991k

= √1

5 990

X

k=0

αk+992.(−1)k.C991k − 990

X

k=0

βk+992.(−1)k.C991k

!

= √1

5 α

992.X990 k=0

αk.(−1)k.C991k −β992 990

X

k=0

βk.(−1)k.C991k

!

= √1

5 α 992.X990

k=0

(−α)k.Ck 991

−β992.X990 k=0

(−β)k.Ck 991

(68)

= √1

5

α992.(1−α)991−(−α)991−β992.(1−β)991−(−β)991

=√1

5

α992.(1−α)991+α991−β992.(1−β)991+β991

=√1

5

α992.β991+α991−β992.α991+β991

= √1

5

α991−β991 α992−β992= √1

5

α1983 −α991β992+α992β991−β1983

= √1

5

α1983−β1983−(αβ)991.(β−α)= √1

5

α1983−β1983−(−1)991.−√5

= √1

5

α1983−β1983−√5= √1

5

5.F1983−

5=F1983−1

Nhận xét.Bài tốn tổng qt hóa sau:

Bài tốn tổng qt.Giả sử(Fn)n≥1 là dãy Fibonacci

F1 =F2 = 1, Fn+2 =Fn+1+Fn(n ≥1) P (x)là đa thức có bậcn0(n0 ≥1)thỏa mãn

P (n0+k) =Fn0+k, k∈ {2; 3; ;n0+ 2} Chứng minh rằngP (2n0+ 3) =F2n0+3−1

Giải.Áp dụng Công thức nội suy Lagrange vớin0+ 1mốc nội suy xn0+k=n0 +k, k∈ {2,3, ,n0+ 2},

ta có

P (x) = 2nX0+2 k=n0+2

P (xk)

2nY0+2 i=n0+2,i6=k

x−i k−i

 

= 2nX0+2 k=n0+2

 P (k)

2nY0+2 i=n0+2,i6=k

x−i k−i

 =

2nX0+2 k=n0+2

 Fk

2nY0+2 i=n0+2,i6=k

x−i k−i

 

= 2nX0+2 k=n0+2

 √1

5

αk−βk

2nY0+2 i=n0+2,i6=k

x−i k−i

 = √1

5 2nX0+2 k=n0+2

αk−βk

2nY0+2 i=n0+2,i6=k

x−i k−i

 

Do đó, Đẳng thức Đẳng thức 7, ta có

P (2n0+ 3) =

5 2nX0+2 k=n0+2

αk−βk

2nY0+2 i=n0+2,i6=k

(2n0 + 3)−i k−i

 

= √1

5 n0

X

k=0

αk+n0+2−βk+n0+2. n0

Y

i=0,i6=k

(n0+ 1)−i k−i

(69)

1 √ X k=0

αk+n0+2−βk+n0+2.(−1)n0−k.Ck n0+1

= √1

5 n0

X

k=0

αk+n0+2−βk+n0+2.(−1)n0.(−1)−k.Ck n0+1

= (−1) n0 √ n0 X k=0

αk+n0+2−βk+n0+2.(−1)−k.Ck n0+1

= (−1) n0 √ n0 X k=0

αk+n0+2−βk+n0+2.(−1)k.Ck n0+1

= (−1) n0 √ n0 X k=0

αk+n0+2.(−1)k.Ck n0+1

− n0 X k=0

βk+n0+2.(−1)k.Ck n0+1

!

= (−1) n0

5 α n0+2

n0

X

k=0

αk.(−1)k.Cnk0+1−βn0+2 n0

X

k=0

βk.(−1)k.Cnk0+1

!

= (−1) n0

5 α n0+2

n0

X

k=0

(−α)k.Cnk0+1−βn0+2 n0

X

k=0

(−β)k.Cnk0+1

!

= (−1) n0

5

αn0+2.(1−α)n0+1

−(−α)n0+1

−βn0+2.(1−β)n0+1

−(−β)n0+1

= (−1) n0

5

αn0+2.βn0+1−(−α)n0+1

−βn0+2.αn0+1−(−β)n0+1

- Nếun0 số chẵn, P (2n0+ 3) =

1

5

αn0+2.βn0+1+αn0+1−βn0+2.αn0+1+βn0+1

= √1

5

αn0+1+βn0+1 αn0+2−βn0+2

= √1

5

α2n0+3−αn0+1βn0+2+αn0+2βn0+1−β2n0+3

= √1

5

α2n0+3−αn0+1βn0+2+αn0+2βn0+1−β2n0+3

= √1

5

α2n0+3−β2n0+3−(αβ)n0+1.(β

−α)= √1

5

α2n0+3−β2n0+3−(−1)n0+1.

−√5

= √1

5

α2n0+3−β2n0+3−√5= √1

5.F2n0+3−

5=F2n0+3−1

- Nếun0 số lẻ, P (2n0+ 3) =−

1

5

αn0+2.βn0+1−αn0+1−βn0+2.αn0+1−βn0+1

=−√1

5

(70)

= √1

5

αn0+2.αn0+1−βn0+1+βn0+2.αn0+1−βn0+1

= √1

5

αn0+1−βn0+1 αn0+2+βn0+2

= √1

5

α2n0+3+αn0+1βn0+2−αn0+2βn0+1−β2n0+3

= √1

5

α2n0+3+αn0+1βn0+2−αn0+2βn0+1−β2n0+3

= √1

5

α2n0+3−β2n0+3+ (αβ)n0+1.(β

−α)= √1

5

α2n0+3−β2n0+3+ (−1)n0+1.

−√5

= √1

5

α2n0+3−β2n0+3−√5= √1

5.F2n0+3−

5=F2n0+3−1

Vậy, hai trường hợp, ta ln cóP(2n0+ 3) =F2n0+3−1

Tài liệu tham khảo

[1] Võ Quốc Bá Cẩn,Một số toán ứng dụng cơng thức nội suy Lagrange, Kỷ yếu “Gặp gỡ Tốn học” lần thứ 10, TP Hồ Chí Minh, 2018

(71)

ĐỊNH HƯỚNG GIẢI MỘT SỐ BÀI TOÁN CỰC TRỊ TỔ

HỢP DÀNH CHO THCS

Nguyễn Tất Thu - Đồng Nai

GIỚI THIỆU

Trong viết này, chúng tơi trình bày số phân tích để tìm hướng giải tốn cực trị tổ hợp dành cho THCS

0.1 Lý thuyết

Bài toán cực trị thường xuyên xuất toán tổ hợp Để giải toán cực trị ta cần vận dụng nhiều nội dung kiến thức liên quan đại số, tổ hợp Thông thường ta gặp tốn sau:

Bài tốn 1.Tìm số ngun dươngknhỏ (lớn nhất) cho tậpAmà|A|=k có

tính chấtT

Hướng giải:Xét tậpAcó tính chất đặc biệt mà|A|=mvàAkhơng thỏa tính chất T, suy rakmin ≥ m+ Tiếp theo ta chứng minh tậpAmà|A| =m+ 1đều có tính chất T Vậykmin =m+

Để chứng minh tập Amà|A| = m+ 1đều có tính chất T ta sử dụng nguyên lí

Dirichlet kĩ thuật xây dựng tập hợp (xây dựng phần tử xuất phát)

Bài tốn 2.Tìm số phần tử lớn (nhỏ nhất) tậpAgồm phần tử có tính chấtT Hướng giải: Đặt |A| = k, ta chứng minhk ≤ m (k ≥ m), sau xây dựng tậpA0 thỏa

tính chấtT và|A0|=m.

0.2 Các ví dụ

Ví dụ Xét20số nguyên dương đầu tiên1, 2, 3, , 20 Hãy tìm số nguyên dươngknhỏ nhất có tính chất: Với cách lấy ra ksố phân biệt từ20số trên, lấy hai số phân biệta bsao choa+blà số nguyên tố.

(72)

1 Chỉ lấy số phần tử lớn khơng thỏa tốn

Để chọn phần tử khơng thỏa tốn, có nghĩa ta cần chọn phần tử, cho phần tử lấy với hai số bất kìa, b thìa+b hợp số Điều ta nghĩ

đến ta chọn số chẵn (vì tổng hai số lấy ln số chẵn lớn hơn2, nên

là hợp số) Ta chọn được10số chẵn, đók ≥11

2 Ta chứng minh với cách lấy 11 số ta chọn hai số có tổng số

nguyên tố Để chứng minh tồn ta dùng nguyên lí Dirichlet Để thực điều này, ta chia số1, 2, , 20thành10cặp cho tổng cặp số nguyên tố

Ta có lời giải sau

Chứng minh. Xét tập hợpX ={2; 4; 6; 8; 10; 12; 14; 16; 18; 20}ta thấy tổng hai phần tử tập hợp số nguyên tố Do k ≥ 11, ta chứng minh k = 11là số nhỏ thỏa mãn yêu cầu toán

Thật vậy, ta chia tập hợp

A={1, 2, 3, , 20}

thành10cặp số sau:

(1,2) ; (3,16) ; (4,19) ; (5,6) ; (7,10) ; (8,9) ; (11,20) ; (12,17) ; (13,18) ; (14,15)

Tổng hai số cặp số số nguyên tố Khi tập củaAcó11phần tử

thì tồn hai phần tử thuộc vào trong10cặp số Suy trongAln có

hai phần tử phân biệt có tổng số nguyên tố

Ví dụ GọiAlà tập tất số tự nhiên lẻ không chia hết cho 5và nhỏ hơn30 Tìm sốk nhỏ cho tập củaAgồmkphần tử tồn hai số chia hết cho nhau? Chứng minh. Ta có A = {1; 3; 7; 9; 11; 13; 17; 19; 21; 23; 27; 29}nên số phần tử |A| = 12 Xét tập

A0 ={9; 11; 13; 17; 19; 21; 23; 29}

Dễ thấy hai phần tử thuộcA0 khơng chia hết cho Từ ta suy đượck ≥9

Ta chứng minhk = 9thỏa đề

XétS tập củaAvà|S|= Xét ba cặp{21; 7},{27; 9},{1; 11}, ta thấy cặp bội Nếu trong3cặp có cặp thuộcSthì tốn giải

Giả sử ba cặp cặp thuộc S, do|S| = 9nênS phải chứa số

trong cặp chứa6số lại Từ suy trongSphải có cặp{3; 9}hoặc{3; 27}và cặp bội Hay nói cách khác trongS tồn hai số chia hết cho

Vậykmin =

Lưu ý.Mấu chốt toán phát tậpA0để từ ta khẳng định k ≥9và dự đốnkmin = Để tìm tậpA0, ta liệt kê hết số trongAmà khơng có hai số

là bội Với tốn này, việc tìm tậpA0khá đơn giản

(73)

Phân tích Ta biết với điều kiện0 < x < y < zthìx, y, zlà độ dài ba cạnh tam giác

và khix+y > z Vìz ∈X nênz ≤2018 Do ta cần chọnx, ysao chox+y >2018

thì hiển nhiên ta có x+y > z Do ta xét tập A0 = {1009, 1010, , 2018} với x, y, z ∈ A0 ta ln có x+y > z.Ta dự đốn tậpA có nhiều nhất1010 phần tử Để chứng

minh khẳng định ta có hai hướng để

Hướng 1:Ta chứng minh tậpAthỏa u cầu tốn thì|A| ≤1010

Hướng 2:Với tập conB củaX mà|B| ≥ 1011thì trongB tồn ba sốx, y, zkhông

độ dài ba cạnh tam giác

Chứng minh. Xét tập hợp A0 = {1009; 1010; .; 2018}, ta có|a0| = 1010và vớix, y, z ∈ A0 màx < y < zthì ta có

x+y≥1009 + 1010 = 2019>2018≥z,

hayx,y,z độ dài ba cạnh tam giác

Cách 1: Giả sử A = {a1; a2; .; ak} với a1 < a2 < · · · < ak Nếu ak < 2018, đặt t = 2018−akvàbi =ai+tvớii= 1, 2, , kvà tập{b1; b2; .; bk}cũng thỏa mãn toán bk= 2018 Do ta giả sửak = 2018

Ta cóa1+a2 > akvàa1 < a2 nên

2a2 > ak= 2018⇒a2 >1009⇒a2 ≥1010

Từ ta cóA\ {a1} ⊂ {1010; 1011; ; 2018},

|A| −1≤2018−1010 + = 1009⇒ |A| ≤1010

Vậymax|a|= 1010

Cách 2: Tương tự ta xét tậpA ={a1, a2, , ak = 2018}vớik ≥1011và ba phần

tử phân biệt thuộcAluôn độ dài ba cạnh tam giác

Xét cặp(i; 2018−i)vớii= 1,1008 Vì2018thuộcAnên cặp có nhiều

một số thuộcA, đó|A| ≤1008 + + = 1010(vơ lí)

Vậy tậpAcó nhiều nhất1010phần tử

Ví dụ Từ625số tự nhiên1, 2, , 624, 625ta chọn raksố cho hai số có tổng bằng625.Tìmknhỏ cho trongksố chọn có số phương.

Phân tích Để tìmmink, trước hết ta cách chọnksố (nhiều có thể) khơng

có hai số có tổng bằng625và khơng có số số phương

Vì cần chọn raksố mà hai số có tổng khác625nên cặp(i; 625−i), i= 1,312ta

chọn nhiều số (do ta chọn nhiều nhất312 số), đồng thời trongk chọn

khơng có số số phương, nên ta phải loại sối ∈ {12; 22; 32; .; 172} Hơn

nữa, cói∈152, 72 thìivà625−iđều số phương, ta chọn tiếp số 625−i2 vớii ∈ {1; 2; .; 17}vài 6= 7, 15 Khi ta chọn được310 số Chính vậy, ta dự

(74)

Chứng minh. Xét cách chọn ra310số gồm2,3,5, ,312(gồm số nguyên dương từ1đến 312trừ số phương) số có dạng625−i2 vớiithuộc số nguyên dương từ1

đến17và bỏ hai số7và15 Ta có tất cả310số rõ ràng trong310số khơng có số

là số phương hai số có tổng ln khác625 Suy rak ≥311

Ta xét cách chọn ra311số cho khơng có hai số có tổng bằng625

Ta chia625số tự nhiên thành311nhóm sau:310nhóm đầu, nhóm gồm hai số có tổng

bằng625tức có dạng{a, 625−a}vớia 6= 49, a6= 225 Nhóm311gồm5số phương

{49,225,400,576,625} Vì311số chọn khơng có hai số có tổng bằng625nên 310nhóm đầu ta chọn nhóm nhiều nhất1số Do đó, ln có số thuộc nhóm311

được chọn Số chọn số phương Vậy tốn chứng minh

Ví dụ Cho số nguyên1≤a1 < a2 <· · ·< an= 100thỏa mãn: Với mọii≥2, tồn tại1≤p≤q ≤r ≤i−1sao cho

ai =ap+aq+ar Tìm giá trị nhỏ giá trị lớn củan?

Phân tích Dễ thấy vớin = 1,n = 2thì tốn khơng thỏa mãn

Xétn = 3ta cóa2 =a1+a1 +a1 = 3a1 vàa3ta chọn khả a3 = 3a2, a3 =a2+ 2a1, a3 = 2a2+a1

Do ta chọna1 = 20, a2 = 60vàa3 = 100

Để tìmmaxnta xét thử giá trị củaa1 Đầu tiên, dĩ nhiên ta xéta1 = 1, đóa2 = 3a1 =

Tuy nhiêna3 có nhiều khả năng, nhưnga3 ln số lẻ, từ suy ln số lẻ, điều

trái vớian = 100 Tương tự ta thấy đượca1 số lẻ, đóa1phải số chẵn Tiếp

theo ta xéta1 = 2, ta cóa3 = 6khi đóai ≡2 (mod 4)hayan= 100 ≡2 (mod 4)(vơ lí),

đóa1là bội của4 Tiếp theo ta xéta1 = 4,a2 = 12, đóa3 nhận giá trị36,20,30

Do ta cần tìm giá trị lớn nnên ta chọna3 = 20, từ ta có dự đốnai+1 −ai =

Khi

100 =an = (an−an−1) +· · ·+ (a2−a1) +a1 = 8(n−1) + 4⇒n = 13

Do ta dự đốnmaxn = 13

Chứng minh. Ta thấy n 6= 1, 2nên n ≥ Xét ba số a1 = 20, a2 = 60, a3 = 100, ta có a2 =a1 +a1+a1 vàa3 =a2+a1+a1 Do đóminn =

Ta thấy a1 ≡ (mod 2) a2 = 3a1 ≡ (mod 2) Bằng quy nạp ta có ≡ (mod 2)∀i = 1,2, , n Dẫn đếnan = 100 ≡1 (mod 2)(vơ lí) Do đóa1 ≡0 (mod 2) ⇒ a1 ≡0, (mod 4)

Nếu a1 ≡ (mod 4) chứng minh tương tự ta cóan = 100 ≡ (mod 4) (vơ lí) Suy a1 ≡0 (mod 4)haya1 ≡0, (mod 8)

(75)

Suy

ai+1−ai 8⇒ai+1−ai ≥8⇒ai+1 ≥ai+

Do

100 =an≥a1+ 8(n−1)≥4 + 8(n−1) = 8n−4⇒n ≤13

Xét sốai = 8i−4∀i= 1,2, ,13ta có

ai+1 =ai+ =ai+a1+a1

Vậymaxn = 13

Ví dụ (HSG Hà Nội năm 2017) Trên bàn có 100tấm thẻ đánh số từ 1đến100 Hai người A B lần lượt người lấy thẻ bàn cho ngườiA lấy thẻ đánh sốnthì đảm bảo ngườiB chọn thẻ đánh số2n+ Hỏi ngườiAcó thể lấy được nhiều thẻ bàn thỏa mãn yêu cầu trên?

Phân tích VìB viết số2n+ 2, nên2n+ 2khơng vượt q100, hay 2n+ ≤100⇔n≤49,

nghĩa làAchỉ viết số không vượt 49 Tuy nhiênAkhông thể viết tất số từ 1đến49, chẳng hạnAviết số1thì khơng thể viết số4 = 2·1 + Do đó, ta chia số từ1

đến49, cặp có dạngn và2n+ Khi cặpAchỉ chọn nhiều nhất1số

Ta xét cặp(1; 4),(2; 6),(3; 8),(5; 12),(7; 16),(9; 20),(10; 22),(11; 24),(13; 28), (14; 30), (15; 32), (17; 36), (18; 38), (19; 40), (21; 44), (23; 46)(có tất 16 cặp) Còn lại số chưa

được chọn là25,26,27,29,31,33,34,35,37,39,41,42,43,45,47,48,49(có tất cả17số) Do

đó,Achọn nhiều nhất16 + 17 = 33số

Chứng minh. VìB bốc thẻ2n+ 2nên2n+ ≤100 Suy ran ≤49 Do đó,Achỉ bốc

các thẻ đánh từ1đến49

Bây giờ, ta chia tập{1; 2; · · · ; 49}thành33tập sau:

{1; 4}, {3; 8}, {5; 12};· · · , {23; 49}

{2; 6}, {10; 22},{14; 30}, {18; 38} (16nhóm)

{25}, {27},{29},· · · ,{49}

{26}, {34}, {42}, {46} (17nhóm)

Ở nhóm,Ađược chọn tối đa số NếuAchọn nhiều hơn34số số từ1đến49

thì theo nguyên lý Dirichlet, tồn hai số thuộc nhóm (vơ lý) Do đó,A chọn

khơng q33số

Mặt khác,Acó thể chọn33số gồm25số lẻ khơng vượt quá49và7số chẵn2; 10; 14; 18; 34; 42; 48

thì thỏa mãn yêu cầu đề

Vậy ngườiAcó thể lấy nhiều nhất33tấm thẻ bàn thỏa mãn yêu cầu

Ví dụ (HSG Hà Tĩnh 2019) Cho bảng vng (2n+ 1)×(2n+ 1) (n ≥ 3), ta điền vào

(76)

Phân tích Trước hết ta thử vớin = 3, ta có bảng7×7như hình

1 1

1 1

1 1

1 1

0 0

0 0

0 0

0 0

0 0

0 0

0 0

1 1

1 1

1 1

Ta thấy số1chỉ nằm ô (i;j)với i, jcùng lẻ chẵn Số số 1nằm ôi, j

cùng lẻ là16và ôi, j chẵn là9 Hơn hình chữLphủ nhiều nhất1số1ở i, j chẵn Do đó, ta thấy cần nhất16hình chữLđể phủ bảng Ta cách phủ

hình vng bởi16hình chữLnhư sau

1 1

1 1

1 1

1 1

0 0

0 0

0 0

0 0

0 0

0 0

0 0

1 1

1 1

1 1

Phần lại ta lấy đối xứng Vậyn = 3thì ta có dáp số là16

Xét trường hợp tổng quát, tương tự ta thấy có(n+ 1)2 số1nằm ơ(i;j)vớii, j

lẻ Do đó, số hình chữLcần dùng là(n+ 1)2 Phần lại cách phủ bởi(n+ 1)2

số hình chữL

Chứng minh. Ta chứng minh quy nạp số hình chữLít cần dùng là(n+ 1)2.

Thật vậy, với bảng (2n+ 1)×(2n+ 1)ơ vng ta xét vng vị trí(i, j)vớii, j lẻ Rõ

(77)

Mặt khác số vng vị trí(i, j)vớii, j lẻ là(n+ 1)2 Vì số hình chữLcần dùng tối

thiểu là(n+ 1)2.

Ta chứng minh dùng(n+ 1)2để phủ theo yêu cầu toán Khin= 3ta dễ

thấy phủ bảng vng7×7bằng16hình chữL

Cách vẽ bảng bên (thể hiện8hình chữLở phần trên, phần cịn làm tương tự

bằng cách lấy đối xứng qua ô vị trí(4; 4))

1 1

1 1

1 1

1 1

0 0

0 0

0 0

0 0

0 0

0 0

0 0

1 1

1 1

1 1

Giả sử với bảng(2n+ 1)(2n+ 1)có thể phủ bởi(n+ 1)2hình chữLthỏa mãn Xét bảng (2n+ 3)(2n+ 3), chia bảng thành4bảng

(2n+ 1)×(2n+ 1); 2×2; 2×(2n+ 1); (2n+ 1)×2

Ta thấy bảng(2n+ 1)×2và2×(2n+ 1)có thể phủ bằngn+ 1hình chữL, cụ thể

như sau:

Đối với hình2×(2n+ 1)chia làm2phần gồm1hình2×3và(n−1)hình2×2 Hình2×3

phủ bởi2hình chữL; hình2×2phủ bởi1hình chữL, cón+ 1hình chữLphủ

hết chữ số1của hình2×(2n+ 1)

Tương tự cho hình(2n+ 1)×2

Bảng2×2có thể phủ hình chữL Do theo giả thiết quy nạp số hình chữ Lcó thể dùng để phủ

(n+ 1)2 + 2(n+ 1) + = (n+ 2)2

Do ta có điều phải chứng minh Kết luậnk = (n+ 1)2.

0.3 Bài tập

Bài tập Cho tập hợpAgồm 100 số nguyên dương tử số đến số 100 Tìm số nguyên dương knhỏ cho cách chọn rakphần tử tập hợpAthì ln có ba số độ dài ba

(78)

Bài tập Cho tậpAgồm16số nguyên dương Hãy tìm số ngun dươngknhỏ

có tính chất: Trong tập có k phần tử A tồn hai số phân biệta, b cho a2+b2là số nguyên tố.

Bài tập Cho tập hợp vàM tập hợp tậpX có tính chấtT nếu: tích phần tử

phân biệt trongM khơng số phương Tìm số phần tử lớn củaM

Bài tập Cho tập hợpX ={1,2,3, ,16}vàM tập hợp tậpXcó tính chấtT nếu: M khơng chứa ba phần tử đơi ngun tố Tìm số phần tử lớn củaM

Bài tập Cho tập hợpM = {1,2, , n}, n ≥ Hãy tìm sốm nhỏ cho

tập chứamphần tử M tồn hai sốa, bmà số bội số

Bài tập (Ninh Bình) Tìm số nguyên dươngn nhỏ cho nếua1, a2, , an số

phân biệt tùy ý chọn từ tậpX ={1; 2; .; 17}ta ln tìm số ngun dươngk

cho phương trìnhai−aj =kcó nhấtknghiệm

Bài tập Cho tập hợpX ={2, 3, 4, , 2014} Tìm số nguyên dương nhỏ nhấtnsao cho với

mọi cách chọnnsố nguyên tố từ tậpX ta ln chọn số nguyên

tố

Bài tập (SP Hà Nội 2014-2015) Một ba số nguyên dương(x; y; z)được gọi

baPythagorenếux2+y2 =z2 Tìm sốknhỏ cho tập gồmkphần tử tập S ={1,2,3, ,25}ln có ba phần tử tạo thành baPythagore

Bài tập (Brasil 2015) Cho tậpS = {1,2, ,6n}, n ≥2 Tìm số nguyên dươngk lớn

sao cho khẳng định sau đúng: tập conA củaS , |A|= 4n, có nhấtk cặp(a, b), a < b

và a| b

Bài tập 10 ChoX tập tập{1,2,3, ,10000}, cho nếua, bnằm X

thìabkhơng nằm trongX Tìm số phần tử lớn tậpX

Bài tập 11 ChoAlà tập tập hợp{1; 2; 3; ; 100},Acó phần tử nhỏ là1và

phần tử lớn là100 Giả sửAcó tính chất: Với phần tửxcủaA,x6= 1thìxhoặc

tổng hai phần tử thuộc Ahoặc hai lần phần tử thuộcA Tìm số phần tử nhỏ

có thể tập hợpA

Bài tập 12 ChoAlà tập tậpX ={1; 2; 3; .; 2018}sao cho với hai sốai, aj ∈A

màai 6= aj tồn tam giác cân có độ dài cạnh làai, aj Hỏi tậpAcó nhiều

nhất phần tử ?

Bài tập 13 ChoA tập tập số nguyên dương thỏa mãn: với mọix, y ∈ A mà x6=ythì ta có|x−y| ≥ xy

25.Tìm số phần tử lớn tậpA?

Bài tập 14 (Hà Tĩnh Ngày 1-2015) Cho tậpSlà tập hợp tất số, gồm2014×

2014 số thực thuộc đoạn[−1; 1]và có tổng Xét bảng vng kích thước 2014×2014 Tìm số dươngknhỏ cho điền số thuộc tậpSvào bảng,

mỗi số tồn hàng cột có giá trị tuyệt đối tổng số hàng cột khơng vượt quák

Bài tập 15 Giả sử S tập tập A = {1; 2; 3; .; 14; 15} thỏa mãn: tích3phần tử

(79)

Bài tập 16 Trong mặt phẳng cho100điểm tô hai màu đỏ trắng cho, điểm

tơ màu đỏ tâm đường trịn qua ba điểm màu trắng Hỏi số điểm trắng bao nhiêu?

Bài tập 17 Một tậpAgồm số nguyên dương gọi “đều”, sau bỏ phần

tử ta chia phần tử cịn lại thành hai tập có tổng phần tử Tìm số phần tử nhỏ tập “đều”

Bài tập 18 ChoX ={1; 2; .; 2013}vàA⊂ X, |A| =n Tìmnnhỏ cho với

cách chọn tập A ta ln tìm đượca, b∈A: a b =

Bài tập 19 (China 2001) Tìm số nguyên dươngmnhỏ cho với tập conAcủa tập X ={1, 2, 3, , 2001}đều có hai phần từa, b(khơng thiết phân biệt) thỏaa+b = 2n

vớinlà số nguyên dương

Bài tập 20 (P33) Hỏi mặt phẳng, vẽ nhiều tam giác mà hai tam giác (trong số đó) có đỉnh chung khơng có điểm đỉnh chung tất tam giác

Bài tập 21 (PTNK 2017-2018) Gọi S tập tập X = {1, 2,· · · , 2017}sao cho S

không chứa hai phần tử mà phần tử chia hết cho phần tử khơng chứa hai phần tử ngun tố Tìm số phần tử lớn củaS

Bài tập 22 Cho bảng vng2017×2019 Ta tơ màu vuông đơn vị bảng

màu đỏ Hỏi ta tơ nhiều vng cho mội hình vng2×2cả bảng

(80)

SỐ CHÍNH PHƯƠNG

TRONG BIỂU DIỄN CÁC SỐ NGUYÊN

Nguyễn Quang Minh

(Saint Joseph’s Institution, Singapore)

GIỚI THIỆU

Khơng nhà tốn học kể từ thời Diophantus băn khoăn với câu hỏi: số biểu diễn dạng tổng hai số phương Thế sau toán giải quyết, người lại đề loạt câu hỏi liên quan đến biểu diễn số qua tổng bình phương bên cạnh kết sâu sắc khai phá, nhiều giả thuyết bỏ ngõ ngày Trong viết này, xin bạn đọc lược lại số kết liên quan đến chủ đề

1 Tổng hai số phương

Qua thư gửi ngày 9/12/1632, nhà toán học người Hà Lan A Girard (1595 – 1632) tuyên bố rằng, số ngun tố có dạng4nC1, số phương, tích số nguyên tố số phương, hai lần số biểu diễn dạng tổng hai số phương Ít lâu sau, thư khác gửi cho Mersenne vào ngày 25/12/1640, Fermat nhìn lại kết bổ sung thêm nhiều nhận xét liên quan Ông tuyên bố chứng minh chặt chẽ kết lại không ghi Sau nhiều nỗ lực đáng kể, chứng minh thuộc Euler công bố vào năm 1754 Trong phần này, sau làm quen với số toán bổ đề, ta khai phá nhận xét Girard

Ví dụ (Putnam Mathematical Competition 2000) Chứng minh tồn vô số số nguyên nsao cho sốn; nC1nC2đều tổng hai số phương.

Lời giải Ta chứng minh rằng: với số ngun dươngmkhơng phải số phương, tồn vô số bộmC1số nguyên liên tiếp mà số tổng củamsố phương Thật vậy, phương trình Pell

x2 my2D1

(81)

Ví dụ (IMO Shortlist 1978) Chứng minh với số nguyên dương x; y; z thoả mãn xy z2 D1, tồn số nguyên không âma; b; c; d sao cho

xDa2Cb2; y Dc2Cd2; z DacCbd:

Từ chứng minh số nguyên tốp D4qC1ln biểu diễn dạng tổng hai

số phương.

Lời giải Xét số nguyên dươngz0 Giả sử mệnh đề với ba số.x; y; z/(vớiz < z0), ta chứng minh vớiz Dz0:

Dễ thấy ta cần xét số.x0; y0; z0/vớiz0 > 1vàx0 < y0 Đặt x Dx0; y Dx0Cy0 2z0; z Dz0 x0:

Khi đó.x; y; z/là số nguyên dương thoả mãnxy z2 D 1và z < z0 Theo giả thiết, tồn số nguyên a,b,c,d cho

xDa2Cb2; y Dc2Cd2; z DacCbd: Từ ta tính

x0 Da2Cb2; y0 D.aCc/2C.bCd /2; z0 Da.aCc/Cb.bCd /: Tiếp theo, đặtz D.2q/Švà áp dụng định lý Wilson, ta có

z2 D.2q/Š.2q/.2q 1/ : : : 1.2q/ŠŒ 2qC1/Œ 2qC2/ : : : 4q/ 1/2q.4q/Š mod p/:

Vậyp ước củaz2C1, theo kết thìp biểu diễn dạng tổng hai số phương

Nhận xét Sử dụng kiến thức liên quan đến vành số nguyên Gauss, ta chứng minh kết tổng quát sau : Nếua; b; c; d số nguyên dương choa2Cb2 Dcd, khi tồn số nguyênx; y; z; w; t thoả mãn

aDt xz yw/; b Dt xwCyz/; c Dt x2Cy2/; d Dt z2Cw2/:

Tiếp theo, ta chứng minh bổ đề quan trọng sau nhà toán học người Na Uy, Axel Thue (1863 – 1922)

Bổ đề (Thue) Nếuplà số nguyên tố vàalà số ngun khơng chia hết chopthì phương trình

đồng dư

axy mod p/

(82)

Lời giải Đặtb D bppcthì ta thấy tồn tại.bC1/2 > psố nguyên có dạngax y, đóx; y2 0; 1; : : : ; b Vậy tồn hai cp.x1; y1/Ô.x2; y2/sao cho

ax1 y1 Dax2 y2 mod p/: Đặtx0 Dx1 x2; y0 Dy1 y2 Khi

ax0 y0 mod p/; và0 < jx0j; jy0j <pp:

Bổ đề sau tổng quát hoá trường hợp riêng có lẽ quen thuộc với nhiều bạn đọc

Bổ đề Giả sửp D.2kC1/2t C1là số nguyên tố, vớit là số nguyên dương vàklà số

tự nhiên Nếux; ylà số tự nhiên màx2t Cy2t ::: pthìxy đều chia hết chop: Lời giải Giả sửxvàyđều khơng chia hết chop Theo định lý Fermat nhỏ, ta có

xp 1Cyp modp/: Mặt khác,

xp 1Cyp D.x2t/2kC1C.x2t/2kC1 ::: x2t Cy2t ::: p; dẫn đến mâu thuẫn Ta hoàn tất chứng minh

Định lý (Fermat) Số nguyên tố p có thể biểu diễn dạng tổng hai số chính

phương khipD2hoặcp mod 4/: Lời giải Nếup D2thìp D12C12:

Ta thấy rằng, từ Bổ đề cho trường hợp t D 1, số nguyên tố pkhông thể biểu diễn dạng tổng hai số phương nếup mod 4/:

Nếup D4kC1thì theo tiêu chuẩn Euler:

1 p

D 1/p21 D1;

hay tồn số nguyênathoả mãna2 1 mod p/ Mặt khác, theo Bổ đề Thue tồn các số nguyênx; y, với0 <jx0j; jy0j<ppvà

axy mod p/)a2x2 x2 y2 mod p/: Vậy2p > x2Cy2/: p, suy ra:: p Dx2Cy2 Hoàn tất chứng minh

(83)

Định lý Nếup D4kC1là số nguyên tố thìp Dx2Cy2, với0 <jx0j; jy0j<pp x

2 2k

k

!

.mod p/; y.2k/Šx mod p/:

Gauss không chứng minh điều này, chứng minh Cauchy Jacobsthal sau không đơn giản

Tiếp theo, ta quan tâm đến số cách biểu diễn số nguyên dương dạng tổng hai số phương, bắt đầu với đồng thức mang nhiều ứng dụng sau

Đồng thức Vớix1; x2; y1; y2; d số thực x12Cdy

2 1/.x

2 Cdy

2

2/D.x1x2˙dy1y2/2Cd x1y2x2y1/2:

Định lý Giả sửa; blà số tự nhiên cho trước, khơng có q cách biểu diễn số

ngun tốp thành dạng ax2Cby2, vớix; ylà số tự nhiên Trường hợpa D b D 1thì ta

bỏ qua tính thứ tự thành phần.

Lời giải Giả sử số ngun tốp có hai cách phân tích

p Dax2Cby2Dam2Cbn2; đóx; y; m; nlà số tự nhiên Ta nhận

p2 D.axmCby n/2Cab.x n y m/2 D.axm by n/2Cab.x nCy m/2: Ngoài ra,

.axmCby n/.x nCy m/Dmn.ax2Cby2/Cxy.am2Cbn2/Dp.mnCxy/: Vì có thừa số vế trái đẳng thức chia hết chop:

Nếup j.axmCby n/thì ta suy rax n y m D0nênp DaxmCby n, dẫn đến pxDax2mCbyx nDm.ax2Cby2/Dpm)x Dm; y Dn:

Nếup j.x nCy m/thì ta suy raaxm by nD0vàp2 Dab.x nCy m/2 Dễ thấyaDb D1 nênp Dx nCy mvàxm y nD0 Suy

px Dx2nCxy mDn.x2Cy2/Dpn)xDn; y Dm: Hoàn tất chứng minh

Bổ đề Số tự nhiên có dạng4k C1 > là số nguyên tố có cách

biểu diễn (khơng tính hốn vị) dạng tổng hai số phương nguyên tố cùng nhau.

(84)

Bổ đề Nếu hai số tự nhiên có dạng4kC1 > 1là tổng hai số phương tích của

chúng khơng thoả mãn điều kiện Bổ đề 3.

Lời giải Giả sửmDa2Cb2; nDc2Cd2, vớia; b; c; d nguyên Ta có mnD.acCbd /2C.ad bc/2 D.ac bd /2C.adCbc/2:

Giả sử hai biểu diễn sai khác thứ tự hạng tử thìac Cbd D ad CbchoặcacCbd D jac bdj:

Trong trường hợp đầu, ta suy a D b hoặcc D d, vô lý dom; n lẻ Trong trường hợp sau, acCbd Dac bd hoặcacCbd Dbd ac Cả hai hai số phương có tổng mnkhông nguyên tố nhau, trái với điều kiện Bổ đề Bổ đề chứng minh Trở lại Bổ đề Giả sửs D4kC1 > 1thoả mãn điều kiện Bổ đề hợp số Gọip ước nguyên tố s Nếup có dạng4t C3, theo Định lý 1, hai số phương có tổng bằngsđều chia hết chopnên không nguyên tố nhau, trái với giả thiết Vậypcó dạng 4t C1và tổng hai số phương Kết hợp với Bổ đề 4, ta suy điều mâu thuẫn với giả thiết Bổ đề chứng minh

Bổ đề Nếuplà số ngun tố có dạng4tC1thì vớik D1; 2; : : :thì sốpk có cách

biểu diễn thành tổng hai bình phương số tự nhiên nguyên tố nhau.

Lời giải Theo Bổ đề toán vớik D1:

Giả sử bổ đề đến giá trịknào Khi tồn số tự nhiênc; d nguyên tố thoảpk Dc2Cd2 Cũng tồn số tự nhiêna; bnguyên tố màp Da2Cb2 Vì

pkC1D.acCbd /2C.ad bc/2 D.ad Cbc/2C.ac bd /2:

Nếu số ad bc ac bd chia hết cho p thìcd.a2 b2/::: p Dopk D c2Cd2 và.c; d / D 1nên.c; p/ D.d; p/D1và.a2 b2/ ::: p, vớip Da2Cb2, ta suy raa; b chia hết chop, trái giả thiết

Vậy hai sốad bcvàac bd không chia hết chop Giả sử số làad bc thìac Cbd không chia hết chop chúng nguyên tố nhau, cho ta cách biểu diễn cần tìm Theo nguyên lý quy nạp tốn với k nguyên dương

Bây giả sử sốpk có hai cách biểu diễn phân biệt Đặt pk Da2Cb2 Dc2Cd2; với.a; b/D.c; d / D1vàa>b; c >d; a > c Ta có

p2k D.acCbd /2C.ad bc/2 D.adCbc/2C.ac bd /2;

(85)

Do có hai sốacCbd ad Cbc chia hết chop Nếu hai số chia hết chop tương tự trên, ta suy điều mâu thuẫn Vậy có hai số chia hết chopk

Nếu.acCbd /::: pk thìad bc D 0, mà.a; b/D .c; d / D1nêna Dc, vô lý Tương tự với trường hợp cịn lại Bổ đề chứng minh hồn toàn

Bổ đề Nếum; nlà số tự nhiên lẻ nguyên tố số tổng hai số chính

phương nguyên tố tích chúng có hai cách biểu diễn thành tổng hai số chính phương nguyên tố (khơng tính hốn vị).

Lời giải Giả sửm; nlà số tự nhiên lẻ nguyên tố vàa; b; c; d số tự nhiên thoả mãn.a; b/D.c; d / D1vàmDa2Cb2; nDc2Cd2, vớia>b; c >d Khi

mnD.acCbd /2C.ad bc/2 D.adCbc/2C.ac bd /2:

Dễ thấy biểu diễn mn phân biệt Ta cần chứng minh acCbd; ad bc/ D 1và ad Cbc; ac bd /D1:

Giả sửacCbd vàad bccó ước nguyên tố chungp Vậymhoặcnchia hết chop Giả sửm chia hết chopthì từ đẳng thức

.acCbd /.ad Cbc/Dcd mCabn;

ta thấy ahoặcb chia hết cho p Lại có m D a2 Cb2 nêna b chia hết chop, vơ lý Tương tự cho trường hợp cịn lại Bổ đề chứng minh

Từ bổ đề trên, ta thu định lý sau

Định lý Số nguyên dươngncó thể biểu diễn dạngqhoặc2q, vớiqlà tích số ngun

tố có dạng4mC1khi khinlà tổng hai số phương nguyên tố nhau. Các kết có dạng tổng quát sau, khuôn khổ viết, cách chứng minh tham khảo [2]

Định lý Đặtr.n/là số cách biểu diễn số tự nhiênn > 1thành tổng hai số phương

nguyên tố (khơng tính thứ tự thành phần) Khi đó

r.n/ D0

nếun ::: 4hoặc tồn số ngun tốp3 mod 4/là ước củan; khơng thì r.n/D2d 1;

trong đód là số ước nguyên tố lẻ phân biệt n. Định lý ĐặtnD 2rQ

psi i

Q

qti

i , vớipi mod 4/qi mod 4/ Sốncó thể biểu

(86)

Lời giải Giả sửncó thể biểu diễn dạng tổng hai số phương tồn tạiti lẻ cho nDx2Cy2Dqti

i b;

với.b; qi/D1 Theo Bổ đề thìxDqix1; y Dqix2, dẫn đếnx21Cy12 Dq ti

i b Sau số hữu hạn bước lặp lại, ta thu

xk2Cyk2Dqib; Dẫn đến điều vô lý

Đặt D D fn j n N; n D x2 Cy2g Giả sử t

i chẵn với mọii Do2 D; pi Dnên theo Đồng thức tồn số nguyênx; y thoả mãnx2Cy2 D2rQ

psi

i Doti chẵn với i nênQ

qti i Dh

2 vànD.xh/2

C.yh/2, hoàn tất chứng minh

Ví dụ (American Mathamatical Monthly) Chứng minh tồn dãy số nguyên liên

tiếp có độ dài lớn cho khơng có số tổng hai số phương.

Lời giải Do có vơ hạn số ngun tố có dạng4kC3nên vớinlớn bất kì, tồn tạinsố ngun tố có dạng4kC3, đặt làp1; p2; : : : ; pn Theo Định lý Thặng dư Trung Hoa tồn tạixsao cho

8 ˆ ˆ ˆ < ˆ ˆ ˆ :

x p1 mod p21/ x p2 mod p22/

: : :

x pn n mod pn2/

Kết hợp với Định lý 6, ta thu số xC1; xC2; : : : ; xCnkhông thể biểu diễn dạng tổng hai số phương doxCi chia hết chopi khơng chia hết chopi2: Ví dụ (Korean MO 2017) Tìm tất số nguyên tốp sao cho tồn số nguyênnvà số

nguyên dươngk; msao cho

.mk2C2/p m2C2k2/

mpC2 Dn

2 :

Lời giải Sau vài biến đổi đại số bản, tốn tương đương với việc tìm số nguyên tốpsao cho tồn số nguyên a,b,c với hai ba số khơng âm cho

p D a

Cb2Cc2 2Cabc :

Ta chứng minhp khơng có dạng4k C3 Cố định số nguyên tốp, ta giả sử.a; b; c/là nghiệm của phương trình với tổngaCbCc nhỏ Viết lại dạng phương trình bậc hai:

a2 bcp/aC.b2Cc2 2p/D0:

(87)

Nếu không, giả sử a > b > c Trường hợpc < 0, dễ thấy 2Cabc > 0, suy a; b; c/ D 1; 1; 1/vàp D3 Trường hợpc > 0thì

aD pbc

q

.pbc/2 4b2 4c2C8p

2 :

Doa>bnên

pbc

q

.pbc/2 4b2 4c2C8p >2b; 2b2Cc2>2pCpb2c:

Dob>c > 0nên

3b2 >2b2Cc2 > pb2c;

suy rap D2(loại không tồn tạim; k; ntương ứng) VậypD3hoặcp mod 4/: Với m; k; n/ D 4; 1; 0/thìp D Vớip mod4/thì tồn số nguyên dươngx; y chox2Cy2 Dp Ta chọn.m; k; n/D.p.x2 y2/; x; y/ Kết thúc tốn

2 Các số ngun tố có dạng x2 ˙ny2

Xác định số nguyên tố có dạng x2 ˙ny2 vấn đề thú vị khơng đơn giản, bạn đọc đọc nghiên cứu cách chi tiết [5] Trong phần này, xin qua kết kinh điển với lời giải sơ cấp, mở đầu giả thuyết Fermat thư gửi cho Pascal vào năm 1654

Định lý Số nguyên tố lẻpcó thể biểu diễn dạnga2C2b2với cặp số

tự nhiên.a; b/khi khip 1; mod 8/:

Lời giải Nếup Da2C2b2thì đặtb0là nghịch đảo modulopcủab, ta có ab0/2 2.bb0/2 mod p/;

hay 2là số phương modulop Áp dụng tính chất thặng dư bình phương: 1D

2

p

D

1

p p

D 1/p21 1/ p2

8 D 1/

.p 1/.pC5/

8 :

Từ ta suy đượcp mod 8/hoặcp mod 8/:

Ngược lại, giả sửp 1; mod 8/ Khi tồn tạin2Nsao chon2 mod p/ Theo Bổ đề Thue tồn tạia; b nguyên với0 < a; b <ppsao cho.nb a/::: p, suy

.n2b2 a2/ ::: p )3p > a2C2b2 Dpk; k f1; 2g: Nếuk D1thì ta có điều cần chứng minh Nếuk D2thìbchẵn vàp Da2C2 b

2

2

(88)

Để chứng minh tính cặp số.a; b/, giả sử tồn tạia; b; a0; b02 Nsao cho p Da2C2b2 Da02C2b02;

vànb aDpw; nb0 a0Dpz, vớiw; z Z Theo Đồng thức 1, p2 D.aa0C2bb0/2C2.a0b ab0/2: Vậy0 < aa0C2bb0 6p Mặt khác,

aa0C2bb0 D.nb pw/.nb0 pz/C2bb0Dbb0.n2C2/Cp.pwz nbz wnb0/; nên.aa0C2bb0/::: p, có nghĩa làaa0C2bb0Dpvàa0b ab0 D0 Từ dễ dàng thu aDa0vàb Db0 Hoàn tất chứng minh

Nhận xét Nếuplà số ngun tố có dạng8k 1hoặc8k 3và.a2C2b2/ ::: p, thìavàbcùng chia hết chop:

Từ ta đến định lý sau

Định lý Số nguyên dương n có thể biểu diễn dạng a2 C2b2 với a; b là số

nguyên với ước nguyên tốpcủa n màp 5; mod 8/, số mũ củaptrong

phân tíchnra thừa số nguyên tố số chẵn.

Lời giải Đặt n D s2m với m khơng có ước phương Giả sử với ước nguyên tố p 5; mod 8/củan, số mũ củap phân tíchnra thừa số nguyên tố số chẵn Theo Định lý 7, ước nguyên tố củamđều có dạngx2C2y2, nên theo Đồng thức ta có điều cần chứng minh

Giả sửnD x2C2y2với.x; y/ D d Xét ước nguyên tốpcủanthoảp 5; mod 8/ Đặt vp.n/Dc vàvp.d /Dr Ta chứng minhc D2rlà số chẵn

ĐặtxDda; y Ddb Khi đónDd2.a2C2b2/Dd2t, ta suy rav

p.t /Dc 2r Giả sử c > 2r Nếup ước củaab từ.a2C2b2/ ::: p, ta suy ra.a; b/ > p, mâu thuẫn Vậy p khơng ước củaab:

ĐặtuDabp

b2.u2C2/.abp 1/2C2b2 a2C2b2 mod p/:

Vậy.u2C2/ ::: phay 2là số phương modulop, vơ lý dop 5; mod 8/ Ta hoàn tất chứng minh

Định lý Số nguyên tố lẻpcó thể biểu diễn dạng˙p Da2 2b2với vô số cặp số

(89)

Lời giải Nếu˙p D a2 2b2và p 3; mod 8/thì doalẻ nênb2 2; mod 4/, vơ lý Tiếp tục, để ý p 1; mod 8/tương đương với2là số phương modulop, tương tự cách chứng minh Định lý 7, ta suy đượcp D2b2 a2 Mặt khác,

p D2b2 a2D.aC2b/2 2.aCb/2:

Tiếp theo, ta sử dụng hai dãy số Theon Thành Smyrna (thế kỉ I-II), biết đến hai dãy số cạnh đường chéo

s1 Dd1 D1; snC1 DsnCdn; dnC1 D2snCdn; 8n2 N: Bằng quy nạp, ta códn2 2sn2 D 1/

n

Do

p D.d2n2 2s22n/.x2 2y2/D.d2nx 2s2ny/2 2.d2nyCs2nx/2: Vậy có vơ số cách biểu diễn số nguyên tốp dạng

Nhận xét Nếuplà số nguyên tố có dạng8k 3hoặc8kC3và.a2 2b2/ ::: p, thìavàbcùng chia hết chop:

Định lý 10 Chứng minh số nguyên tố p có thể biểu diễn dạng p D x2C3y2, với x; y là số nguyên khip D3hoặcp mod3/:

Lời giải Dễ thấy nếupDx2C3y2thìp D3hoặcp mod 3/:

Ngược lại, giả sửp D3hoặc số nguyên tố có dạng3kC1 Để ý 3D02C312 Mặt khác, khiplà số nguyên tố có dạng3kC1, tồn số nguyênasao choa2 mod p/ Mặt khác, áp dụng Bổ đề Thue tồn tạix; ynguyên thoả mãn0 < x; y <pp vàp j.a2y2 x2/ Từ suy ra4p > x2C3y2 Dhp, vớih2 f1; 2; 3g:

NếuhD1thì ta suy điều phải chứng minh NếuhD2thìx2C3y2 D2p2 mod 3/, vơ lý NếuhD3thìxchia hết cho3vàpDy2C3 x32:

Bằng kiến thức tốn cao cấp, nhà toán học chứng minh đinh lý sau

Định lý 11 Số nguyên dươngncó thể biểu diễn dạngnD x2C3y2, vớix; y nguyên khi

và ước nguyên tố dạng3k 1củanđều có số mũ chẵn. Áp dụng Định lý 6, ta suy định lý sau

Định lý 12 Số nguyên tố p có thể biểu diễn dạng x2C4y2, vớix; y là số tự

nhiên khip mod 4/:

Một số kết liên quan : Vớix; ylà số nguyên dương vàp; qlà số nguyên tố thì: p Dx2 3y2 , p mod 12/

p D3x2 y2 , p 11 mod 12/ p Dx2C5y2 , p 1; mod 20/

(90)

vàp; q 3; mod 20/)pq Dx2C5y2:

Ví dụ (Korea TST 2014) Chop > 5là số nguyên tố Giả sử tồn số nguyênk sao cho k2C5chia hết chop Chứng minh tồn số nguyênm; nsao chop2 Dm2C5n2: Lời giải Ta chứng minh tồn số nguyênhthoả mãn.h2C5/ ::: p2 Theo giả thiết, tồn số nguyênasao chok2 Dap ĐặthDkCpt, vớit 2Z Do

h2Dap 5C2kpt Cp2t2;

để.h2C5/ ::: p2, ta xác địnht cho.aC2k t / ::: p Dễ thấyt tồn do.2k; p/ D1: Xét tập hợp

S D fxCyhj06x6bp4 5pc; 06y 6bp4 5pc; x; y2 Zg:

Do jSj > p2 nên tương tự cách chứng minh Bổ đề Thue, tồn số nguyênm; n cho m nh mod p2/, kết hợp với.h2C5/: p:: 2, ta thu được5p2> m2C5n2 ::: p2:

Bng cỏch xột modulo 5, ta cúm2 C5n2 Ô 2p2; 3p2 Nếum2C5n2 D p2 hoặc4p2 cặp số.m; n/và.m2;n2/thoả mãn

3 Tổng ba số phương

Diophantus nêu lên giả thuyết rằng, số ngun có dạng8mC7đều khơng tổng ba số phương, điều chứng minh Descartes vào năm 1638 Fermat người nêu lên số nguyên dương biểu diễn dạng tổng ba số phương khơng có dạng4n.8mC7/ Legendre chứng minh điều vào năm 1798, Gauss làm gọn lại vào năm 1801 Ngồi ra, Disquisitiones mình, Gauss xa cách đếm số cách biểu diễn số nguyên Cũng đáng ý rằng, vào năm 1957, sử dụng định lý Minkowski, Ankerny chứng minh lại định lý sau theo cách đơn giản

Định lý 13 (Gauss) Một số nguyên dương biểu diễn dạng tổng ba số chính

phương khơng có dạng4n.8mC7/:

Ví dụ Chứng minh số tự nhiên tổng ba bình phương số hữu tỉ là

tổng ba bình phương số nguyên.

Lời giải Giả sử số tự nhiênn tổng ba bình phương số hữu tỉ quy đồng số hữu tỉ đó, ta viết m2n D a2Cb2Cc2, vớia; b; c; mlà số nguyên Nếun D 4l.8kC7/ vớik; l số ngun khơng âm thìm2n D 4r.8t C7/vớir; t số nguyên không âm Nhưng theo Định lý 13 điều vơ lý Vậynkhơng có dạng4l.8kC7/và tổng ba số phương Điều kiện điều kiện đủ

Định lý 14 Một số nguyên dương có dạng a2 Cb2C2c2 vớia; b; c nguyên nó

(91)

Lời giải Giả sửt số nguyên không âm tuỳ ý theo Định lý 13, tồn tạix; y; z nguyên cho4t C2Dx2Cy2Cz2 Dễ thấy sốx; y; z, có hai số lẻ số chẵn, giả sửx; ylẻ vàzchẵn Khi

2tC1D

x

Cy

2

2

Cx y

2

2

C2z

2

:

Vậy số lẻ có dạnga2Cb2C2c2 Nếum ¤4k.8mC7/thìm Dx2Cy2Cz2 theo Định lý 13 Vì

2mD.xCy/2C.x y/2C2z2:

Cịn nếumD4k.16nC14/Dx2Cy2C2z2thìx; y tính chẵn lẻ 4k.8nC7/D.x Cy/2C.x y/2Cz2;

điều vô lý

Ta có hai định lý tương tự sau

Định lý 15 Một số nguyên dương có dạnga2C2b2C2c2, vớia; b; c ngun nó

khơng có dạng4k.8nC7/, vớik; nlà số tự nhiên.

Định lý 16 Một số nguyên dương có dạnga2C2b2C4c2, vớia; b; c ngun nó

khơng có dạng4k.16nC14/, vớik; nlà số tự nhiên.

Tất số nguyên dương.a; b; c/, vớia 6b 6c cho số lẻ biểu diễn dạngax2Cby2Ccz2là.1; 1; 2/; 1; 2; 3/và.1; 2; 4/ Hơn nữa,

Định lý 17 Không tồn số nguyên dương.a; b; c/, vớia6b 6c sao cho số nguyên

dương biểu diễn dạngax2Cby2Ccz2, đóx; y; zlà số nguyên. Lời giải Thật vậy, để biểu diễn số1như yêu cầu thìaD 1, để biểu diễn số2thì phải có b D 1hoặcb D2 Lại thấy,x2Cy2Ccz2 khác7nếuc D1, khác14nếuc D2, khác6nếu c D 3và khác 3nếuc > Hơn nữa,x2C2y2Ccz2 khác7nếuc D2, khác 10nếuc D 3 hoặc5, khác14nếuc D4và khác5nếuc > Hoàn tất chứng minh

Trong phần lại chuyên mục, xin giới thiệu tới bạn đọc hai phương pháp vô hữu dụng để biết xem số.a; b; c/có thể biểu diễn số nguyên

Bổ đề Cho k là số nguyên dương Đặta1; : : : ; ak là số nguyên dương vớia1 a2 : : :6ak Xét

q WDa1x12C: : :Cakxk2; q1 D0;

và vớii D2; : : : ; k thì

qi WDa1x12C: : :Cai 1xi 12 :

Nếu dạngq biểu diễn số nguyên dươngnnhưng tồn tạii f1; 2; : : : ; kgsao choqi không biểu

(92)

Lời giải Tồn số nguyêny1; : : : ; yk cho

nDa1y12C: : :Cakyk2:

Xét sối số ngun dươngnsao cho dạngqbiểu diễn sốnnhưngqi khơng Theo gi thit bi toỏn thỡ.yi; : : : ; yk/Ô.0; : : : ; 0/,

nDa1y12C: : :Cakyk2 >aiyi2C: : :Cakyk2 >ai.yi2C: : :Cyk2/>ai: Bổ đề chứng minh

Định lý 18 Nếu với ba số nguyên dương.a; b; c/, số1; 3; 515có thể biểu diễn

dưới dạng ax2Cby2Ccz2, đó x; y; z là số nguyên số lẻ biểu

diễn dạng này.

Lời giải Xét số nguyên dươnga; b; c, vớia 6b csao cho dạngax2Cby2Ccz2 biểu diễn số1; 3; 5và15 Do1có dạng nên theo Bổ đề 7,aD1:

Lại có dạngx2Cby2Ccz2 nên theo Bổ đề 7, b Khi b D 1, có dạng x2Cy2Ccz2 nên theo Bổ đề 7,1 c Khib D 2, do5có dạngx2C2y2Ccz2 nên theo Bổ đề 7,26 c 65 Khib D3thì5có dạng có dạngx2C3y2Ccz2nên theo Bổ đề 7, 6c 65 Vậy ta thu được10bộ số.a; b; c/, loại bỏ không biểu diễn đủ bốn số 1; 3; và15, ta lại ba dạng

x2Cy2C2z2; x2C2y2C3z2; x2C2y2C4z2: Nhưng theo nhận xét ta suy điều phải chứng minh

Bằng phương pháp tương tự, ta có định lý sau

Định lý 19 Nếu với ba số nguyên dương.a; b; c/, số2; 6; 10; 1430có thể biểu

diễn dạngax2Cby2Ccz2, đóx; y; zlà số nguyên số tự nhiên chia

2cũng biểu diễn dạng này.

4 Biểu diễn số dạng a2 Cb2 dc2

Sau đây, ta quan tâm đến trường hợp đặc biệt biểu diễn số qua bình phương Ta gọi số nguyên dương d đặc biệt số nguyên biểu diễn dạng m Da2Cb2 dc2, đóa; b; c là số nguyên dương.

Bổ đề Mọi số đặc biệt không chia hết cho4:

Lời giải Giả sửd số đặc biệt chia hết cho4 Khi tồn số nguyên dươnga; b; c choa2Cb2 dc2D3, suy raa2Cb23 mod 4/, trái với Định lý

(93)

Lời giải Xét số đặc biệtd Tồn số nguyên dươnga; b; c cho a2Cb2 Dd.c2C1/:

Để ý rằngd.c2C1/vàc2C1đều tổng hai số phương, nên suy từ Định lý 6,d tổng hai số phương

Bổ đề 10 Mọi số đặc biệt khơng có ước nguyên tố dạng4kC3:

Lời giải Giả sửp D4kC3là ước nguyên tố số đặc biệtd Dod tổng hai số phương theo Bổ đề 9,d ::: p2 Lại tồn số nguyên dươnga; b; c cho

a2Cb2 dc2 Dp;

nên suy ra.a2Cb2/::: p2 Do đóp ::: p2, vơ lý Vậy điều giả sử sai,d khơng có ước ngun tố dạng4kC3:

Dựa vào bổ đề trên, ta chứng minh : số ngun đặc biệt là1, tích số nguyên tố dạng 4mC1, hai lần số Ta chứng minh chiều ngược lại đúng, tức : nếud có dạngqhoặc2q, đóq D1hoặcqlà tích số nguyên tố dạng4mC1thìd số đặc biệt

Thật vậy, ta cód D1là số đặc biệt dựa vào hai đồng thức sau: 2k 1D.2t /2C.2t2 k/2 2t2 kC1/2; 2k D.2t2 2t k/2C.2t 1/2 2t2 2t kC1/2:

Xét d > lẻ Theo Định lý d D x2 C y2, với x; y/ D Đặt a D xk C ˛; b D ykCˇ; c Dk, đó˛; ˇ số nguyên Ta có

a2Cb2 dc2D2.x˛Cyˇ/kC˛2Cˇ2: Xét cặp nghiệm.˛; ˇ/của phương trình

x˛CyˇD1:

Phương trình có nghiệm ngun ˛0; ˇ0/ Ta đặt˛1 D ˛0Cy ˇ1 D ˇ0 x, ˛1; ˇ1/là nghiệm khác Ta lại chứng minh được˛02Cˇ20 ˛21 Cˇ12 mod 2/ Xét hai đồng thức

.xkC˛i/2C.yk Cˇi/2 x2Cy2/k2 D2kC˛i2Cˇ i;

(94)

Bây xétd chẵn Lại theo Định lý thìd D x2 Cy2, với.x; y/ D 1và x; y lẻ Tương tự trường hợp trước, nếux˛Cyˇ D1thì˛2Cˇ2lẻ số lẻ biểu diễn Xét phương trình tuyến tính

x˛Cyˇ D2:

Chọn cặp nghiệm.˛0; ˇ0/và xây dựng cặp nghiệm.˛1; ˇ1/theo cách tương tự, ta chứng minh

˛21Cˇ12D˛02Cˇ02C2 mod 4/: Ta có hai đồng thức

.xkC˛i/2C.yk Cˇi/2 x2Cy2/k2 D2kC˛i2Cˇ i;

trong đói D 0; 1, biểu diễn số nguyên có dạng4mvà 4mC2, nên số nguyên có dạnga2Cb2 dc2, vớia; b; c nguyên Các trường hợp ngoại lệ giải tương tự khid Dq:

Kết hợp kết trên, ta đến kết luận sau

Định lý 20 Một số nguyên đặc biệt có dạng q hoặc 2q, đó q D

hoặcqlà tích số nguyên tố dạng4mC1:

5 Tổng bốn số phương

Bachet vào năm 1621 lần nêu giả thuyết số tự nhiên tổng bốn số phương, kiểm tra điều đến số325 Mười lăm năm sau, Fermat tun bố ơng tìm cách chứng minh, thường lệ, khơng đưa chi tiết Phải đến năm 1772, Lagrange chứng minh cách đầy đủ giả thuyết trên, đồng thời thừa nhận ơng có ý tưởng từ Euler, người dành 40 năm vật lộn với toán Một năm sau đó, Euler cơng bố cách chứng minh đơn giản Trước hết, ta đến với số kết cần thiết sau Đồng thức Vớix; y; z; u; a; b; c; d số thực

.x2Cy2Cz2Cw2/.a2Cb2Cc2Cd2/Dr2Cs2Ct2Cu2;

r DxaCybCzcCud; s Dxb yaCzd uc; t Dxc yd zaCub; uDxd Cyc zb ua:

Thực ra, viết vào ngày 4/5/1748, Euler nhiều đồng thức vậy, với Đồng thức trường hợp đặc biệt Vào năm 1751, Euler tiếp tục với chứng minh bổ đề sau

(95)

Lời giải Xét hai tập hợpAD fx2 jx D0; 1; : : : ;p 12 gvàB D f y2 1jxD0; 1; : : : ;p 12 g Dễ thấy phần tử tập hợp phân biệt theo modulop, màjAjCjBj D pC1 nên tồn tạix; y f0; 1; : : : ;p 12 gsao chox2Cy2C10 mod p/:

Định lý 21 Mọi số nguyên tốpđều biểu diễn dạng tổng của4số phương. Lời giải Với nhận xét rằng2 D02C02C12C12, ta cần xét khip lẻ Đặtklà số nguyên dương nhỏ chokplà tổng bốn số phương, giả sử

kpDx2Cy2Cz2Cw2:

Theo cách chứng minh Bổ đề 11, ta suy đượck < p Ta rakD1: Nếuklà số chẵn xếp lại sốx; y; z; wsao cho

xy mod 2/; z w mod 2/: Ta thấy

k p D

x y

2

2

C

x

Cy

2

2

Cz w

2

2

C

z

Cw

2

2

; mâu thuẫn với cách chọnk:

Giả sửklà số lẻ lớn hơn1 Khi ta chọn số nguyêna; b; c; d cho

ax mod k/; b y mod k/; c z mod k/; d w mod k/; vàjaj;jbj;jcj;jdj< k2 Khi

a2Cb2Cc2Cd2Dnk; vớinlà số ngun khơng âm Ta có

06nk Da2Cb2Cc2Cd2 <

k

2

Dk2:

Ta khơng thể cón D 0vì nếunD 0thìx; y; z; w bội số củak, suy rap chia hết chok, vơ lý Từ suy ra0 < n < k Kết hợp hai đẳng thức

k2npD.x2Cy2Cz2Cw2/.a2Cb2Cc2Cd2/Dr2Cs2Ct2Cu2; với

r DxaCybCzc Cwd; s Dxb yaCzd wc; t Dxc yd zaCwb; uDxd Cyc zb wa: Ta thấy rằngr; s; t; uđều chia hết chokvà

npDr k

2

Cs

k

2

C

t k

2

Cu

k

2

(96)

Từ Định lý 21 Đồng thức 2, ta thu kết số học kinh điển sau

Định lý 22 (Legendre) Mọi số nguyên dương biểu diễn dạng tổng của số

chính phương.

Định lý 23 Mọi số tự nhiên có dạngx2Cy2Cz2C4t2, vớix; y; z; t nguyên.

Lời giải Ta cần chứng minh định lý với số nguyên dươngnkhông chia hết cho4 Theo Định lý 22 tồn tạix; y; z; wsao chon D x2Cy2Cz2 Cw2 Dễ thấy tồn số chẵn sốx; y; z; wvà suy điều cần chứng minh

Định lý 24 Mọi số tự nhiên có dạngx2C2y2C2z2C4t2, vớix; y; z; t nguyên.

Lời giải Dễ thấy Định lý với số 0; 1; Ta cần chứng minh định lý với số nguyên tốp > Lần lượt xétz D t D 0vày Dz D 0, ta thấy số nguyên tố chia8 dư1; 3; 5đều có dạng theo Định lý Định lý 12 Xétp mod 8/ Theo Định lý 22 tồn tạix; y; z; t nguyên cho

p Dx2Cy2Cz2Ct2: Ta giả sửy; z lẻ vàt chẵn,

p Dx2C2y z

2

C2

yCz

2

C4

t

2

: Hoàn tất chứng minh

Định lý 25 Mọi số tự nhiên có dạngx2C2y2C3z2C6t2, vớix; y; z; t nguyên. Lời giải Giả sửnlà số tự nhiên theo Định lý 22, tồn số nguyêna; b; c; d mà

nDa2Cb2Cc2Cd2:

Bằng phép đổi dấu thích hợp, ta giả sửaCbCc D 3zchia hết cho3 Lại giả sử ba sốa; b; c thìaCb D2klà số chẵn Ta có

3.a2Cb2Cc2/D.aCbCc/2C2

a

Cb

2 c

2

C6

a b

2

2

; Ta lại cók cD3t chia hết cho3và đặta b D2y Viết lại ta

a2Cb2Cc2D3z2C6t2C2y2; nênnDd2C2y2C3z2C6t2:

(97)

6 Tổng bình phương dương

Ta nói số có tính chất Sm tổng bình phương m số nguyên dương Sử dụng Định lý 13, ta chứng minh định lý sau

Định lý 26 Số ngun dươngncó tính chấtS4khi khinkhông thuộc dãy số 1; 3; 5; 9; 11; 17; 29; 41; 24h; 64h; 144h;

vớihD0; 1; 2; : : :

Do khuôn khổ viết nên bạn đọc tham khảo lời giải [6] Tiếp theo, ý 3; 9; 11; 17; 29; 41; ; 64h; 144hđều có có tính chấtS

3còn1; 5và số2n, vớinD1; 2; : : : khơng

Định lý 27 Số ngun dươngn có tính chấtS3hayS4 khi khơng phải số1;

24h, vớihD0; 1; 2; : : :

Định lý 28 Tất số tự nhiên khơng có tính chấtS5

1; 2; 3; 4; 5; 6; 7; 8; 9; 10; 12; 15; 18; 33:

Lời giải Từ Định lý 26 số lẻ lớn hơn41đều có tính chất S4 Vậy số chẵn lớn 42 số lẻ lớn 45 có tính chất S5 Ta có số 4; 7; 10; 12; 15; 16; 18; 19, 20; 21; 22; 23; 25; 26; 27; 28; 30; 31; 33; 34; 35; 36; 37; 38; 39; 40; 42; 43; 44đều có tính chấtS4 Do cộng thêm hoặc4 vào số ta nhận sốS5 Ta kiểm tra số lại, 1; 2; 3; 4; 6; 7; 9; 10; 12; 15; 18; 33khơng có tính chấtS5:

Ta kết thúc phần lý thuyết viết kết sau

Định lý 29 Nếu m>6là số tự nhiên tất số ngun dương khơng có tính chấtSm 1; 2; 3; : : : ; m 1; mC1; mC2; mC4; mC5; mC7; mC10; mC13:

Lời giải Xét số nguyên dươngn mC13 Giả sử tồn số nguyên dươnga1 > a2 > : : :>amvà

nDa21Ca

2C: : :Ca m:

Doa21 C.m 1/ n mC13nêna1 Nếu a1 D 1thìm D n Giả sửa1 D 2thì có nhiều ba sốa2; a3; : : : ; ambằng2vìn6mC13 Từ tìm tương ứng nDmC3; mC6; mC9; mC12 Bây xéta1 D3thìa2 62 Nếua2 D1thìnDmC8 Nếua2 D2thìa3 Da4 D: : :Dam D1vàm DnC11:

(98)

7 Bài tập tự luyện

Bài tập Vớinlà số nguyên dương bất kì, chứng minh đồ thị phương trình đường trịnx2Cy2 D nđi qua điểm hữu tỉ qua điểm nguyên vô số điểm hữu tỉ khác

Bài tập Chứng minh với số nguyên tốp số nguyênk, tồn số nguyênn k mod p/sao chonlà tổng hai số phương

Bài tập Cho p > số nguyên tố a số nguyên dương không chia hết cho p Chứng minh số sau biểu diễn dạng tổng hai số phương

pCa; 2pCa; 3pCa; : : : ;p

2 pCa:

Bài tập (Iran MO 2013, vòng 3) Chop > 3là số nguyên tố Chứng minh tồn số nguyên x,y cho

p D2x2C3y2 khip 5; 11 mod 24/:

Bài tập (Iran TST 2019) Cho số nguyên dươngn, định nghĩa tập hợpSngồm số nguyên sau

SnD fx2Cny2 jx; y2 Zg:

Tìm tất số nguyên dươngnsao cho tồn phần tử củaSnkhơng thuộc tập hợp tập hợpS1; S2; : : : ; Sn 1:

Bài tập Hỏi có nhiều số tự nhiên liên tiếp mà số biểu diễn dạng tổng ba số phương?

Bài tập Chứng minh số nguyên dương có dạngx2C2y2C9z2, vớix; y; znguyên, chia8dư4:

Tài liệu

[1] Hardy, Wright,An Introduction to the Theory of Numbers, Oxford, 1954

[2] Richard A Mollin, Fundamental Number Theory with Applications, Chapman and Hall/CRC, 2008

[3] Dorin Andrica, Ion Cucurezeanu, Titu Andreescu, An Introduction to Diophantine Equa-tions: A Problem-Based Approach, Springer, 2010

[4] Peter D Schumer,Introduction to Number Theory, Brooks - Cole Publishing Co., 1995 [5] David A Cox,Primes of the Formx2Cny2, Wiley – Interscience, 1997

(99)

[7] Peter Cho-Ho Lam, Representation of Integers Using a2 Cb2 dc2, Journal of Integer Sequences, Vol 18, 2015

[8] Kenneth S Williams,A “Four Integers” Theorem and a “Five Integers” Theorem, Ameri-can Mathematical Monthly, 2015

[9] L E Dickson, Integers represented by positive ternary quadratic forms, Bulletin of the American Mathematical Society, 1927

(100)

NHỮNG BÀI TOÁN SƠ CẤP TRONG KÌ THI IMC

Ngơ Hồng Anh

(Sinh viên École Polytechnique, Pháp)

GIỚI THIỆU

Nhắc đến kì thi Olympic Tốn học dành cho Sinh viên tồn giới, khơng thể khơng kể đến kì thi IMC (Intcernational Mathematics Competition for Univer-sity Students), kì thi lớn quy tụ học sinh đến từ nhiều quốc gia vùng lãnh thổ, trải rộng khắp châu lục Mặc dù kì thi Olympic tốn dành cho Sinh viên, có nhiều toán IMC phát biểu đơn giản, nhẹ nhàng, ý tưởng ẩn chứa đằng sau lại sâu sắc không cần dùng nhiều công cụ tốn học cao cấp Trong khn khổ viết này, nghiên cứu, đào sâu mở rộng số toán, sử dụng tuý kiến thức Toán học sơ cấp Đồng thời, trình này, tác giả giới thiệu số khái niệm, định lý hướng mới, có ích việc tiếp cận toán sơ cấp khác

Do giới hạn khuôn khổ viết không cho phép, tác giả cung cấp hết kiến thức liên quan, chứng minh hay ứng dụng vài phương pháp sử dụng tốn Chính vậy, bạn đọc có hứng thú tham khảo phần Tài liệu để nắm rõ phương pháp ứng dụng

Mặc dù trải qua trình biên tập kĩ lưỡng, nhiên, sai sót khơng thể tránh khỏi Tác giả mong nhận đóng góp bạn đọc để viết thêm phần hoàn thiện

Nếu bạn đọc có câu hỏi hay ý kiến đóng góp liên quan đến viết, xin vui lịng liên hệ với tác giả thơng qua email: hoang-anh.ngo@polytechnique.edu

1 Giới thiệu kì thi

(101)

đến năm 1999, IMC hoạt động Dự án Structural Joint European Union TEMPUS với tiêu đề "Modular Education in Mathematics and Informatics" Đây đầu tàu dự án TEMPUS Bulgaria vào thời điểm đó, với mong muốn đưa chương trình Đại học mơn Tốn Tin học Bulgaria sánh vai với trường Đại học khối Liên minh Châu Âu, nhằm chuẩn bị cho gia nhập Bulgaria vào tổ chức Trường University College London "nhà thầu" dự án này, Giáo Sư John E Jayne, Giáo sư Khoa Tốn trường, người đứng đầu dự án, đồng thời Chủ tịch kì thi từ năm 1994

Đến năm 1998, kì thi IMC lần thứ năm chuyển đến Blagoevgrad, Bulgaria đồng tổ chức Trường Đại học Tây Nam "Neofit Rilski" Blagoevgrad Trường Đại học Mỹ Bulgaria (American University in Bulgaria), với tham dự 80 thí sinh đến từ quốc gia Từ năm 1999 đến 2009, kì thi IMC tổ chức nhiều trường Đại học nhiều Quốc gia khác nhau, chẳng hạn Hungary, Anh, Cộng hoà Sec, Ba Lan, Romania, Macedonia, Ukraine, Budapest Đến năm 2010, Kì thi cố định tổ chức American University in Bulgaria, với hỗ trợ Trường Đại học Tây Nam "Neofit Rilski" Tính đến thời điểm tại, có 200 trường Đại học đến từ 50 Quốc gia tham gia vào Kì thi IMC, kể từ lần Kì IMC gần (lần thứ 25, 2018), có 351 thí sinh tham gia đến từ 70 đội tuyển khác Trong kì thi này, có nhiều giải thưởng khác trao, bao gồm Giải cá nhân, Giải đồng đọi, Giải Fairplay Giải Trưởng đoàn hiệu (dành cho trường đồn đồn có điểm tăng cao sau khiếu nại)

Kì thi IMC diễn vịng ngày, thí sinh tham gia tranh tài vào hai ngày, ngày bao gồm thi tiếng với câu hỏi khác Những câu hỏi chọn đại diện đến từ trường Đại học tham gia tranh tài Các toán nằm lĩnh vực Đại số, Giải tích (Thực Ảo), Tổ hợp Lý thuyết số

Các học sinh tham gia Kì thi cung cấp chỗ khu sinh hoạt chung, yêu cầu chỗ suốt trình diễn kì thi Mục đích việc nhằm tạo môi trường thân thiện, thoải mái an tồn cho sinh viên Tốn tồn giới thưởng thức Toán học bên cạnh người bạn toàn giới, mở rộng tầm nhìn truyền cảm hứng để đặt mục tiêu cho thân mà trước họ chưa nghĩ đến Đáng ý, vào năm 2018, nhà Tốn học tham gia kì thi IMC năm 2000 tổ chức University College London trao giải thưởng danh giá làng Toán học giới, Giải thưởng Fields

2 Những tốn sơ cấp điển hình

2.1 Tổ hợp

Bài toán (IMC 2016, Day 1, Problem 4) Chon > k là số nguyên dương, choF

họ tập hữu hạn với tính chất sau: 1. F có chứa nhất n

k

(102)

2 với hai tậpA; B bất kì thuộcF, hợpAS

B của chúng thuộcF

Chứng minh rằngF có chứa nhất3tập có nhấtnphần tử.

Lời giải 1. Nếun D k, có hai tập phân biệt họ với nhấtn phần tử

hợp hai tập đó, có điều phải chứng minh Kể từ bây giờ, giả sử

n > k

Cố định n k

C1tập có số phần tử làktrongF gọi chúng "tập nguồn" GọiV Flà

hợp "tập nguồn" Bởi vìV có n k

C1tập cókphần tử, cójVj > n

Gọi phần tử v V "chuẩn" nếuv thuộc n 1k 1tập nguồn Khi đó, tồn

n k

C1 k 1n 1

D n 1k

C1"tập nguồn" không chứa phần tử v Hợp chúng có n

phần tử, hợp không chứa phần tửv

Bây giờ, chứng minh rằng, bất kìnphần tửx1; x2; : : : ; xnnào củaV, tồn

phần tử "chuẩn" Xét tất cặp.G; xi/sao choG "tập nguồn" vàxi G.Mỗi "tập

nguồn" có k phần tử, nên số cặp tối đa nk

C1

k Nếu phần tửxi

khơng "chuẩn",xi nằm nhấtn n 1k 1

C1

tập nguồn Tuy nhiên,n k 1n

C1

>

n k

C1

k, nên điều xảy ra; vậy, phần tử x1; x2; : : : ; xnphải phần tử "chuẩn"

VìjVj > n, tậpV chứa phần tử chuẩnv1 Gọi U1 F giao tất tập

nguồn" khơng chứa phần tửv1 Khi đó,jU1j>nvàv1 …U1 Bây giờ, lấy phần tử

"chuẩn"v2 từU1và gọiU2 2F giao tất "tập nguồn" không chứa phần tửv2 Khi

đó,U2 >n, bây giờ, s cú ba tpV;U1vU2thucF vi ớt nhtnphn t:V ÔU1

bởi vìv1 2V vàv1 …U1, vàU2 khácV vàU1bởi vìv2 2V;U1nhưngv2 …U2

Lời giải 2. Chúng ta giải toán quy nạp theok, nên giả sử

phát biểu đề với giá trịknhỏ Chúng ta chứng minh phản chứng,

bằng cách giả sử rằngF có hơn3tập với nhấtnphần tử, tức số tập có tính chất

vậy là0; 1hoặc2 Khơng tính tổng qt, giả sử rằngF chứa

đúngN D nk

C1tập phân biệt với k phần tử tất hợp tập hợp Đặt

tập cókphần tử làS1; S2; : : :

Xét tập tối đaI f1; : : : ; Ngsao choADS

i2I Si có số phần tử nhỏ hơnn, hayjAj < n

Điều có nghĩa rằng, thêm tậpSj (j I) làm cho số phần tử lớn

hơn n, hay jSj SAj > n Đầu tiên, chứng minh tồn sốj

như Giả sử ngược lại; đó, tất tậpSi chứa trongA Tuy nhiên, có

jAj

k

6 n k

< N tập cók phần tử phân biệt củaA, điều trái với giả thiết ban đầu

Chính vậy, tồn sốj chojSjSAj>n

Xét tất tập có dạngSjSAvớij … I Mỗi tập có nhấtn phần tử, nên số tập dạng

này là1hoặc2 Nếu có hai tập dạng này, tạm gọi chúng làB vàC,

đó B C hoặcC B, khơng, hợp cũa hai tập B C khác B C,

đó có ba tậpB, C BS

C có nphần tử Chúng ta thấy rằng,

(103)

tậpSj0 DSj X vớij …I Trong tập này, tập cók 1phần tử Số tập

nhất

N n k

!

D n

k

!

C1

Theo giả thiết quy nạp, lập3tập, tập có nhấtn 1phần tử cách lấy

hợp tậpSj0 vớij …I Thêmx vào tập này, thấy hợp tương ứng

các tậpSj0 có nhấtnphần tử Đến đây, hoàn thành bước quy nạp

Các bước lý luận cho phép giảmkxuống đếnk D0, nên cơng việc

lại kiểm tra phát biểu đề với trường hợpk D Đề giả sử

chúng ta có n

C1 D 2tập có0phần tử Điều khơng thể xảy ra, có

nhất tập rỗng Chính vậy, phát biểu đề với trường hợpkD0

Bài toán (IMC 2018, Day 2, Problem 8) ChoD f.x; y; z/2Z3 WyC1>x >y >z > 0g Một cóc di chuyển điểm trong bằng bước nhảy với độ dài1 Với mỗi

số nguyên dương n, tính số đường mà cóc để đến điểm.n; n; n/ từ điểm 0; 0; 0/với đúng3nbước nhảy.

(Đề xuất Fedor Petrov Anatoly Vershik, Đại học Bang St Petersburg) Lời giải. Đặt ‰ D f.u; v/ Z2 W v > 0; u > 2vg Chúng ta ý ánh xạ: W  !

‰; x; y; z/ D x Cy; z/ song ánh hai tập hợp; ra, chiếu tất

đường mà cóc thành đường điểm tập‰, sử dụng bước

nhảy bước nhảy đơn vị Chính thế, bây giờ, quan tâm đến số đường từ điểm

.0; 0; 0/ D 0; 0/ đến điểm n; n; n/ D 2n; n/, sử dụng bước nhảy bước nhảy 1; 0/và.0; 1/:

Với điểm.u; v/bất kì tập‰, đặtf u; v/là số đường đi từ điểm.0; 0/đến

điểm.u; v/trong‰với đúnguCvbước nhảy Rõ ràng, cóf 0; 0/D1 Mở rộng định

nghĩa với điểm với trường hợpv D1và2v DuC1, có:

f u; 1/D0; f 2v 1; v/D0 (1)

Để đến điểm u; v/trong‰ ngoại trừ điểm gốc.0; 0/, di chuyển

từ điểm.u 1; v/hoặc điểm.u; v 1/, hay

f u; v/Df u 1; v/Cf u; v 1/với.u; v/2 Pf.0; 0/g (2)

Nếu không xét đến điều kiện biên S1/, có lớp nhiều hàm sỗ thoả

mãn phương trình hàm S 2/ Chẳng hạn như, với số nguyên c, u; v/ 7! uvCCvc

sẽ hàm số thoả mãn điều kiện trên, thích thêm hàm số

nếuvCc âm lớn hơnuCv:

Với điều kiện2v DuC1, có uCvvD 3v 1v

D2 3v 1v 1 D2 uv 1Cv Chính thế, hàm

số:

f.u; v/D uCv

v

!

2 uCv v

(104)

sẽ thoả mãn điều kiện.3/; 4/và đồng thờif 0; 0/D1 Những điều kiện đặc trưng cho

hàm sốf, vậy,f Df:

Đến đây, tính số đường mà cóc từ điểm.0; 0; 0/đến.n; n; n/

là:

f n; n; n//Df 2n; n/ D 3n

n

!

2 3n n

!

D 3n

n

2nC1

Bài toán (IMC 2014, Day 2, Problem 5) Với số nguyên dươngn, gọiDnlà số hoán vị

.x1; : : : ; xn/của.1; 2; : : : ; n/sao choxj Ôj vi mi1>j >n Vi1>k > n2, t.n; k/

là số hoán vị.x1; : : : ; xn/của.1; 2; : : : ; n/sao choxi DkCi với mi1 6i kvxj Ôj

vi mi16j 6n Chng minh rằng: .n; k/D

k

X

iD0

k i

!

D.nC1/ kCi /

n kCi /

(Đề xuất Combinatorics; Ferdowski University of Mashhad, Iran; Mirzavaziri) Lời giải. Đặtar fi1; : : : ; ikg \ fa1; : : : ; akg Khi đó,ar D is với sốs ¤ r Lúc

này, có hai trường hợp xảy ra:

Trường hợp 1: as fi1; : : : ; ikg Đặt as D it Trong trường hợp này, phép dời

chỗ x D x1; : : : ; xn/ thoả mãn điều kiện xij D aj phép dời chỗx D

.x10; : : : ; xi0t 1; x

itC1; x

n/của tpnn fitgtho iu kinxi0j D a0j vi mij Ô t

Bây giờ, có mối quan hệ - phép dời chỗx D.x1; : : : ; xn/

củaŒn với xij D aj với hai tập cho sẵnfi1; : : : ; ikgvà fa1; : : : ; akgcól phần tử giao hai tập hợp đó, với phép dời chỗx0D.x01; : : : ; xi0t 1; x

0

itC1; x

n/củaŒnn fitg

vớixij Da

j với hai tập cho sẵnfi1; : : : ; ikg n fitgvàfa01; : : : ; a0kg n fat0gcól 1phần tử

trong giao hai tập hợp

Trường hợp 2:as … fi1; : : : ; ikg Trong trường hợp này, phép dời chỗxD.x1; : : : ; xn/

thoả mãn điều kiệnxij Daj phép dời chỗx

0 D.x0

1; : : : ; xas 1; x

asC1; xn/ tậpŒnn fasgthoả mãn iu kinxi0j D aj vi mij Ôs Bõy gi, chỳng ta có

mới quan hệ - phép dời chỗx D.x1; : : : ; xn/củaŒnvớixij Daj với hai tập cho sẵnfi1; : : : ; ikgvàfa1; : : : ; akgcólphần tử giao hai tập hợp đó, với

các phép dời chỗx0 D x10; : : : ; xi0t 1; xi0tC1; xn0/củaŒnn fasgvới xij D aj với hai tập cho sẵnfi1; : : : ; ikg n fitgvà fa1; : : : ; akg n fasgcól 1phần tử giao hai tập

hợp

Những lý luận cho thấy rằng.n; k; l/ D .n 1; k 1; l 1/ Lặp lặp lại

quá trình này, được:

(105)

Đến đây, giả sử rằngl D Chỳng ta s xỏc nh.n; k; 0/, vi2k Ô n Với k D0, rõ ràng có được.n; 0; 0/DDn Vớik>1, có:

.n; k; 0/D.n 1; k 1; 0/C.n 2; k 1; 0/

Với phép dời chỗx D.x1; : : : ; xn/thoả mãnxij Daj, cú hai trng hp xy ra:xa1 Di1 hocxa1 Ôi1:

Nếu trường hợp xảy ra, phải xác định số phép dời chỗ tậpŒnfi1; a1gvới

hai tập cho sẵn fi2; : : : ; ikg fa2; : : : ; akg với0 phần tử giao hai tập hợp Số phép

dời chỗ là.n 2; k 1; 0/:

Nếu trường hợp thứ hai xảy ra, phải xác định số phép dời chỗ tậpŒnfa1gvới hai

tập cho sẵn fi2; : : : ; ikgvà fa2; : : : ; akgvới 0phần tử giao hai tập hợp Số phép dời

chỗ là.n 1; k 1; 0/:

Chúng ta dùng quy tạp theokđể chứng minh rằng: .n; k; 0/D

k

X

iD0

k i

!

D.nC1/ k i /

n kCi /; 262k 6n

VớikD1, có:

.n; 1; 0/D.n 1; 0; 0/C.n 2; 0; 0/ DDn 1CDn D

Dn

n

Chúng ta giả sử kết vớik Chúng ta viết: .n; k; 0/D.n 1; k 1; 0/C.n 2; k 1; 0/

D k

X

iD0

k i

!

Dn k 1Ci /

.n 1/ k 1Ci / C

k

X

iD0

k i

!

D.n 1/ k 1Ci /

.n 2/ k 1Ci /

D k

X

iD0

k i

!

D.nC1/ kCi /

n k Ci / C

k

X

iD0

k i

!

Dn kCi 1/

.n 1/ kCi 1/

D D.nC1/ k

n k C

k

X

iD1

k i

!

D.nC1/ kCi /

n kCi /

C D.nC1/ 2k 1/

n 2k 1/ C

k

X

iD1

k i

!

D.nC1/ kCi /

n kCi /

D D.nC1/ k

n k C

k

X

iD1

"

k i

!

C k

i

!#

D.nC1/ kCi /

n kCi / C

D.nC1/ 2k 1/

n 2k 1/

D D.nC1/ k

n k C

k

X

iD1

k i

!

D.nC1/ kCi /

n kCi / C

D.nC1/ 2k 1/

n 2k 1/

D k

X

iD0

k i

!

D.nC1/ kCi /

(106)

Nhận xét Như hệ tất yếu toán trên, đặtn D2k; ij Dj vàaj DkCj với

j D 1; ::; k Khi đó, phép dời chỗ x D x1; : : : ; xn/thoả mãn điều kiện xij D aj khix0D.xkC1; : : : ; xn/là phép chuyển vị củaŒk Số phép chuyển vịx0như

làkŠ Như vậy, có:

k

X

iD0

k i

!

DkC1 i

k i DkŠ

Đây kết qủa trực tiếp sử dụng để giải toán ngày thi thứ hai kì thi năm

Ngồi lời giải thức, tác giả muốn giới thiệu lời giải mới, dù có đơi chút cồng kềnh mặt tính tốn lời giải có phần tự nhiên, trực quan mà xét nhiều trường hợp xảy

Lời giải phụ. Đặt.n; k; j /là số phép chuyển vị trongSnsao choxi D kCi với16i 6k

và có xácj điểm cố định (tức làxj Dj) Khi đó,.n; k/D.n; k; 0/và rõ ràng rằng,

X

j

.n; k; j / D.n k/Š

Mặt khác, có

.n; k; j /D n 2k

j

!

.n j; k/

Từ đó, có đẳng thức sau:

X

j

n 2k j

!

.n j; k/ D.n k/Š (3)

Để ý chọnn D 2k, sau n D 2kC1, v.v, tính được.n; k/

cách đệ quy phương trình này, nên phương trình (đúng với phép chọnn; k)

cho cách xác định .n; k/ cách Đến đây, cần chứng

minh rằng: X j n 2k j ! X i k i !

Dn k i j

n k i j D.n k/Š

Tuy nhiên, sử dụng công thức Vandermonde sau:

mCn r

!

D r

X

kD0

m k ! n r k ! ;

chúng ta viết lại vế trái sau:

X

s

Dn k s

n k s

X

iCjDs

n 2k j ! k i ! DX s

Dn k s

n k s

n k s

!

DX

j

DjC2

j C1

n k j

(107)

và đến đây, cơng việc cịn phải chứng minh

X

j

DjC2

j C1 m j

!

D.mC1/Š

hay tương đương với

X

j

DjC1

m j

!

DmmŠ

Ngoài ra,

m j

!

D mC1

j C1

!

m j C1

!

;

nên cần chứng minh

X

j

Dj

m j

!

DmŠ

Điều hiển nhiên để ý m j

Dn j số phép chuyển vị trongSnvới

đúngj điểm cố định, đơn giản cách chok D 0vào phương trình.S 3/và để ý

rằng.n; 0/DDn Đến đây, toán giải hồn tồn

Bài tốn (IMC 2009, Day 1, Problem 3) Trong thị trấn, hai cư dân không bạn

của nhau, họ có bạn chung, khơng có bạn với tất người lại Chúng ta đánh số dân cư thị trấn từ1đếnnva đặtai là số bạn dân cư thứi Giả sử rằng

Pn

iD1ai2 Dn2 n Đặtklà số dân cư nhỏ (ít 3) ngồi vào bàn tròn sao

cho hai người ngồi cạnh bạn Xác định tất giá trị có củak.

Lời giải. Chúng ta định nghĩa đồ thị đơn giản, không định hướng G cho tất

đỉnh củaG cư dân thị trấn cạnh củaG thể tình bạn cư dân Đặt V G/ D fv1; v2; : : : ; vnglà đỉnh củaG,ai bậc củavi với mọii ĐặtE.G/ cạnh

củaG Với thuật ngữ trên, đề yêu cầu miêu tả độ dàikcủa chu trình

ngắn củaG:

Chúng ta đếm bước với độ dài2trênG, tức tìm ba đỉnh có thứ tự.vi; vj; vl/

vớivivj; vjvl E.G/(trường hợpi Dl xảy ra) Với mỗij cho trước, số ba

hiển nhiên làa2j, nên tổng ba làPn

iD1a

i Dn

2 n:

Bây giờ, chứng minh tồn ánh xạf từ tập cặp đỉnh riêng biệt đến tập

bước Vớivivj … E.G/, đặtf vi; vj/D.vi; vl; vj/vớil tuỳ ý chovivl; vlvj E.G/

Vớivivj 2E.G/, đặtf vi; vj/D.vi; vj; vi/.f l mt ỏnh x bi vỡ vii Ôl,.vi; vj; vl/chỉ

có thể ảnh của.vi; vl/và vớii Dl, khơng thể ảnh của.vi; vj/:

Vì số cặp đỉnh có thứ tự làn2 n;Pn

iD1a i > n

2 n Đẳng thức xảy khif

toàn ánh, tức là, có mộtlvớivivl; vlvj E.G/với mỗii; j cho

vivj … E.G/ khơng tồn tạil với bất kìi; j chovivj E.G/ Nói cách khác,

(108)

là rừng (forest) (một đồ thị khơng có chu trình), độ dài ngắn chu trình là5:

Dễ dàng kiểm tra hai đỉnh "rừng" liên kết đường có độ dài lớn là2, "rừng" lúc "sao" (tức đồ thị có đỉnh liên kết với

tất đỉnh cịn lại cạnh) Nhưng,Gcónđỉnh, khơng có đỉnh có bậcn

Chính vậy,G khơng thể "rừng", tức có chu trình Mặt khác, độ dài tối thiểu

của chu trìnhC củaGlà6, có hai đỉnh cho hai cung củaC nối hai đỉnh

sẽ khơng dài hơn2 Chính vậy, có đường chúng ngắn độ dài hai

cung Thay hai cung đường đó, có đường khép kín ngắn hơnc Chính vậy, độ dài chu trình ngắn phải là5:

Cuối cùng, ý có mộtGvới tính chất cho, tức thân

chu trìnhC5thoả mãn điều kiện đề Khi đó,5sẽ giá trị củak

Nhận xét Đây sử dụng định nghĩa ứng dụng đơn giản lý thuyết đồ thị Tiếp theo, đến với tốn có nhiều ứng dụng hơn, độ phức tạp tính tốn cao

Bài tốn (IMC 2011, Day 2, Problem 2) Một tộc người ngồi hành tinh có ba giới tính:

male, female emale Một "bộ ba cưới nhau" bao gồm ba người, người có giới tính khác nhau, họ thích Một người cho phép nằm tối đa "bộ ba cưới nhau" Một đặc điểm đặc biệt tộc người cảm xúc luôn xuất phát từ hai - tức nếux thíchy,y sẽ thíchx:

Tộc người gửi đoàn thám hiểm đến xâm lược hành tinh Đồn thám hiểm này gồm cónmale,nfemale vànemale Biết thành viên đồn thám hiểm thích

ít nhấtkngười giới tính cịn lại Vấn đề tạo nhiều "bộ ba cưới nhau"

càng tốt để tạo cháu khoẻ mạnh để thuộc địa phát triển trở nên thịnh vượng.

Chứng minh nếu nlẻ và k D n2, tạo dù "bộ ba

cưới nhau".

Chứng minh nếuk > 3n4 , ln có cách để tạo ran"bộ ba cưới nhau" phân biệt,

tức tất người đoàn thám hiểm "tác hợp".

(Đề xuất Fedor Duzhin Nick Gravin, Singapore) Lời giải.

ĐặtM tập người có giới tính male,F tập người có giới tính female,

và E tập người có giới tính emale Xét đồ thị tripartite G với đỉnh M [F [E cạnh thể mối quan hệ thích Một chu trình độ dài3,

lúc này, "bộ ba cưới nhau" Chúng ta gọiG đồ thị mối quan hệ thích

nhau

(109)

của mối quan hệ thích mà khơng có chu trình có độ dài3nào Chúng ta thực

như sau: chia tậpM; F vàE thành hai phần vẽ tất cạnh

hai phần sau:

Rõ ràng, đồ thị khơng có chu trình có độ dài3nào

Đầu tiên, chia đoàn thám hiểm thành ba male - female - emale tuỳ

ý Gọi độ "không hạnh phúc" phép chia số cặp người hành tinh thuộc ba, khơng thích Chúng ta chứng minh rằng, độ khơng hạnh phúc số dương độ khơng hạnh phúc giảm phép biến đổi đơn giản Từ đó, sau nhiều bước biến đổi, độ "không hạnh phúc" giảm về0, tức tất ba lúc "bộ ba cưới nhau", tạo đám

cưới hạnh phúc cho tất người!

Giả sử có emale khơng thích thành viên ba (những trường hợp khác, thao tác tương tự) Chúng ta thực phép biến đổi sau: đổi emale với emale khác, cho emale số hai người thích thành viên ba Khi đó, độ "khơng hạnh phúc" liên quan đến emale giảm xuống, cặp khác liên quan đến độ "không hạnh phúc" cách xếp khơng thay đổi, nên nhìn chung, độ "khơng hạnh phúc" giảm xuống

Vì vậy, việc cịn lại chứng minh phép biến đổi ln ln xảy Đánh số ba từ1; 2; : : : ; n gọiEi; Fi; Mi tương ứng emale,

female emale ba thứ i Khơng tính tổng qt, giả sử E1khơng thích hoặcF1, hoặcM1, hai Chúng ta phải tìm sối > 1sao cho

Ei thích cặp đơi F1; M1 vàE1 thích cặp đơiFi; Mi; đó, hốn đổi vị

tríE1 vàEi:

Có tối đa n

4 số i cho E1 khơng thíchFi tối đa n

4 số i cho E1 khơng

thíchMi, nên khơng qn2 sối choE1khơng thích cặpMi; Fi,

và tập số không phù hợp bao gồm1 Tương tự, có tối đa n2 sối cho

hoặcM1 hoặcF1 không thíchEi Vì hai tập sối khơng thoả mãn

chứa tối đa n

2 phần tử có chứa1, có sối thoả mãn tất

(110)

Lời giải 2. Giả sử k > 3n4 chứng minh "tác hợp" cho tất

thành viên đoàn thám hiểm Đầu tiên, chứng minh tồn cách "tác hợp" hoàn hảo M F Chúng ta cần kiểm tra điều kiện Định lý đám cưới (hay

biết đến với tên gọi Định lý Hall) Nói cách khác, vớiAM, đặtB F tập tất đỉnh

củaF kề với đỉnh củaA Khi đó, cần chứng minh rằngjAj6jBj Chúng ta

giả sử ngược lại, tức làjAj >jBj Rõ ràng,jBj >knếuAkhông phải tập rỗng Chúng

ta xétf F nBbất kì Khi đó,f khơng kề với đỉnh củaA, tức làf có bậc M khơng lớn hơnn jAj < n jBj < n k6 n4, mâu thuẫn

Bây giờ, xây dựng đồ thị bipartile, gọi làH Tập đỉnh làP [E,

vớiP tập cặp male - female từ cặp "tác hợp" hoàn hảo vừa tìm Chúng

ta có cạnh từ.m; f / D p P đếne E với chu trình độ dài3 m; f; e/của đồ

thịG, với.m; f / P vàe E Chú ý rằng, bậc đỉnh củaP trongH tối thiểu 2k n:

Phần lại, chứng minh H thoả mãn điều kiện Định lý đám cưới (hay Định

lý Hall) đó, có phép "tác hợp" hồn hảo Giả sử ngược lại, điều sau xảy ra: Tồn A P B E cho jAj D l, jBj < p, vàB tập tất đỉnh E kề

với đỉnh A Vì bậc đỉnh B tối thiểu 2k n, có 2k n6jBj < l Mặt khác, đặte EnB Khi đó, với cặp.m; f /Dp P, tối đa

trong cặp.e; m/và m; f /sẽ nối cạnh đó, bậc củaetrongG tối đa

sẽ làjM nAj C jF nAj C jAj D 2.n 1/Cl D 2n l Tuy nhiên, bậc đỉnh

trongGcũng là2kvà có2k 62n l, hayl 62k 2n:

Cuối cùng,2k n < l 62n 2k, tức làk < 3n4 (mâu thuẫn) Điều mâu thuẫn đưa chúng

ta đến điều phải chứng minh

Nhận xét Trong lời giải trên, sử dụng thuật ngữ đồ thị bipartile tripartile Nhằm giúp bạn đọc dễ dàng theo dõi hơn, tiến hành định nghĩa đồ thị k-partile sau:

Định nghĩa (Đồ thị k-partile) Đồ thị k-partile đồ thị có đỉnh phân

hoạc thành k tập độc lập riêng biệt Hay nói cách khác, đồ thị tơ bằngk

màu, cho khơng có hai đầu đỉnh chung màu KhikD2, gọi

đồ thị đồ thị bipartile, khikD3, gọi đồ thị đồ thị tripartile.

Chúng ta cịn có định nghĩa khác đồ thị k-partile hoàn chỉnh sau:

Định nghĩa (Đồ thị k-partile hoàn chỉnh) Một đồ thị partile hoàn chỉnh đồ thị

k-partile cho ln ln có cạnh nối hai đỉnh thuộc tập độc lập khác Những đồ thị kí hiệu chữ K dãy kích thước tập cách chia. Ví dụ,K2;2;2là đồ thị tripartile hồn chỉnh có dạng lục giác đều, chia thành ba

tập độc lập, tập gồm hai đỉnh đối nhau.

(111)

Định lý (Định lý Hall) Cho đồ thị bipartile gồm hai tập độc lập phân biệt X, Y Với tập

con A thuộc X, gọi G(A) tập đỉnh thuộc Y kể với đỉnh thuộc A Khi đó, điều kiện cần đủ để tồn đơn ánhf WX !Y sao choxkềf x/

jG.A/j>jAj8AÔ ;; AX

2.2 Gii tớch

Bi toán (IMC 2018, Day 2, Problem 7) Cho.an/C1 là dãy số thực vớia0D0

an3C1 Da2n 8:

Chứng minh dãy số sau hội tụ:

1

X

nD0

janC1 anj:

(Đề xuất Orif Ibrogimov, Đại học Quốc gia Uzbekistan) Lời giải thức. Chúng ta dự đốn tỉ số số hạngjanC2 anC1jvàjanC1 anj:

Trước đó, giới hạn số hạnganbằng cách chứng minh rằng:

26an6

3

p

4; 8n>1

Chặn suy thẳng từ quy luật dãy số:

anD

3

q

a2n 1 8> p3 8D 2:

Chúng ta chứng minh chặn cách sử dụng quy nạp: Chúng ta có a1 D

2 < p3 4, 26an< 0, có

anC1 D

q

a2

n 86

3

p

22 8D p3

4:

Bây giờ, so sánhjanC2 anC1jvàjanC1 anj Bằng cách áp dụng đẳng

thức nhưx3 y3 D x y/.x2CxyCy2/,x2 y2 D x y/.x Cy/và quy luật dãy

số, có:

.a2nC2CanC2anC1Can2C1/ janC2 anC1j D ja3nC2 a nC1j

D j.a2nC1 8/ a

n 8/j

D janC1Canj janC1 anj:

Ở vế trái, có:

an2C2CanC2anC1Ca2nC1 >342=3I

và vế phải:

(112)

Chính vậy,

janC1 anj6

4

342=3janC1 anj D

3

p

4

3 janC1 anj:

Bằng quy nạp, hiển nhiên có:

janC1 anj<

3

p

4

!n

ja2 a1j:

Đến đây, kết luận rằngP1

nD0có thể làm trội dãy cấp số nhân

với công bội p34

3 < 1, điều chứng minh dãy hội tụ

Lời giải tác giả. Đầu tiên, thông qua phương pháp tương tự Lời giải thức, suy

26an6

3

p

4; 8n>1

Chúng ta xét hàm số sau đây:f W R! R; f x/ D p3 x2 8 Dễ dàng nhận thấy, là

hàm số đặc trưng dãy số.an/1 đề Đạo hàm hàm số là:

f0.x/D 2x

3p3

.x2 8/2:

Tiếp theo, thừa nhận không chứng minh định lý sau:

Định lý (Mean Value Theorem) Choab là hai số thực choa < b Biết hàm số f WŒa; b!Rlà hàm liên tục Giả sử rằngf có đạo hàm đoạnŒa; b Khi đó, tồn tại

một số thựcc a; b/sao cho:

f b/ f a/ b a Df

0.c/

Định lý dễ dàng chứng minh cách xét hàm sốh.x/Df x/ f b/ f a/b a x:

Sử dụng định lý với x; y thuộc khoảng Œ 2; 0/, tồn số thực c x; y/

cho:

f0.c/ D f y/ f x/

y x D

2c 3p3

.x2 8/2 D

2

3q3 c

8 c3

2

Dof0.c/ > 08c 2 2; 0/, kết luận rằng, với mọix; y 2Œ 2; 0/; x < y, luôn tồn số thựccsao cho:

f0.c/D

ˇ ˇ ˇ ˇ

f y/ f x/ y x

ˇ ˇ ˇ ˇD

2

3q3

c c3

2

Bây giờ, xét hàm sốg.x/ D 8x3 để dự đoán giá trị f0.c/ Đạo hàm

(113)

nhỏ Trong trường hợp này, vìc 2; 0/, có 1c > 12, hay 1c c83 đạt giá trị cực đại

c D

1

2, hayc D Khi đó,

f0.c/6 3q3

2 2/3

2 <

ThayanC1vàanvàoxvàytuỳ ý, có:

janC2 anC1j Df0.c/janC1 anj<janC1 anjvìf0.c/ < 18c 2Œ 2; 0/

Đến đây, kết luận dãyP1

nD0janC1 anjhội tụ

Bài toán (IMC 2012, Day 1, Problem 4) Cho hàm số f W R ! R là hàm số liên

tục có đạo hàm thoả mãn điều kiện f0.t / > f f t // với mọi t R Chứng minh rằng

f f f t ///60với mọit >0.

(Đề xuất Tomas Bárta, Đại học Charles, Prague) Lời giải. Chúng ta phát biểu chứng minh bổ đề sau:

Bổ đề Hoặclimt!C1f t /không tồn tại, hoặclimt!C1f t /Ô C1.

Gi s rng gii hn ca nú C1 Khi đó, tồn T1 > cho với t > T1, chúng

ta có f t / > Sau đó, tồn T2 > cho f t / > T1 với t > T2 Vì vậy,

f0.t t / > f f t // > 28t > T2

Lúc này, tồn số T3 cho f t / > t8t > T3 Vậy nên, f0.t / > f f t // >

f t /;ff t /0.t / > Lấy tích phân hai vế, có ln.f t // ln.T3/ > t t3, hay f t / >

T3et T38t > T3 Lấy tích phân lần từ T3 đến t, có e f T3/ e f t / >

.t T3/T3e T3 Vế phải tiến đến vô cùng, vế trái bị chặn trên, vô lý Chúng ta thu

kết Bổ đề

Bổ đề Với mọit > 0, cóf t / < t.

Theo Bổ đề 1, có số số thựct chof t / < t Chính vậy, điều giả sử

là sai, tồn sốt0 > 0nào chof t0/Dt0

Trường hợp 1: Tồn số sốt > t0sao cho f t / < t0 ĐặtT Dinfft >t0 Wf t / <

t0g Vì tính liên tục củaf, cóf T /Dt0 Khi đó,f0.T / > f f T //Df t0/D

t0 > Điều có nghĩa rằngf > f T / D t0 lân cận bên phải, trái ngược với định

nghĩa củaT

Trường hợp 2: f t / > 08t > t0 Khi đó, có f0.t / > f f t // > t0 > Vì

vậy,f0 có chặn dương khoảng.t0;C1/, điều trái ngược với Bổ đề

(114)

(a) Nếuf s1/ > 0f s2/>s1, đóf s/ > s18s > s2.

(b) Trong trường hợp cụ thể hơn, nếus160f s1/ > 0, đóf s/ > s18s > s1.

Giả sử có số giá trị s > s2 chof s/ s1 đặtS D inffs > s2 W f s/ s1g

Vì tính liên tục hàm số , có f S / D s1 Tương tự với Bổ đề 2, có

f0.S / > f f S // D f s1/ > NếuS > s2, đó, lân cận bên trái S có

f < s1, trái ngược với định nghĩa S Nếu không, trường hợp S D s2, có

f > s1trong lân cận bên phải củas2, lại điều trái ngược

Phần (b) hệ tất yếu, đặts2 Ds1

Trở lại toán, với hỗ trợ Bổ đề trên, chứng minh phát biểu đề sau

Giả sử ngược lại, tồn sốt0 > 0sao chof f f t0/// > Đặt t1 D f t0/; t2 D f t1/

vàt3 Df t2/ > Chúng ta chứng minh rằng0 < t3 < t2 < t1 < t0 Theo bổ đề hai,

cót1vàt2dương Nếut1 < 0,f t1/60(nếuf t1/ > 0, chọns D t1ở Bổ đề 3(b)

đượcf t0/ > t1, mâu thuẫn) Nếut1 D0, cóf t1/6 0theo bổ đề tính liên tục

của hàmf Chính vậy, nếut1 60, t2 Nếut2 D0, f t2/ 60theo Bổ đề tính liên

tục hàm sốf (mâu thuẫn, vìf t2/Dt3 > 0) Nếut2 < 0, theo Bổ đề 3(b),f t0/ > t2, nên

t1 > t2 Áp dụng Bổ đề 3(a), cóf t1/ > t2 (mâu thuẫn) Đến đây, chứng

minh được0 < t3 < t2 < t1 < t0:

Theo Bổ đề 3(a), f t1/ > 0; f t0/ > t1/, cóf t / > t1 với t > t0, tương tự,

f t / > t2với t > t1 Từ đây, có vớit > t0,f0.t / > f f t / > t2 > Vì

vậy, limt!C1 D C1, mâu thuẫn Điều mâu thuẫn cho kết f f f t ///

08t > Vớit D0, kết nhờ vào tính liên tục hàmf

Bài toán (IMC 2014, Day 1, Problem 3) Chonlà số nguyên dương Chứng minh rằng

tồn số thực dươnga0; a1; : : : ; ansao cho với cách chọn dấu đa thức ˙anxnC ˙an 1xn 1˙: : :˙a1xCa0

nnghiệm thực phân biệt.

(Đề xuất Stephan Neupert, TUM, Munchen) Lời giải. Chúng ta chứng minh quy nạp theon Phát biểu đề hiển nhiên

với trường hợpnD1 Khi đó, giả sử có số sốan; : : : ; a0thoả mãn điều kiện

đề với sốnnào Bây giờ, xét đa thức:

Q

P x/D ˙anxnC1˙an 1xn˙: : :˙a1x2˙a0x

Theo giả thiết quy nạp v iu kina0 Ô0, mi a thc s ny cónC1nghiệm phân

biệt, bao gồmnnghiệm khác0của˙anxn˙an 1xn 1˙: : :˙a1x˙a0 Đặc biệt, khơng có

(115)

chọn số > 0nào cho với đa thứcP x/Q cực trị địa phươngxcủa chúng,

chúng ta cój QP x/j > Chúng ta có, đa thức số đa thức: P x/ D ˙anxnC1˙an 1xn˙: : :˙a1x2˙a0x˙

cũng có đúngnC1nghiệm phân biệt VìPQ cónC1nghiệm phân biệt, nhậnnđiểm cực trị địa

phương Gọi điểm cực trị địa phương Ds0 < s1 < s2 < : : : < sn< snC1 D

Vì vậy, với giá trịi f0; 1; : : : ; ng, giá trịP sQ i/vàP sQ iC1/ngược dấu (với quy ước rõ

ràng vô cực) Bằng cách chọn, điều choP si/vàP siC1/ Chính vậy, tồn

tại nghiệm thực củaP x/ khoảng.si; siC1/, hayP x/có (đồng nghĩa

với việc có xác)nC1nghiệm thực Điều cho thấy tìm thấy tập

các số thực dươnga0nC1 Dan; a0n Dan 1; : : : ; a01 D a0; a00 D thoả mãn tính chất đề

bài

Bài tốn (IMC 2006, Day 1, Problem 4) Biết rằngf là hàm hữu tỉ (tức thương của

hai đa thức hệ số thực), giả sử rằng f n/ là số nguyên với vô hạn số nguyên n.

Chứng minh rằngf là đa thức.

Lời giải ĐặtS tập vô hạn số nguyên cho hàm hữu tỉf x/có thể lấy tích phân

với mọix 2S

Giả sử

f x/D p.x/

q.x/

vớiplà đa thức bậckvàq đa thức bậcn Khi đó,p; qsẽ nghiệm hệ phương trình p.x/Dq.x/f x/

với mọix S khơng nghiệm củaq Đây hệ phương trình tuyến tính với hệ số củap; q

các đa thức với hệ số hữu tỉ Bởi chúng có nghiệm, chúng có nghiệm hữu tỉ Khi đó, có đa thứcp0; q0với hệ số hữu tỉ cho

p0.x/Dq0.x/f x/

với mọix S không nghiệm củaq Nhân phương trỉnh với phương trình trên, chúng

ta

p0.x/q.x/f x/Dp.x/q0.x/f x/

với mọix S không nghiệm cũaq Nux khụng phi l nghim cap hocq, f x/Ô 0,

và đó,

p0.x/q.x/Dp.x/q0.x/

với x S, trừ hữu hạn nghiệm củap vàq Vì vậy, hai đa thức p0q và pq0bằng nhau với vô hạn cách chọn giá trị củax Chính vậy,

p0.x/q.x/ Dp.x/q0.x/:

Chia hai vế choq.x/q0.x/, thấy rằng

p0.x/ q0.x/ D

p.x/

(116)

Chúng ta kết luận rằngf x/có thể viết dạng thương hai đa thức với hệ số hữu

tỉ Nhân hai đa thức với vài số nguyên, kết luậnf x/có thể viết dại

thương hai đa thức với hệ số nguyên Giả sử

f x/D p".x/

q".x/

vớip".x/vàq".x/đều có hệ số nguyên Khi đó, thuật tốn chia đa thức Euler, tồn

tại đa thứcs vàr, hai có hệ số hữu tỉ cho p".x/Dq".x/s.x/Cr.x/

với bậc củar nhỏ bậc củaq" Chia hai vế vớiq".x/, f x/Ds.x/C r.x/

q".x/

Lúc này, tồn số nguyênN choN s.x/ có hệ số dương Khi đó,Nf x/ N s.x/

một số nguyên với mọix 2S Tuy nhiên, hàm số lại hàm hữu tỉN r.x/q".x/ , hàm số có bậc

của mẫu cao bậc tử, nên hàm tiến đến 0khix tiến đếnC1 Vì vậy, vớix S

đủ lớn, có

Nf x/ N s.x/D0;

hayr.x/ D0 Như vậy,r có vơ hạn nghiệm, tức đa thức trùng với đa thức0 Lúc f x/Ds.x/, hay kết luận đượcf đa thức

Bài toán 10 (IMC 2015, Day 2, Question 8) Chonlà số nguyên dương, gọip.n/

một đa thức bậcnvới hệ số nguyên Chứng minh rằng:

max

06x61jp.x/j>

1 en

(Đề xuất Géza Kós, Đại học Eotvos, Budapest)

Lời giải Đặt

M D max

06x61jp.x/j

Với số nguyên dươngk, đặt

kD

Z

0

.p.x//2kdx

Rõ ràng, < Jk < M2k số hữu tỉ Nếu p.x// D P 2k n

iD0ak;ixi, Jk D P

2k n iD0

ak;i

iC1 Lấy

mẫu chung nhỏ nhất, thấy rằngJk > lcm.1;2;:::;2k n1 C1/:

Một biến thể Bổ đề số nguyên tố cho biết loglcm.1; 2; : : : ; N / N N ! Khi đấy, với > 0vàkđủ lớn, có:

(117)

M2k > Jk >

1

lcm.1; 2; ::; 2nkC1/ >

1 e.1C/.2k nC1/

M > e.1C/.nC2k1/ Đưak! C1và lấy! C0, có:

M > en

Vìe số siêu việt, dấu hồn tồn xảy

Nhận xét Hằng số 1e ước lượng chặt Hằng số ước lượng chặt cho

biểu thức biết đến nằm khoảng từ 0:4213đến0:4232 (I E Prisker, The

Gelfond-Schnirelman method in prime number theory, Canad J Math, 57 (2005), 1080-1101.) Ngoài ra, chứng minh trên, sử dụng kết tiếng Lý thuyết số, Bổ đề số ngun tố (Prime number theory - PNT) Bổ đề cụ thể hoá ý tưởng số nguyên tố xuất thưa dần chúng tiến đến vô cách ước lượng tốc độ chậm Đã có hai phép chứng minh cơng bố độc lập hai nhà toán học, Jacques Hadamard Charles Jean de la Vallée Poussin vào năm 198, sử dụng ý tưởng Barnhard Riemann, cụ thể hàm Riemann Zeta Bổ đề phát biểu cụ thể sau: Định lý (Prime number theory - PNT) Gọi.x/là hàm đếm số nguyên tố, cho kết quả

là số số nguyên tố nhỏ bằngx với số thựcx Ví dụ,.10/ D 4, có số

nguyên tố.2; 3; 5; 7/nhỏ bằng10 Định lý số nguyên tố phát biểu rằnglogx.x/ là một

ước lượng tốt của.x/, với ý nghĩa thương số hai hàm số.x/ logx.x/ khixtăng vô

hạn là1:

lim

x!C1

.x/

h

x

log.x/

i D1;

biểu thức xem uóc lượng hàm phân phối số nguyên tố.

Sử dụng dấu xấp xỉ, viết lại biểu thức sau:

.x/ x

log.x/

Bổ đề số nguyên tố tương đương với phát biểu số nguyên tố thứn; pnthoả mãn:

pn nlog.n/

Dấu xấp xỉ, lần nữa, cho biết sai số tương đối ước lượng tiến về0khi ntăng vô hạn

Bổ đề số nguyên tố tương đương với biểu thức sau: lim

x!1

#.x/

x Dxlim!1

(118)

trong đó# tương ứng hàm Chebyshev thứ thứ hai

Hàm Chebyshev thứ nhất#.x/hay.x/, cho công thức: #.x/D X

p6x

log.p/

với tổng phủ tất số nguyên tốpnhở bằngx

Hàm Chebyshev thứ hai, x/được định nghĩa tương tự, với tổng trải dài luỹ thừa số

nguyên tố không vượt quáx: x/D X

pk6x

log.p/DX n6x

ƒ.n/DX p6x

blogpxclogp

với ƒ hàm von Mangoldt Các hàm Chebyshev, đặc biệt hàm Chebyshev thứ hai, thường

được dùng chứng minh liên quan đến số nguyên tố

Bây giờ, xét quan hệ hai hàm số Chebyshev, hàm Chebyshev với hàm Hàm số Chebyshev thứ hai liên hệ đến hàm số thứ thơng qua việc viết dạng

x/ DX p6x

klog.p/

với k số nguyên chopk x pkC1 Một mối tương quan trực tiếp

giữa hai hàm thấy thơng qua cách biểu diễn sau:

x/ D

C1

X

nD1

#x1n

Chú ý tổng có số hữu hạng hạng tử khơng bị tiêu biến,

#xn1

D0vớin > log2xD

log.x/

log.2/

Hàm Chebyshev thứ hai log bội chung nhỏ số nguyên từ1đếnn: lcm.1; 2; : : : ; n/De n/

Đến đây, suy quan hệ sử dụng lời giải Bài toán 11 (IMC 2015, Day 2, Problem 7) Tính:

lim

A!C1

1 A

Z A

1

A1xdx:

(119)

Lời giải 1.. Chúng ta chứng minh rằng: lim A!C1 A Z A

A1xdx D1: VớiAD1, hàm lấy tích phân lớn hơn1 Khi đó,

1 a

Z A

1

A1x >

A

Z A

1

1dx D

A.A 1/D1 A:

Để tìm chặn chặt, cố định hai số thực,ı > 0vàK > 0, chia

khoảng thành phần điểm1CıvàKlogA Để ý vớiAđủ lớn (ví dụ,A > A0.ı; K/

với sốA0.ı; K/ > 1nào đó), có1Cı < KlogA < A VớiA > 1, hàm lấy tích

phân giảm, nên ước lượng giá trị điểm biên khoảng:

1 A

Z A

1

A1xdx D

A

Z 1Cı

1 C

Z KlogA

1Cı C

Z A

KlogA

!

<

D

A

ıAC.KlogA ı/A1C1ı C.A KlogA/A KlogA

< A

ıACKA1C1ı logACAA KlogA

D DıCKA 1Cıı logACe

1 K

Vì vậy, vớiA > A0.ı; K/, có:

1 A < A Z A

A1xdx < ıCKA ı

1Cı logACe K

Lấy giới hạnA! C1, có:

16 lim

A!C1inf

1 A

Z A

1

Ax1dx6 lim

A!C1sup

1 A

Z A

1

Ax1 6ıCe K:

Bây giờ, từı! C0vàK ! 1, có:

16 lim

A!C1inf

1 A

Z A

1

Ax1dx6 lim

A!C1sup

1 A

Z A

1

Ax1 61; Khi đó,

lim

A!C1inf

1 A

Z A

1

Ax1dxD lim

A!C1sup

1 A

Z A

1

Ax1 D1 kết luận

lim A!C1 A Z A

(120)

Lời giải 2.. Một phương án tiếp cận khác hữu ích cho tốn ngun tắc L’Hospital Có nhiều cách giải sử dụng nguyên tắc này, tác giả giới thiệu cách sử dụng đơn giản trực quan nhất, bao gồm phép biến đổi tuý

Chúng ta có:

lim A!1 A Z A

A1xdx D lim

A!1A

1 A C

Z A

1

A1x

x dxD

D1C lim

A!1

1 A

Z A

1

Ax1

x dxD

D1C lim

A!1

AA1

A C

Z A

1

Ax1

x2 dx D

D1C lim

A!1

1 A

Z A

1

Ax1

x2 dxD

D1C lim

A!1

1 A

A AA1 lnA D

D1

Lời giải 3. Đặtf x/DAx1, có, với số nguyên dươngn:

f nC1/ <

Z nC1

n

f x/dx < f n/

Khi đó,

PŒA

iD2f i /

ŒAC1 < ŒAC1

Z ŒA

1

Ax1 dx <

A

Z A

1

Ax1 dx <

ŒA

Z ŒAC1

1

Ax1 dx <

PŒA

iD1f i /

ŒA

Áp dụng Định lý kẹp Shtolz - Chezaro, có: lim

A!C1

PŒA

iD1f i /

ŒA DA!C1lim

f ŒAC1/ ŒAC1 ŒA D1

lim

A t oC1

PŒA

iD1f i /

ŒA DA!C1lim

f ŒAC1/ ŒAC1 ŒA D1

Theo nguyên lý kẹp, có: lim A!C1 A Z A

(121)

2.3 Số học

Bài toán 12 (IMC 2012, Day 2, Problem 3) Số số nguyên dươngnsao chonŠC1chia hết 2012n/Šlà hữu hạn hay vô hạn ?

(Đề xuất Fedor Petrov, Đại học bang St Petersburg) Lời giải 1. Xét số thực dươngnsao chonŠC1j.2012n/Š

Chúng ta biết điều hiển nhiên với số nguyên không âm tuỳ ý a1; : : : ; ak, a1 C

: : :Cak/Šchia hết choa1Š : : : akŠ (Số dãy số có chứaa1 số1, ,ak sốklà a1aC:::Cak/Š

1Š:::akŠ : Cụ thể hơn, đến đây, có.nŠ/2012chia hết.2012n/Š:

Bởi nŠC1ngun tố với.nŠ/2012, tích chúng.nŠC1/.nŠ/2012 chia

hết.2012n/Š, đó:

.nŠC1/.nŠ/2012 6.2012n/Š

Sử dụng bất đẳng thức tiếng sau:

n

C1

e

n

< nŠ6nn;

chúng ta có

n

e

2013n

< nŠ/2013 < nŠC1/.nŠ/2012 6.2012n/Š < 2012n/2012n n < 20122012e2013

Như vậy, có hữu hạn số tự nhiênnthoả mãn điều kiện đề

Nhận xét Thay sử dụng ước lượng nCe1n

< nŠ, sử dụng hệ số hoán vị có lặp: x1C: : :Cxl/N D

X

k1C:::CklDN

N Š k1Š : : : klŠ

xk1

1 : : : x k1

l

Áp dụng công thức vớiN D2012n; l D2012vàx1D: : :Dxl D1, có:

.2012n/Š

.nŠ/2012 < 1C1C: : :C1/ 2012n

D20122012n;

nŠ < nŠC1.2012n/Š

.nŠ/2012 < 2012 2012n

:

(122)

Lời giải 2. Giả sử rằngn > 2012là số nguyên chonŠC1j.2012n/Š Để ý tất

các ước nguyên tố củanŠC1đều lớn hơnn, tất ước nguyên tố của.2012n/Šđều nhỏ

hơn2012n:

Xét số nguyên tốp chon < p < 2012n Trong số1; 2; : : : ; 2012n, cóh2012np i < 2012số chia hết chop; vìp2 > n2 > 2012n, khơng có số chia hết chop2 Chính vậy,

luỹ thừa củaptrong khai triển thừa số nguyên tố của.2012n/Šcao là2011 Khi đó, nŠC1Dgcd.nŠC1; 2012n/Š/ < Y

n<p<2012p

p2011

Áp dụng bất đẳng thứcQ

p6X p <

X, có:

nŠ < Y

n<p<2012p

p2011< Y

p<2012n

p

!2011

< 42012n/2011 D.420122011/n

Một lần nữa, lại có hàm luỹ thừa vế trái cấp số nhân vế phải

Lời giải 3. Vì.nŠ/2012j.2012n/Š, gcd.nŠ; nŠC1/D1, có.nŠ/2013C.nŠ/2012j.2012n/Š

Ngồi ra:

.nC1/2012.nC1/ŠC1

nŠC1 > 2012nC2012/.2012nC2011/ : : : 2012nC1/

,f n/D 2012n/Š

.nŠ/2013C.nŠ/2012 >

.2012nC2012/Š

nC1/Š/2013C nC1/Š/2012 Df nC1/

Chúng ta thấy nC1/ŠC1

nŠC1 > n,

.2012nC2012/.2012nC2011/ : : : 2012nC1/ < 2012nC2012/2012 D20122012.nC1/2012:

Chính thế, bất đẳng thức với giá trịnđủ lớn Chính thế, đến giá trị nđủ lớn, hàm sốf n/sẽ hàm giảm ngặt Giả sử tồn vô hạn số nguyênnsao cho f n/là số nguyên, gọi chúng làa1 < a2 < : : :, đó, đến lúc đó, dãy số

nguyên trở thành dãy hằng, tức là9k; f ak/D f akC1/D : : : Vìf hàm giảm,

chúng ta cóf ak/Df akC1/DfakC2 D: : : Chính vậy, với mọit > ak, có:

.2012t /Š

.t Š/2013C.t Š/2012 D

.2012t C2012/Š t C1/Š/2013C t C1/Š/2012

hay

.t C1/2012.t C1/ŠC1

t ŠC1 D.2012t C2012/.2012t C2011/ : : : 2012tC1/

Áp dụng lý luận tương tự bên trên, thấy rằng, đẳng thức khơng cịn với số t đủ lớn (vì đó, vế trái tăng nhanh lớn vế phải) Chúng ta có điều phải

(123)

Bài toán 13 (IMC 2013, Day 2, Problem 4) Có tồn hay khơng tập vô hạnM gồm

những số nguyên dương cho với hai sốa; b M bất kì,a < b, có tổngaCb

một số phi-bình-phương ?

(Định nghĩa: Một số nguyên dương xem phi-bình-phương khơng có số chính phương lớn hơn1chia hết nó.)

(Đề xuất Fedor Petrov, Đại học Bang St Petersburg) Lời giải. Câu trả lời có Chúng ta xây dựng dãy vô hạn1 D n1 < D n2 < n3 < : : :

sao cho ni Cnj số phi-bình-phương với mọii < j Giả sử có số

số nk > : : : < nk.k > 2/thoả mãn điều kiện đó, tìm số hạngnkC1 thích

hợp để làm số hạng dãy

Chúng ta chọnnkC1dưới dạngnkC1 D1CM x, vớiM D n1C: : :CnkC2k/Š/2và

số nguyên dương x Vớii D 1; 2; : : : ; k, có ni CnkC1 D 1CM x Cni D

.1 C ni/mi, với mi M nguyên tố nhau, nên số phương chia hết

1CM xCni nguyên tố vớiM

Để tìm sốxthích hợp, chọn sốN đủ lớn xét giá trịx D1; 2; : : : ; N

Nếu giá trị > x < N không phù hợp, điều có nghĩa tồn số > i > kvà số nguyên tốp chop2j1CM x Cn

i Vớip 2k, điều

không thể xảy vìpjM Hơn nữa, cịn cóp2 61CM xCni < M.N C1/, nên

2k < p < pM.N C1/

Vớii vàp cố định, giá trị củaxsao chop2j1CM xCni cấp số cộng với công sai

p2 Chính vậy, có tối đa pN2C1các giá trị Tổng kết lại, số sốxkhông phù hợp

k

X

iD1

X

2k<p<pM.NC1/

N

p2 C1

< k

0

@N X

p>2k

1 p2 C

X

p<pM.NC1

1

1

A

< kN X

p>2k

1

p 1 p

CkpM.N C1/ < N Ck

p

M.N C1/

NếuN đủ lớn, số nhỏ hơnN; đó, chắn tồn sốxthích hợp

Nhận xét Đây toán số học đẹp kì thi Tốn học năm gần đấy, tính kì thi quốc tế cho học sinh sinh viên Điểm mấu chốt nếua1; : : : ; ak số nguyên dương choa1; : : : ; ak bao phủ hết

số dư phép chia chop2, vớip số ngun tố, tìm vơ hạn sốn >

sao chonCai phi-bình-phương với mọii:

Nhận xét cho phép xây dựng dãy a1; a2; : : : bầng cách xây dựng cách thêm

một phần tử từ dãy phần tử có sẵn Chú ý thêm rằng, nếua1; a2; : : : ; ak không

thể bao phủ hết số dư phép chia chop2 với p, thêm phần

tử akC1 cho akC1 Ca1; : : : ; akC1 Cak số phi-bình-phương, thìa1; : : : ; akC1

(124)

Bài toán 14 (IMC 2016, Day 2, Problem 3) Chonlà số nguyên dương, gọiZnlà vành

các nhóm số nguyên modulon Giả sử tồn hàm sốf WZn!Zntho ba tớnh

cht sau: (i) f x/Ôx

(ii) f f x//Dx

(iii) f f f xC1/C1/C1/Dxvới mọix2 Zn

Chứng minh rằngn2.mod4/:

Lời giải. Từ giả thiết (ii), thấy rằngf tồn ánh, vậyf phép hốn

vị phần tử trongZn, bậc hoán vị cao là2 Chính vậy, phép hốn vị

f tích phép chuyển vị có dạng.x; f x//:

Từ giả thiết (i), có phép hốn vị khơng có điểm cố định cả, tức không tồn phần tửx chof x/ D x, nênnphải chẵn, số phép chuyển vị

đúng n 2:

Xét phép hoán vị g.x/D f xC1/ Nếug hoán vị lẻ, gıgıgcũng hoán vị lẻ

Tuy nhiên, giả thiết (iii.) xác nhận rằnggıgıg hoán vị chẵn, hốn vị

biếnxthành Vì vậy,gkhơng thể hốn vị lẻ;g phải hốn vị chẵn Phép

hốn vị vịng quanhh.x/Dx 1có bậcn, vớinlà số chẵn, nênhlà hốn vị lẻ Khi

đó,f x/Dgıhsẽ hốn vị chẵn Vìf tích củan2 phép chuyển vị, điều cho chúng

ta biết n

2 lẻ, hayn2.mod4/

Nhận xét Ở trên, sử dụng khái niệm hoán vị chẵn hoán vị lẻ Để làm rõ hai thuật ngữ này, đầu tiên, định nghĩa lại nghịch

Bổ đề (Nghịch thế) Với phép hoán vị, ta gọi xi; xj/ là nghịch của nếu

xi < xj nhưng xi/ > xj/

Đến đây, định nghĩa hốn vị chẵn lẻ, dựa vào số lượng phép nghịch phép hốn vị

Bổ đề (Tính chẵn lẻ hốn vị) Tính chẵn lẻ hốn vị là tính chẵn lẻ số

nghịch của.

Nếu số cặp nghịch phép hốn vị là số chẵn thì được gọi phép

hoán vị chẵn

Nếu số cặp nghịch phép hoán vị là số lẻ thì được gọi phép

(125)

Với mỗin2.mod4/, xây dựng hàm số với tính chất thoả mãn

3 tính chất đề

Với n D 2, chọnf vớif 1/ D 2; f 2/ D Ở đây, xây dựng cách

xây dựng hàmf vớin>6:

Đặt n D 4k C2, xét đa giác với kC2 cạnh, chia thànhk tam giác vớik

đường chéo Vẽ hình trịn cho cắt cạnh mội đường chéo hai lần; tổng cộng có4k C2giao điểm Đánh số giao điểm theo chiều kim đồng hồ xung quanh

hình trịn Với cạnh đường chéo, có giao điểm; xây dựng f

biến điểm thành điểm kia, ngược lại

Hàmf vừa xây dựng rõ ràng thoả mãn tính chất (i) (ii) Với mỗixchúng

ta có, hoặcf xC1/Dx, cộng vào lấy hàmf lần dọc theo cạnh tam giác,

nên hàm số thoả mãn tính chất (iii) đề

Sau biểu diễn hình học hàmf vừa xây dựng

Bài toán 15 Choklà số nguyên dương Với số nguyên không âmn, gọif n/là số bộ x1; x2; : : : ; xn/2Zk thoả mãn bất đẳng thứcjx1j C jx2j C: : :C jxkj6n Chứng minh rng

vi minÔ1, chỳng ta cúf n 1/f nC1/6 f n//2:

(Đề xuất Esteban Arreaga, renan Finder José Madrid, IMPA, Rio de Janeiro) Lời giải 1. Chúng ta chứng minh quy nạp theok NếukD1, cóf n/D2nC1

và phát biểu đề đúng, theo bất đẳng thức AM - GM

Giả sử rằngk>2và phát biểu đề vớik Gọig.m/là số bộ.x1; : : : ; xk 1/thoả

mãn bất đẳng thứcjx1j C jx2j C: : :C jxk 1j6m; theo giả thiết quy nạp,g.m 1/g.mC1/6

.g.m//2; viết giả thiết dạng g.0/

g.1/ g.1/ g.2/

g.2/ g.3/ 6: : :

Với số nguyênc, bất đẳng thứcjx1j C: : :C jXk 1j jnjcóg.n jcj/bộ nghiệm

ngun Chính vậy, có quan hệ sau:

f n/D n

X

cD n

(126)

Điều dẫn đến

f n 1/ f n/ D

g.n 1/C2g.n 2/C: : :C2g.0/ g.n/C2g.n 1/C: : :C2g.1/C2g.0/

6 g.n/Cg.n 1/C.g.n 1/C: : :C2g.0/C2:0/

g.nC1/C2g.n/C.g.n/C: : :C2g.1/C2g.0// D

f n/ f nC1/

Bất đẳng thức tương đương với điều phải chứng minh

Lời giải 2. Đầu tiên, tính hàm sinh hàm sốf n/:

1

X

nD0

D X

.x1;x2;:::;xk/2Zk

X

cD0

qjx1jCjx2jC:::CjxkjCc D X

x2Z

qjxj

!k

1 q D

.1Cq/k q/kC1

Với mỗiaD0; 1; : : :gọiga.n/.nD0; 1; 2; : : :/là hệ số khai triển sau:

.1Cq/a

.1 q/kC1 D

1

X

nD0

ga.n/qn

Rõ ràng rằng,gaC1.n/ Dga.n/Cga.n 1/.n >1/; ga.0/ D1 Chúng ta gọi dãy số

nguyên dươngg.0/; g.1/; g.2/; : : :là "tốt" g.n 1/g.n/ nD1; 2; : : :/là dãy tăng Chúng ta

kiểm tra dãyg0là dãy "tốt":

g0.n/D

kCn k

!

;g0.n 1/ g0.n/

D n

kCn

Nếu g dãy tốt, dãy g0 định nghĩa g0.0/ D g.0/; g0.n/ D g.n/C

g.n 1/.n>1/cũng dãy tốt: g0.n 1/

g0.n/ D

g.n 1/Cg.n 2/ g.n/Cg.n 1/ D

1Cg.n 2/g.n 1/

1Cg.n 1/g.n/

nếu quy địnhg 1/D Như vậy, thấy dãyg1; g2; : : : ; gk Df

dãy tốt Chúng ta có bất đẳng thức cần phải chứng minh

Nhận xét Ở Lời giải 2, có sử dụng khái niệm mới, hàm sinh, hay cụ thể hàm sinh thường Hàm sinh sáng tạo mang nhiều ứng dụng toán rời rạc Sử dụng hàm sinh, chuyển toán dãy số thành toán hàm số, xét toán quan điểm tuý hàm số Khi áp dụng hàm sinh, làm cho tốn gọn gàng dễ giải

Vì giới hạn viết, tác giả không bàn sâu ứng dụng hàm sinh, mà giới thiệu qua định nghĩa vài ứng dụng hàm Các bạn tìm đọc thêm hàm số thú vị phần trích dẫn

Định nghĩa Hàm sinh thường (Ordinary Generating Function) dãy số vơ hạn.an/n>0

chuỗi luỹ thừa hình thức:

(127)

Ta gọi hàm sinh chuỗi luỹ thừa hình thức thơng thường, ta coix kí hiệu

thay thay số Chỉ vài trường hợp, ta choxnhận giá trị thực; vậy, chúng

ta gần không để ý đến hội tụ chuỗi Có nhiều dạng hàm sinh khác nhau, đây, nêu định nghĩa sử dụng hàm sinh thường

Ý tưởng sử dụng hàm sinh thường đơn giản sau: Giả sử ta phải tính dãy số

S.n/phụ thuộc vàon, vớinchạy từ0đến vô cùng, cho quy tắc truy hồi tổ

hợp Khi đó, ta viết

F x/DX n

S.n/xn

Thay tínhS.n/với giá trị củan, ta tính xem hàmF x/là hàm Nếu tính đượcF x/,

ta cần lấy đạo hàm điểm0, từ suy sốS.n/:

Trong phương pháp này,F x/chính hàm sinh thường

Ngay sau ví dụ áp dụng hàm sinh thường

Bài toán 16 (IMC 2016, Day 1, Problem 5) GọiSnlà tập pháp hoán vị dãy.1; 2; : : : ; n/.

Với phép hoán vị D.1; 2; : : : ; n/2Sn, gọi inv./là số cặp16i < j 6n

vớii > j; tức là, số nghịch Gọif n/là số phép hoán vị 2Snsao cho inv./

chia hết chonC1:

Chứng minh có vơ hạn số ngun tố p sao cho f p 1/ > p 1/Šp , có vơ hạn số số

nguyên tốp sao chof p 1/ < p 1/Šp :

(Đề xuất Fedor Petrov, Đại học Bang St Petersburg) Lời giải. Chúng ta sử dụng công thức hàm sinh sau:

X

2Sn

xinv./ D1.1Cx/.1CxCx2/: : :.1CxC: : :Cxn 1/

Cơng thức hàm sinh chứng minh quy nạp theo n Trường hợp n D 1;

khá rõ ràng Từ hoán vị 1; 2; : : : ; n 1/, có hốn vị 1; 2; : : : ; n/

bằng cách thêm phần tửnvàonvị trí trước, giữa, sau số1; 2; : : : ; n 1; số nghịch

sẽ tăng tương ứng 1; n 2; : : : ; 1; 0:

Bây giờ, đặt

Gn.x/D

X

2Sn

xinv./

và đặt " D en2 iC1 Tổng hệ số số hạng có số mũ chia hết chonC1 viết thành tổng lượng giác sau:

f n/D

nC1

n

X

kD0

Gn."k/D

nŠ nC1 C

1 nC1

n

X

kD1

(128)

Ở đây, ý đến dấu

f n/ nŠ nC1 D

k D1 n 1Gn."

k/

vớinDp 1, đóplà số nguyên lẻ, đủ lớn

Với mỗikcố định,16k6p 1, có: Gp 1."k/D

p

Y

jD1

.1C"kC"2kC C".j 1/k/D p

Y

jD1

1 "j k "k D

.1 "k/.1 "2k/ ".p 1/k/ "k/p

Chú ý rằng, hạng tử tử số là.1 "/; "2/; : : : ; "p 1/, có bậc chúng

thay đổi Vì thế, sử dụng đẳng thức.z "/.z "k/ : : : z "p 1/D1CzC: : :Czp 1, Gp 1."k/D

p

.1 "k/p D

p

1 e2k ip

p

Khi đó,f p 1/ p 1/Šp có dấu với:

p

X

kD1

1 e2k ip

1 p

D p

X

k

ek.1pp/ i

2isink p

1 p

D221 p 1/p21 p

2

X

kD1

cosk.p 1/

p

sink

p

1 p

Với số nguyên tốplớn, hạng tử vớik D1tăng theo luỹ thừa nhanh tất

hạng tử khác, nên hạng tử định dấu tổng Để ý p 1/

p hội tụ

Chính thế, tổng dương nếup 1lẻ âm nếup 1chẵn Chính vậy, Với số

nguyên đủ lớn,f p 1/ n 1/Š

p dương nếup 3.mod4/và âm nếup 1.mod4/

2.4 Đại số

Bài toán 17 (IMC 2017, Problem 6, Day 2) Chop.x/là đa thức với hệ số thực khác đa

thức Với số nguyên dươngn, đặt

qn.x/ D.xC1/np.x/Cxnp.xC1/

Chứng minh có hữu hạn sốnsao cho tất nghiệm củaqn.x/là thực.

(129)

Bổ đề Nếuf x/DamxmC: : :Ca1xCa0l a thc viamÔ0, v tt c cỏc nghiệm của

f đều số thực, thì

a2m 2amam >0

Đặt nghiệm củaf làw1; w2; : : : ; wn Theo công thức Viète, có: m

X

iD1

wi D

am

am

;X

i <j

wiwj D

am

am

06

m

X

iD0

wi2 D m

X

iD0

wi

!2

2X

i <j

wiwj D

a

m

am

2

2am am

D a

2

m 2amam

a2 m

Khi áp dụng bổ đề, quan tâm đến tính chất ba hệ số hạng tử trongq.x/có bậc cao nhất, mà không cần phải quan tâm đến hạng tử lại Đặt p.x/ Daxk Cbxk 1Ccxk 2C: : :vàqn.x/ D AnxCk CBnxnCk 1CCnxnCk 2C: : :;

đó:

qn.x/D.xC1/np.x/Cxnp.xC1/D D

xnCnxn 1C n.nC1/

2 x

n C: : :

.axkCbxk 1Ccxk 2C: : :/

Cxn

a

xkCkxk 1C k.k 1/

2 x

k

Cbxk 1C.k 1/xk 2C: : :Ccxk

D2axnCkC nCk/aC2b/xnCk

C

n.n 1/Ck.k 1/

2 aC.nCk 1/bC2c

xnCk 2C: : : ;

Khi đó,

AnD2a; BnD.nCk/aC2b; CnD

n.n 1/Ck.k 1/

2 aC.nCk 1/bC2c

Khin! C1,

Bn2 2AnCn D.naCO.1//2 22a

n2a

2 CO.n/

D an2CO.n/!

Lúc này,Bn2 2AnCnsẽ âm, tức làqnsẽ khơng thể có nghiệm thực

Chúng ta giải tốn cách xét hai hệ số hai hạng tử có bậc cao nhất, đồng thời áp dụng Bất đẳng thức AM - GM sau:

Lời giải phụ. Đặtp.x/ D xk Caxk C: : :và đặtM > 0sao chop.x/ > 08x > M Khi

đó,

(130)

Nếu tất nghiệm qn.x/đều số thực, ta đặt chúng x1; x2; : : : ; xnCk Khi đó,

theo Định lý Viète, có:

a1C: : :CanCk D

2aCnCk D

aC nCk

2

Theo bất đẳng thức AM - GM, lại có:

.M C1/np.M / < qn.M /D2.M a1/.M a2/ : : : M anCk/

62

.n

Ck/M a1Ca2C: : :CanCk/

nCk

nCk

D2

M C

2 C a nCk

nCk

Chính vậy,

jP M /j62 M C

1

2 C

a nCk

nCk

.M C1/n

Điều vơ lý, khintiến đếnC1, vế phải tiến về0:

Chúng ta kết luận qn.x/ khơng thể có tất nghiệm thực với vô hạn số nguyên

dươngn

Bài toán 18 (IMC 2015, Day 1, Problem 2) Với số nguyên dươngn, đặtf n/là số sau

khi viếtnthành hệ nhị phân thay số0bằng số1và ngược lại Ví dụ,nD23sẽ trở thành 10111trong hệ nhị phân, thếf n/ sẽ là1000trong hệ nhị phần, đó f 23/ D 8 Chứng

minh rằng:

n

X

kD1

f k/6 n

2

4

Đẳng thức xảy nào?

(Đề xuất Stephan Wagner, Đại học Stellenbosch) Lời giải. Nếur vàklà số nguyên dương cho2r 6k 62r,ksẽ cór chữ số

(131)

Giả sử rằng2s 16n62s Khi đó: n.nC1/

2 C

n

X

kD1

f k/D n

X

kD1

.kCf k//

D s

X

rD1

X

2r 16k<2r

.kCf k//C X 2s 16k6n

.kCf k// D

D s

X

rD1

sr 1.2r 1/C.n 2s 1C/.2s 1/D

D s

X

rD1

22r

s

X

rD1

2r 1C.n 2s 1C1/.2s 1/D

D

3.4

s 1/ .2s 1/

C.2s 1/n 22s 1C32s 1D

D.2s 1/n

34

s

C2s

3:

n2

n

X

kD1

f k/D n

4

.2s 1/n 34

s

C2s

3

n.nC1/ D D 4n

2s

nC

34

s 2s

C D D n

s 2

3 n

2sC1

:

Để ý hiệu hai nhân tử cuối nhỏ 1, số chúng số nguyên:

2sC1 2

3 số nguyên s chẵn,

2sC1 4

3 số ngun s lẻ Chính thế, số

chúng phải 0, dẫn đến tích bằng0, hai nhân từ đểu phải dấu, dẫn đến tích

ln ln dương Điều giải toán cho thấy đẳng thức xảy khinD 2sC31

(nếuslẻ) hoặcnD 2sC31 (nếus chẵn)

3 Các tập tự luyện

3.1 Giải tích

Bài tập (IMC 2016, Day 1, Problem 1) Chof WŒa; b !Rlà hàm liên tục có đạo

hàm trênŒa; b Giả sử rằngf có vơ hạn nghiệm, khơng tồn số thựcx a; b/

sao chof x/Df0.x/D0

1 Chứng minh rằngf a/f b/ D0

(132)

(Đề xuất Alexandr Bolbot, Đại học Bang Novosibirsk) Bài tập (IMC 2012, Day 2, Problem 2) Định nghĩa dãy số an/ tổng quát sau, với

a0 D1; a1 D 12,

anC1D

na2n 1C.nC1/an

Chứng minh dãyP1

kD0 akC1

ak hội tụ tìm giới hạn

(Đề xuất Christophe Debry, KU Leuven, Bỉ) Bài tập (IMC 2016, Day 2, Problem 2) Hôm nay, Giáo sư Ivan lại thích hàm số liên tụcf WŒ0; 1 !Rthoả mãn điều kiệnf x/Cf y/ >jx yjvới cặpx; y Œ0; 1 Hãy

tìm Giá trị nhỏ tích phânR1

0 f hàm thoả mãn

(Đề xuất Fedor Petrov, Đại học Bang St Petersburg) Bài tập (IMC 2017, Problem 2, Day 1) Chof W R ! 0;1/là hàm có đạo hàm,

giả sử tồn sốL > 0sao cho

jf0.x/ f0.y/j6Ljx yj

với mọix; y Chứng minh rằng:

.f0.x//2 < 2Lf x/

đúng với mọix

(Đề xuất Jan Sustek, Đại học Ostrava) Bài tập (IMC 2017, Day 2, Problem 6) Chof WŒ0I C1/!Rlà hàm liên tục cho

limx!C1f x/DLtồn (nó vô hạn hữu hạn) Chứng minh rằng: lim

n!C1

Z

0

f nx/dxDL

(Đề xuất Alexandr Bolbot, Đại học bang Novosibirsk) Bài tập (IMC 2017, Day 2, Problem 9) Định nghĩa dãyf1; f2; : : :WŒ0; 1/!Rnhững hàm

liên tục có đạo hàm cơng thức truy hồi sau:

f 1/D1Ifn0C1DfnfnC1trên.0; 1/; vàfnC1.0/D1

Chứng minh limn!1fn.x/ tồn với mỗix Œ0; 1/và xác định hàm giới hạn với

giá trịx

(Đề xuất Tomas Barta, Đại học Charles, Prague) Bài tập Xét dãy số sau:

.an/1nD1 D.1; 1; 2; 1; 2; 3; 1; 2; 3; 4; 1; 2; 3; 4; 5; 1; : : :/

Tìm tất cặp số thực dương.˛; ˇ/sao cho limn!1

Pn

kD1ak

n˛ Dˇ

(133)

3.2 Số học

Bài tập (IMC 2015, Problem 3, Day 1) ChoF 0/D0; F 1/ D 32, vàF n/D

2F n 1/

F n 2/vớin>2 Xác định xem tổngP1

nD0

F 2n/ có phải số hữu tỉ hay không (Đề xuất Gerhard Woeginger, Học viện Công nghệ Eindhoven) Bài tập (IMC 2017, Problem 3, Day 1) Với số nguyên dương m, đặtP n/là tích

các ước dương củam(ví dụ,P 6/D36) Với số nguyên dươngn, định nghĩa

dãy số sau:

a1.n/ Dn; akC1.n/ DP ak.n//.k D1; 2; : : : ; 2016/

Hãy xét xem phát biểu sau hay sai: Với tập S f1; 2; : : : ; 2017g, tồn số

nguyên dươngnsao cho điều kiện sau thoả mãn:

Với mỗiksao cho1>k>2017, sốak.n/là số phương khik 2S:

(Đề xuất Matko Ljulj, Đại học Zagreb) Bài tập 10 (IMC 2014, day 1, Problem 4) Chon > 6là số hoàn hảo, đặtnDpe1

1 : : : p ek

k

là phép phân tích thừa số nguyên tố với1 < p1 < : : : < pk Chứng minh rằnge1 số

chẵn Biết rằng, sốnđược gọi hoàn hảo nếus.n/D2n, vớis.n/là tổng ước củan

(Đề xuất Javier Rodrigo, Universidad Pontificia Comillas) 3.3 Đại số

Bài tập 11 (IMC 2017, Day 1, Problem 4) Cón cư dân thành phố, người

trong số họ có đúng1000người bạn (biết tình bạn ln ln đối xứng) Chứng minh

có thể chọn nhóm người S cho 2017n người S có hai người bạn

trongS:

(Đề xuất Rooholah Majdodin Fedor Petrov, Đại học Bang St Petersburg) Bài tập 12 (IMC 2012, Day 2, Problem 1) Xét đa thức

f x/Dx2012Ca2011x2011C: : :Ca1xCa0

Albert Einstein Homer Simpson chơi trò chơi sau Mỗi lượt, họ chọn hệ sốa0; : : : ; a2011 cho giá trị thực Albert nước Một giá trị

được gán cho hệ số đó, khơng thể thay đổi Trò chơi kết thúc sau tất hệ số có giá trị

Mục tiêu Homer làm chof x/chia hết cho đa thức cố địnhm.x/nào mục tiêu

của Albert ngăn điều xảy

Ai người có chiến thuật thắng nếum.x/ Dx 2012?

(134)

3.4 Bất đẳng thức

Bài tập 13 (IMC 2016, Day 2, Problem 1) Cho.x1; x2; : : :/là dãy số thực dương thoả

mãn điều kiện sau:

1

X

nD1

xn

2n D1

Chứng minh rằng:

1

X

kD1 k

X

nD1

xn

k 62

(Đề xuất Gerhard J Woeginger, Hà Lan) Bài tập 14 (IMC 2016, Day 1, Problem 3) Cho nlà số thực dương Ngoài ra,

đặta1; a2; : : : ; an vàb1; b2; : : : ; bn số thực choai Cbi > 0vớii D 1; 2; : : : ; n

Chứng minh

n

X

iD1

aibi bi2

ai Cbi

Pn

iD1ai P

n

iD1bi P

n iD1bi

2

Pn

iD1.ai Cbi/

(Đề xuất Daniel Strzelecki, Đại học Nicolas Copernicus Torun, Ba Lan)

4 Gợi ý cho tập

4.1 Giải tích Bài tập

1 Chọn dãy nghiệm hội tụ.zn/và đặtc D lim.zn/2Œa; b Theo tính liên tục

f, cóf c/D0 Chúng ta muốn chứng mih hoặcc Dahoặcc Db, f a/D0hoặcf b/D0, từ có điều phải chứng minh

2 Một ví dụ hàm số thoả mãn yêu cầu đề là:

f x/D

(

xsin x1

nếu0 < x 61 0nếuxD0

Bài tập Chứng minh đẳng thức sau:

kak D

akC1

ak

(135)

Đồng thời, chứng minh, quy nạp, rằngan< 2Cn với số dươngC Khi đó, kết luận

1

X

kD0

akC1

ak

D1

Bài tập Áp dụng điều kiện đề với06x 12; y DxC 12, có:

f x/Cf

xC

2

>

2

Bằng cách lấy tích phân,

Z

0

f x/dx D

Z 12

0

f x/Cf

1

2

dx>

Z 12

0

dxD

4

Đến đây, chứng minh thêm tồn hàm số f x/ cho R01f x/dx D 14

Chúng ta tiến hành xét hàm số sau đây:

f x/D

ˇ ˇ ˇ ˇ

x

ˇ ˇ ˇ ˇ

Bài tập Đầu tiên, biết rằngf lấy tích phân cục (tức lấy tích phân

trên tập compact thuộc miền xác định)

Với mỗixbất kì, đặtd D f0.x/, cần chứng minhf x/ > d

2

2L Xét trường hợp

như sau:

d D0, phát biểu đề hiển nhiên

d > 0, điều kiện đề cho biếtf0.x t />d Lt, đồng thời ước lượng

dương với mọit choo6t < Ld

d > 0, điều kiện đề cho biếtf0.x Ct />d CLt D jdj CLt, lặp lại

tương tự với trường hợpd >

Bài tập Cách đơn giản để giải tốn đặtF x/DRx

0 f sử dụng

nguyên lý L’Hospital

Bài tập Giải phương trình vi phân công thức truy hồi đề bài, được:

fnC1.x/De

Rx

0 fn.t /dt

Khi đó, dùng quy nạp, chứng minh với sốxcố định khoảngŒ0; 1/, fn.x/là dãy tăng và:

fn.x/6

1 x

Hơn nữa, sử dụng bổ đề sau:

exCx22 C:::C xn

(136)

với mỗix cố định khoảngŒ0; 1/, có:

1 x >fn.x/>

1 xn x

bằng cách sử dụng quy nạp Cuối cùng, kết luận lim

n!C1fn.x/D

1 x:

Nhận xét Lời giải phía khơng phải lời giải thức kì thi, cách với ý tưởng dơn giản sơ cấp để tiếp cận toán này, với kiến thức Giải tích t chương trình THPT Đồng thời, lời giải trên, sử dụng bổ đề.S 3/,

chúng ta tiến hành chứng minh bổ đề sau:

exCx22 C:::C xn

n DexC x2

2C:::CO.xnC1/

De log.1 x/CO.xnC1/

D

1 x CO.x

nC1

/

D1CxCx2C: : :CxnCO.xnC1/

Như vậy, hiệu vế trái vế phải dãy luỹ thừa với số mũ lớn bằngnC1,

nhưng tất hệ số vế trái (dãy luỹ thừa củax) dương Chúng ta có điều cần phải chứng

minh

Bài tập ĐặtN D nC21

(khi đó,aNnsẽ lần xuất số tự nhiênntrong dãy) xét giới hạn dãy sau:

bNn D

PNn

kD1ak

N˛ n

D

Pn

kD11C: : :Ck nC1

2

˛ D

1 6n

3 1

C 2n

1C 1n

1

˛

n2˛ 1C n

˛

Chúng ta thấy limn!1bNn dương hữu hạn khi˛ D

3

2 Trong trường

hợp đó, giới hạn dãy bằngˇ D

p

2

3 Vì vậy, có cặp.˛; ˇ/D

2; p

có khả đáp số cho tốn

Giả sửN nằm khoảngŒNnC1; NnC1, hayN D NnCmvới1 6m 6nC1, với

việc áp dụng Định lý kẹp, suy giới hạn dãy bN, đồng thời suy

được giá trị củaˇ:

4.2 Số học

Bài tập Đầu tiên, cần phải chứng minh rằngF n/D2n 2 n Sau đó, chúng ta

sẽ có hai cách tiếp cận khác với toán sau: Chứng minh đẳng thức sau:

1 F 2n/ D

1 22n

1

1 22nC1

(137)

2 Đầu tiên, chứng minh đượcF n/ D2n n Khi đó,

1

X

nD0

1 F 2n/ D

1

X

nD0

1 22n

2 2n D

X

nD0

1

X

kD0

1

2n.2kC1/

D

1

X

mD1

1

m

D1

Ở đây, thừa nhận số nguyên dươngmcó cách biểu diễn m D2n.2kC1/với số nguyên không âmnvàk)

Đến đây, kết luận dãy số hội tụ về1:

Bài tập Chúng ta chứng minh phát biểu đúng, với tập S f1; 2; : : : ; 2017g, tồn sốnthoả mãn Cụ thể hơn,ncó thể luỹ thừa của2,n D2w1 với số ngun khơng âmw1 Chúng ta sử dụng bổ đề sau để chứng minh:

Bổ đề Giả sử dãy số b1; b2; : : :/ c1; c2; : : :/ thoả mãn điều kiện sau:

bkC1 D

bk.bkC1/

2 , ckC1 D

ck.ckC1/

2 với k > 1, và c1 D b1 C2

m Khi đó, với mỗi

k D1; 2; ::; m, cóck bkC2m kC1.mod2m kC2/.

Một hệ theo sau bổ đề bk ck.mod2/ với k m bmC1

cmC1C1.mod2/

Bài tập 10 Chúng ta chứng minh phản chứng, bắt đầu việc giả sửe1 lẻ

Sử dụng công thức sau:

sn D k

Y

iD1

.1Cpi Cp2i C: : :Cp ei

i /D2nD2p

e1

1 : : : p ek

k

chúng ta chứng minh rằng6jn

Khi đó,n;n2;n3;n6 và1là ước phân biệt củan, vậy: s.n/>nC n

2 C n C

n

6 C1D2nC1 > 2n

mâu thuẫn với điều kiện đề

Nhận xét Chúng ta có bổ đề tiếng sau: Tất số hồn hảo có dạng

nD2p 1.2p 1/sao chopvà2p 1đều số nguyên tố vậy, nếue1lẻ thìkD2; p1 D

2; p2 D2p 1; e1 Dp 1vàe2 D1 Nếun > 6thìp > 2, đóp lẻ vàe1 Dp 1phải

số chẵn

4.3 Đại số

Bài tập 11 Đặtd D 1000và < p < Chọn ngẫu nhiên tập hợpS cho người

(138)

Khi đó, xác suất để người chọn vào tập S biết xác hai thành viên

trong tậpS là:

q D d

2

!

p3.1 p/d

Chọnp D

d C1 (bởi giá trị củap choq đạt giá trị lớn nhất, từ

chứng minh phát biểu đề Bài tập 12

Để ý Homer người cuối, nước cuối nước định Homer thắng khif 2012/D0

Định nghĩa đa thức sau:

g.y/Da0Ca2y Ca4y2C: : :Ca2010y1005Cy1006

h.y/ Da1Ca3yCa5y2C: : :Ca2011y1005

Khi đó,f x/Dg.x2/Ch.x2/x

Lúc này, Homer thắng g.y/ h.y/ chia hết cho y C1, hay g 1/ D

h 1/ D0

4.4 Bất đẳng thức

Bài tập 13 Bằng cách thay đổi tổng

X

kD1 k

X

nD1

xn

k D

X

16k6n

xn

k2 D

1

X

nD1

xn

1

X

kDn

1 k2

!

:

Đồng thời, tìm chặn tổng:

X

kDn

1 k2

1

X

kDn

1 k2

4

D

n 12

và cịn có:

1

X

kD1 k

X

nD1

xn

(139)

Tài liệu

[1] Trang web thức Kì thi:https://www.imc-math.org.uk/

[2] Diễn đàn toán học Art of Problem Solving:https://www.artofproblemsolving com

[3] Lê Nhật Hoàng, Trần Phan Quốc Bảo, Định lý Hall ứng dụng, https://www diendantoanhoc.net

[4] Trương Thị Nhung, Lăng Thuý Nga, Phạm Thị Lan Phương, Mai Thị Ngoan, Chuyên đề Đại số sơ cấp - Phương pháp sử dụng hàm sinh, Trường ĐH Sư phạm Hà Nội, 2010 [5] Ivan Martel,MAA102 - Analysis I, Bachelor 2018 - 2019, École Polytechnique

[6] Nguyễn Tiến Dũng, Phương pháp hàm sinh, https://www.zung.zetamu.net/ 2014/12/generating-function/

[7] Weisstein, Eric W., Prime Number Theorem, from MathWorld–A Wolfram Web Resource

(140)

MỘT SỐ BẤT ĐẲNG THỨC DIỆN TÍCH

TRONG TAM GIÁC

Trần Quang Hùng, Hà Nội

TÓM TẮT

Trong viết thiết lập bất đẳng thức diện tích tam giác Các tốn chủ yếu thuộc phần hệ thức lượng tam giác Tuy nhiên chúng tơi lại giải tốn dựa số kiến thức bất đẳng thức đại số đơn giản kiến thức hình học túy, phù hợp với chương trình hình học cấp THCS Việt Nam

Bài toán 1. Cho tam giácABC vàX, Y, Z điểm bât kỳ đoanBC, CA, AB Chứng

minh

1 SAY Z

+ SBZX

+ SCXY ≥

3 SXY Z

A

B C

Z

Y

X

Hình

Lời giải. Đặt BX BC =x,

CY CA =y,

AZ

AB =z, SABC =Sthì0< x, y, z <1 Ta có SAY Z

S =z(1−y), SBZX

S =x(1−z), SCXY

S =y(1−x)

(141)

1 SAY Z +

1 SBZX +

1 SCXY ≥

3 SXY Z

⇔ S z(1−y) +

S x(1−z) +

S y(1−x) ≥

3S

xyz+(1−x)(1−y)(1−z)

⇔ (1xy−y) + yz (1−z)+

zx (1−x) ≥

3 1+(1−x)(1xyz−y)(1−z)

Đặt 1−x

x =a >0, 1−y

y =b >0, 1−z

z =c > 0ta đưa chứng minh bất đẳng thức đại số

(a+ 1)b + (b+ 1)c +

1 (c+ 1)a ≥

3 +abc

Thật ta có

(1 +abc) b(1+a) + c(1+b)+ a(1+c) + = 1+abc+b+ab b+ab + 1+abc+c+bc c+bc + 1+abc+a+ca a+ca

= ab+b1+b +bc+c1+c +ca+a1+a +a(c+1)a+1 + b(a+1)b+1 +c(b+1)c+1

≥ 3

abc+ 3

abc≥6

Do (a+1)b + (b+1)c+ (c+1)a ≥

1+abc.Ta có điều phải chứng minh Nhận xét. Bất đẳng thức đại số

1 (a+ 1)b +

1 (b+ 1)c +

1 (c+ 1)a ≥

3 +abc

sử dụng toán bất đẳng thức đại số kinh điển thập niên trước Nó xuất thi Balkan 2006 tạp chí Crux, xem [1] Bất đẳng thức hình học tốn thể hình học đặc sắc bất đẳng thức đại số

Bài toán toán cực trị hình học đẹp mắt, ứng dụng bất đẳng thức diện tích

Bài tốn 2. Cho tam giácABC, giả sử có điểmX, Y, Z thuộc đoạnBC, CA, AB cho

các tam giácAY Z, BZX, CXY có diện tích Tìm vị tríX, Y, Z cho diện tích

tam giácXY Z bé

Lời giải. ĐặtSAY Z =SBZX =SCXY =SxthìSXY Z =SABC−3Sx Mặt khác theo trước

thì

1 SAY Z

+ SBZX

+ SCXY ≥

3 SXY Z

Nên Sx ≥

3

SXY Z suy raSXY Z ≥Sx =

SABC−SXY Z

3 đóSXY Z ≥

(142)

Nhận xét. Mặc dù tốn lời giải đơn giản sử dụng toán 1, rõ ràng toán cực trị hay nói cách khác tối ưu hình học thú vị Tác giả viết quan tâm tới lời giải cho toán mà khơng sử dụng tốn

Một hệ quan trọng khác toán toán xuất kỳ thi IMO năm 1966 đây, bạn làm luyện tập

Bài toán 3 (IMO 1966, P6 [5]). ChoABC tam giác LấyP, Q, R ba điểm

nằm cạnh BC, CA, AB tam giác Chứng minh

tam giácAQR,BRP,CP Qcó diện tích bé phần tư diện tích tam giác ABC

Bài tốn 4. Cho tam giácABCcó diện tíchSvà số thực dươngx, y, z ĐặtBC =a, CA= b, AB =c

a) Chứng minh rằngxa2+yb2+zc2 ≥4√xy+yz+zxS.

b) Chứng minh rằngpyz xa2+

q zx

y b2+ pxy

z c2 ≥4

x+y+zS

A

B C

H

Hình

Lời giải. a) Tam giácABC phải có hai góc nhọn Giả sử làB, C, vẽ đường cao AH

thìHnằm giữaB, C Ta có xa2 +yb2+zc2

=xa2+y(HA2+HB2) +z(HA2+HC2) =xa2+ (y+z)HA2+ HB2

1/y + HC2

1/z

≥xa2+ (y+z)HA2+(HB+HC)2 (y+z)/(yz)

=a2(x+ yz

(143)

≥2 xy+yz+zxy+z a2.(y+z)HA2 = 4√yz+zx+xyS

b) Ta sử dụng phép biếnx→pyzx, y →qzx y , z→

pxy

z thìxy+yz+zx→x+y+z

đó áp dụng câu a) thìpyz xa2+

q zx

y b2+ pxy

z c2 ≥4

x+y+zS

Nhận xét. Bài toán phần a) bất đẳng thức hình học kết hợp với đại số kinh điển hệ thức lượng tam giác Chúng ta giải pháp hình học, lời giải có ý nghĩa thực hành giảng dạy

Bài toán 5. Cho tam giácABCcó điểmX, Y, Zthuộc đoạnBC, CA, ABsao choSXY Z =

4SABC Đặt Sa = SAY Z, Sb = SBZX, Sc = SCXY, BC = a, CA = b, AB = c Tìm vị trí X, Y, Z để biểu thức

Saa2+Sbb2+Scc2

SaSbSc

đạt giá trị bé

Lời giải. Theo tốn Sa +

1 Sb +

1 Sc ≥

3 SXY Z =

12

SABC Áp dụng câu b) tốn ta có

Saa2√+Sbb2+Scc2

SaSbSc =

q Sa

SbSca

2+q Sb

ScSab

2+q Sc

SaSbc

2 ≥4q Sa +

1 Sb +

1

ScSABC ≥4

q 12

SABCSABC =

8√3SABC

Dấu xảy khiX, Y, Z trung điểmBC, CA, AB

Nhận xét. Bài toán sáng tạo nhỏ ghép nối kết toán toán Rõ ràng bất đẳng thức diện tích toán toán ghép nối lại với tạo ứng dụng thú vị

Bài tốn 6. Cho tam giácABCcó diện tíchSvà số thực dươngx, y, z ĐặtBC =a, CA= b, AB =c Chứng minh

yz x a

4+ zx y b

4+ xy z c

4 ≥ x+y+z 16S

2.

Lời giải. Áp dụng tốn câu b) ta có3(yzxa4+zx y b

4+xy z c

4)≥(pyz xa

2+qzx y b

2+pxy z c

2)2 ≥ 16(x+y+z)S2 Từ đó yz

xa 4+ zx

y b +xy

z c

4 ≥ x+y+z 16S

2.

Bài tốn 7. Cho tam giácABC có diện tíchSvàBC =a, CA=b, AB =c

a) Chứng minh rằnga4+b4+c4 ≥16S2.

b) Chứng minh rằngb2c2+c2a2+a2b2 ≥16S2.

c) Chứng minh rằngabc ≥

27

(144)

Lời giải. a) Áp dụng toán chox=y=zsuy raa4+b4 +c4 ≥16S2.

b) Áp dụng toán câu b) cho x = a4, y = b4, c = z4 ta có b2c2 + c2a2 + a2b2 ≥ 4√a4+b4 +c4S ≥16S2.

c) Áp dụng tốn câu a) khix=y=zthìa2+b2+c2 ≥4√3S Từ áp dụng tốn 6

chox=a2, y =b2, z=c2thì3a2b2c2 ≥ a2+b2+c2

3 16S2 ≥ 64S3

3 Từ đóabc≥

27

S3.

Bài toán 8. Cho tam giác ABC điểm P nằm tam giác P A, P B, P C cắt BC, CA, AB tạiD, E, F Hãy tìm giá trị lớn diện tích tam giácDEF

A B C P E F D Hình

Lời giải. ĐặtSP BC =Sa, SP CA =Sb, SP AB =Sc, SABC =S Ta dễ thấy AE

AC = Sc

Sc+Sa,

AF AB =

Sb

Sa+Sb Từ

SAEF S = AE AC AF AB =

Sc.Sb

(Sa+Sc)(Sa+Sb)

Tương tự SBF D

S =

Sa.Sc

(Sb+Sa)(Sb+Sc),

SCDE

S =

Sb.Sa

(Sc+Sb)(Sc+Sa)

Từ ta có SAEF+SBF D+SCDE

S =

Sc.Sb

(Sa+Sc)(Sa+Sb) +

Sa.Sc

(Sb+Sa)(Sb+Sc)+

Sb.Sa

(Sc+Sb)(Sc+Sa) ≥

3

MàSAEF +SBF D +SCDE = S−SDEF suy raSDEF ≤ 14S, dấu xảy khiP trọng

tâm tam giácABC

Bài tốn 9. Cho tam giácABCcó diện tíchSvà số thực dươngx, y, z ĐặtBC =a, CA= b, AB =c

a) Chứng minh

a2+b2+c2 ≥4√3S+ x+y+z(

x2−yz

x +

y2−zx y b

2+ z2−xy z c

2).

b) Chứng minh rằnga2+b2+c2 ≥4√3S+ (b−c)2+ (c−a)2+ (a−b)2.(Bất đẳng thức

(145)

Lời giải. a) Từ toán câu a), ta biến x → xyz(x+ y +z)− 2yz(x2 −yz), y → xyz(x+y+z)−2zx(y2−zx), z →xyz(x+y+z)−2xy(z2−xy) Ta thấyxy+yz+zx→ 3(x2y2z2)(x+y+z)2 Từ ta có

(xyz(x+y+z)−2yz(x2−yz))a2+ (xyz(x+y+z)−2zx(y2−zx))b2+ (xyz(x+y+z)− 2xy(z2−xy))c2 ≥4√3xyz(x+y+z)S.

Rút gọn bất đẳng thức tương đương

a2+b2+c2 ≥4√3S+ x+y+z(

x2−yz x a

2+ y2−zx y b

2+ z2−xy z c

2).

b) Chox=a, y=b, z =c, ýa3+b3+c3−3abc=

2(a+b+c)((b−c)

2+(c−a)2+(a−b)2).

Ta thu đượca2+b2+c2 ≥4√3S+ (b−c)2+ (c−a)2+ (a−b)2.

Nhận xét. Bài toán câu a) mở rộng cho toán câu b) Bài tốn câu b) có tên gọi bất đẳng thức Hadwiger-Finsler, bất đẳng thức vô quan trọng tiếng hệ thống bất đẳng thức hình học Bất đẳng thức có nhiều mở rộng, nhiên mở rộng theo cách câu a) mở rộng thú vị bất đẳng thức có tới sáu biến, xem [2,3]

Cuối ta tới ứng dụng hình học đẹp cho bất đẳng thức hình học xây dựng

Bài toán 10. Cho tam giácABC, dựng ngồi hình vngBCM N, CAP Q, ABRS Gọi SM cắtP N tạiX,P N cắtRQtạiY,RQcắtSM tạiZ Chứng minh

SXY Z ≤(2−

3)SABC

A

B C

N R

P S

M

Q

A'

B' C'

H K

X Y Z

(146)

Lời giải. ĐặtBC = a, CA =b, AB = c Gọi H, K hình chiếu AlênBN, CM Ta dễ

thấy tam giácBC0A0 BAN đồng dạng, tam giácCA0B0 vàCM Ađồng dạng tỷ số √1

2

do dó

SBC0A0+SCA0B0 = 12(SBAN +SCAM) = 12(12AH.BN +12AK.CM) = 14BC2

Chứng minh tương tựSCA0B0 +SAB0C0 = CA

2

4 , SAB0C0+SBC0A0 = AB2

4 Do ta thu

SAB0C0 +SBC0A0+SCA0B0 = AB

2+BC2+CA2

8

Theo toán câu a) khix = y = z thìAB2 +BC2 +CA2 ≥ 4√3S

ABC SAB0C0 +

SBC0A0+SCA0B0 ≥

3 SABC

Lại cóSA0B0C0 =SABC +SAB0C0 +SBC0A0 +SCA0B0 ≥(1 +

3

2 )SABC hay SABC

SA0B0C0 ≤

1 1+√23 = 4−2√3

Dễ thấy tam giác XY Z A0B0C0 có cạnh tương ứng song song nên đồng dạng Ta đặt

Y Z

B0C0 = CZX0A0 = AXY0B0 =kthì SSAXY Z0B0C0 =k

2 Ta có (1−k2)(S

A0B0C0) =SA0B0C0−SXY Z = (SA0B0X+SXB0Y)+(SB0C0Y+SY C0Z)+(SC0A0Z+SZA0X)

Chú ý rằngSY C0Z +SB0C0Y = 12d(C0, Y Z).Y Z+ 12d(Y, B0C0).B0C0 = 21d(A, B0C0).kB0C0+

1

2d(A, B0C0).B0C0 = (1 +k)SAB0C0 Tương tự

SA0B0X +SXB0Y = (1 +k)SCA0B0, SC0A0Z+SZA0X = (1 +k)SAB0C0 Từ

(1−k2)S

A0B0C0 = (1 +k)(SAB0C0 +SBC0A0 +SCA0B0) = (1 +k)(SA0B0C0 −SABC)

Hay (1−k)SA0B0C = SA0B0C0 −SABC suy SABC = kSA0B0C0 =

q SXY Z

SA0B0C0SA0B0C0

S2

ABC =SXY Z.SA20B0C0

Từ đóSXY Z =SABC.SSABC

A0B0C0 ≤(4−2

3)SABC Ta có điều phải chứng minh Nhận xét. Bất đẳng thức ứng dụng hình học đẹp mắt bất đẳng thức

a2+b2+c2 ≥4√3S

(147)

Lời cảm ơn. Tác giả viết chân thành cảm ơn bạn Lê Bích Ngọc, người học trị nữ xuất sắc tác giả khóa chun tốn K46 THPT chuyên KHTN Thời gian qua đi, đứng bục giảng, chứng kiến nhiều biến chuyển hình học sơ cấp Nếu tơi thấy có bạn học sinh dùng tốn với mục đích khác riêngBích Ngọcđã gợi nhắc cho tơi rằng, tơi cịn nhiều học trị thực có niềm say mê tốn hình học Nếu tốn khơng thể mãi, thời điểm, có lúc cháy với tốn, giây phút vơ đáng trân trọng

Tài liệu

[1] Balkan MO 2006, Problem 1; Crux 1998,

https://artofproblemsolving.com/community/c6h161059p1205

[2] Hojoo Lee, Tom Lovering, and Cosmin Pohoata,Infinity, The first edition (Oct 2008) on internet

[3] Claudi Alsina and Roger B Nelsen,Geometric Proofs of the Weitzenbăock and Hadwiger-Finsler Inequalities, Mathematics Magazine, Vol 81, No (Jun., 2008), pp 216-219 [4] IMO 1961, Problem 2,

https://artofproblemsolving.com/community/c6h14388p101840

[5] IMO 1966, Problem 6,

(148)

MỘT SỐ VẤN ĐỀ VỀ CÁC ĐƯỜNG TRÒN

MIXTILLINEAR VÀ THÉBAULT

Lê Xuân Hồng

TĨM TẮT

Sau số vấn đề phát triển cấu hình đường trịn Mixtillinear Thébault Đây cấu hình thú vị có nhiều ứng dụng giải tốn

Bài tốn 1(Lê Xn Hồng). Cho4ABCnội tiếp(O), ngoại tiếp(I)

X điểm đường thẳngBC Đường thẳng qua I vng gócIX cắt tiếp tuyến

song songBC của(I)tạiY

AY cắt(O)tạiZkhácA Chứng minh rằngXZđi qua tiếp điểm đường trònA−Mixtilinear

nội tiếp của4ABC với(O)

A

B C

I

D

E

F

X

D0 K L Y

Y0

Z T

O

R

G

H

Chứng minh. GọiRlà giao điểm củaAY,BC

D,E,F tiếp điểm của(I)vớiBC,CA,AB.D0đối xứngDquaI.

T tiếp điểm củaA−Mixtilinear nội tiếp của4ABC với(O)

(149)

Theo tính chất đường trịn mixtilinear, T A, T D đẳng giác góc ∠BT C Sử dụng

các cặp tam giác đồng dạng (4ILD0 ∼ 4BID, 4IKD0 ∼ 4CID, 4IY D0 ∼ 4XID)

và hệ thức T B

T C = AB CD ·

BD

AC, ta chứng minh X, T, Z thẳng hàng cách XB

XC = T B T C ·

ZB ZC

với việc thực biến đổi tương đương:

ZB ZC =

RB RC ·

AC AB = Y L

Y K · AC AB =

LD0−Y D0

Y D0+KD0 ·

AC AB =

r2 BD −

r2 XD r2

XD + r2 CD

· AC

AB =

XB XD·BD

XC XD·CD

· AC

AB = XB

XC · CD BD ·

AC AB =

XB XC ·

T C T B

Như ta có điều phải chứng minh

Nhận xét. Bài toán toán đặc trưng cho phương pháp biến đổi tỉ số giải tốn hình học phẳng Bải tốn phát biểu đơn giản có số hệ thú vị Ta đến với kết hay mơ hình đường tròn Thébault - dạng tổng quát đường tròn Mixtillinear

Bài tốn 2(Trần Qn). Cho4ABCcó tâm ngoại tiếpO.(K)là đường tròn quaB,C (J)là đường tròn tiếp xúcAB,AC tiếp xúc với(K).Dlà điểm cungBC

củaK choA,Dkhác phía so vớiBC Lấy điểmEtrênAOsao choDEvng góc vớiAJ

(150)

A

B C

I

K F

G

R

J

D O

E N

U

L

M

H

B0 C0

Chứng minh. Gọi tiếp điểm (J) (K)là R I tâm nội tiếp4ABC (J)tiếp xúc với CA,ABlần lượt tạiG,F

GF cắtAJ tạiN,DE cắtAJ tạiH,DE cắtABtạiL

M điểm cungBC khơng chứaAcủa(O).U đối xứng vớiDquaK

Ta có:

∠BLD = 90◦−∠BAH = 90◦−

2∠BAC

=∠BM O= 180◦−∠BM D

Điều dẫn đến điểm B, L, D, M đồng viên ∠BLM = ∠BDM Mà theo định lý

Protasov chứng minh, ta cóR,I, U thằng hàng vàB, R, I,F đồng viên Do đó∠BLM =

∠BDM =∠BRI =∠AF I nênLM song song vớiF I Theo định lý Thales: AN

AH = AF AL =

AI AM ⇔

AI AN =

AM AH

Như phép vị tự tâm A biến (O)thành AE biến M thành H, biến I thành N – tức

đường tròn(J)là ảnh đường trònA−Mixtillinear nội tiếp qua phép vị tự này, vậy(AE)

(151)

Từ toán ta chứng minh định lý tiếng hình học phẳng

Bài tốn 3 (Feuerbach). Trong tam giác, đường trịn chín điểm tiếp xúc với đường tròn nội tiếp

A

B C

O H

T I

D

M N

A0 K

J O0

R G

F E

V D0

Chứng minh. Chứng minh phụ thuộc vào hình vẽ

GọiH trực tâm 4ABC.(J)tiếp xúcCA, ABvà tiếp xúc (BHC).M, N điểm

chính cungBCkhơng chứaAvà cungBCkhông chứaAcủa(O) E,F đối xứng củaM,N quaBC

I tâm nội tiếp của4ABC.N I cắt lại(O)tạiT khácN Đường thẳng quaF vng gócAI

cắt AI, AOlần lượt tạiK, G Llà hình chiếu GlênAT, đường tròn nội tiếp(I)tiếp xúc BC tạiDvà gọiD0 đỗi xứngDquaN I Khi ta điều sau:

• T L

T A = cos(∠A)

• D0 thuộcAT và T D

T A = (cos(∠ A

2)

2)⇒D0L=D0A.

• Theo tốn 2, ta có(ALG)tiếp xúc(J)suy ra(J)là ảnh của(I)qua phép vị tự tâmA

tỉ số

Từ ta thu điều phải chứng minh

(152)

Bài toán 4. Cho4ABC cố định nội tiếp(O), ngoại tiếp(I).P trên(O) Hai tiếp tuyến

từ P tới(I)cắt tiếp tuyến song songBC của(I)lần lượt X, Y.Chứng minh rằng(P XY)

tiếp xúc với đường tròn cố định khiP di chuyển trên(O)

(Bạn tìm mở rộng cho toán này!)

Bài toán 5. (Yetti): Cho4ABC Cố định nội tiếp(O),ngoại tiếp(I)vàP mặt phẳng

sao choP không nằm trong(I).AP cắt(O)tạiK khácA Tiếp tuyến tạiP của(I)cắtBC lần

lượt tạiX,Y.Chứng minh rằng(KXY)đi qua điểm cố định khiP di chuyển

Bài toán 6(Lê Xn Hồng). Cho4ABC E trênCA Gọi(J)là đường trịn tiếp xúc

với hai cạnhCA,ABvà tiếp xúc với(BEC)tạiT Một đường trịn(K)bất kì quaB, C (L)là đường tròn tiếp xúc với hai cạnh CA, AB tiếp xúc với (K) tạiR Tiếp

tuyến R của(K)cắtBC tạiS ST cắt(BEC)tạiX khác T Chứng minh BT,AR, XE đồng quy

Bài toán 7(Tổng quát hoá bổ đề Sawayama). Cho4ABC,(K)bất kỳ quaB,C vàAnằm

ngoài(K).(K)cắtABởF khácB.(Oa)tiếp xúc ngoài(K)và tiếp xúc cạnhCA,ABlần

lượt X, Y dlà đường thẳng quaB (O)tiếp xúcCA vàd (như hình vẽ) tiếp

xúc trong(K)ởT Phân giác của∠BT F cắtXY ởJ

Chứng minh rằngJnằm phân giác góc giữaABvàd

(153)

MỘT MỞ RỘNG CỦA ĐƯỜNG THẲNG SIMSON

Nguyễn Minh Hà Lê Viết Ân

GIỚI THIỆU

Trong viết này, giới thiệu đến bạn đọc mở rộng đường thẳng Simson tính chất liên quan đến đường thẳng Simson mở rộng

1 Giới thiệu

Trong hình học, định lý đường thẳng Simson phát biểu sau

Định lý Cho tam giácABC và điểmP nằm đường tròn ngoại tiếp tam giác.

Khi đó, hình chiếu điểmP trên cạnh tam giác thẳng hàng.

Đường thẳng qua hình chiếu gọi Đường thẳng Simsoncủa điểm P

tam giácABC Đường thẳng đặt theo tên nhà toán học Robert Simson, [1] Tuy

nhiên, khái niệm xuất lần đầu William Wallace, [2] Điều ngược lại định lý đúng, cụ thể là:

Định lý Nếu hình chiếu điểmP trên cạnh tam giác thẳng hàng điểm P nằm đường tròn ngoại tiếp tam giác đó.

Đã có nhiều mở rộng cho định lý Tuy nhiên, đưa mở rộng cho định lý sau

Định lý Cho tam giác ABC có trực tâm H Xét điểmP thuộc mặt phẳng tam giác Một

đường tròn I / đi qua H P cắt AH; BH; CH theo thứ tự tại A0; B0; C0 Điểm Q bất kì

thuộc.I / Các đường thẳng quaP song song vớiQA0; QB0; QC0và cắtBC; CA; AB theo

thứ tự tạiA0; B0; C0 Khi đó:

a= Các điểmA0; B0; C0 thẳng hàng khiP thuộc đường tròn ngoại tiếp tam giác

ABC.

b= Trong trường hợpA0; B0; C0 thẳng hàng giao điểm hai đường thẳngA0B0C0

PQ(nếu có) thuộc trung trực củaPH.

(154)

2 Phép chứng minh Định lý 3

a/ (Xem hình vẽ 1) Ta có

.PA0; PB0/.QA0; QB0/.HA0; HB0/.CB; CA/.CA0; CB0/.mod / Suy raP; C; A0; B0đồng viên

A

B A0 C

B0

C0

I

H Q

B0 C0

A0 P

Hình

Tương tự điểmP; B; C0; A0đồng viên điểmP; A; B0; C0cũng đồng viên Vậy

.A0C0; A0B0/.A0C0; A0P / A0B0; A0P / mod /

.BC0; BP / CB0; CP / mod /

.BA; BP / CA; CP / mod /

0 mod /:

(155)

b/ Theo câu a, rõ ràng ta có đường thẳng Ta xét hai trường hợp sau đây:

Trường hợp :P giao điểm củaˇ.ABC / với AH, BH CH

Trường hợp kết hiển nhiên

Trường hơp 2:P không giao điểm củaˇ.ABC / đường cao tam giác ABC Khi đó, trung trực củaHP cắt đường thẳngBC; CA; AB (hình vẽ 2)

A

B A0 C

B0

C0

I

H Q

B0 C

0

A0 P

M N C1

A1

B1

Hình

(156)

Ta có

.PA1; P C1/.H C1; HA1/ mod /

.H C1; HN /C.HN; HA1/ mod /

.HcC1; HcN /C.HaN; HaA1/ mod /

.AB; DcP /C.DaP; BC / mod /

.AB; BC /C.DaP; DcP / mod /

.BA; BC /C.DaB; DcB/ mod /

.BA; BDc/C.BDa; BC / mod /

.BH; BA/C.BC; BH / mod /

.BC; BA/ mod /

.BC1; BA1/ mod /: Suy bốn điểmP; B; C1; A1đồng viên

Tương tự, điểmP; C; A1; B1đồng viên vàP; A; B1; C1đồng viên Lại có

.C0C1; M C1/.HcC1; NC1/ mod /

.HcH; NH / mod /

.C0H; PH / mod /

.C0Q; PQ/ mod /

.C0P; MP / mod /: Suy điểmP; M; C0; C1 đồng viên

Tương tự, ta có điểmP; M; B0; B1đồng viên vàP; M; A0; A1đồng viên Từ bốn điểm đồng viên vừa chứng minh trên, ta có

.MA0; M C0/.MP; M C0/ MP; MA/ mod /

.C1P; C1C0/ A1P; A1A0/ mod /

.C1P; C1B/ A1P; A1B/ mod /: Suy ba điểmM; C0; A0thẳng hàng

Do đóM 2A0B0C0 Ta có điều phải chứng minh

3 Một số tính chất áp dụng Định lý 3

Định lý Cho tam giácABC,.O/là đường tròn ngoại tiếp,H là trực tâm..I /là đường tròn

bất kì quaH và cắt.O/tạiP1; P2và theo thứ tự cắtHA; HB; H C tạiA0; B0; C0 ĐiểmQ

thuộc.I / Các hai điểmfAIA2g,fB1IB2g,fC1IC2gtheo thứ tự thuộcBC; CA; AB sao cho

P1A1 kP2A2kQA0;P1B1kP2B2 kQB0;P1C1 kP2C2 kQC0 Chứng minh rằng

1/ Các ba điểmfA1; B1; C1g fA2; B2; C2gthẳng hàng, kí hiệu41;42 theo thứ tự là

các đường thẳng chứa ba điểm này.

(157)

Lời giải (Xem hình vẽ 3)

1) Các ba điểmfA1; B1; C1gvàfA2; B2; C2gthẳng hàng theo định lý

A

B C

P1

P2

A0

H B0

C0

C1

A1

B1

A2

C2

B2

Q

1

2

O

I

Hình 2) Ta có

.41;42/.A1C1; A2B2/ mod /

.A1C1; A1B/C.A2C; A2B2/ mod /

.P1C1; P1B/C.P2C; P2B2/ mod /

.H C; P1B/C.P2C; P1B/ mod /

.AB; AC /C.P2C; P1B/ mod /

.AB; P1B/C.P2C; AC / mod /

.AB; P1B/C.P2B; AB/ mod /

.P2B; P1B/ mod /

2

!

(158)

Tài liệu

[1] Gibson History - Robert Simson

http://www-groups.dcs.st-and.ac.uk/~history/Extras/Gibson_ history_7.html

[2] Simson line

http://www.cut-the-knot.org/Curriculum/Geometry/Simpson shtml

(159)

MÃ VÀ CÁC KỲ THI OLYMPIC TOÁN (II)

S.B.Gashkov

Tiếp theo kỳ trước, xem phần mở đầu mục1; 2; 3ở Epsilon số15

4 Ví dụ đơn giản mã sửa lỗi

Cần làm thứ trở nên đơn giản có thể, khơng đơn giản hơn

Albert Einstein Claude Shannon21/ đưa ý tưởng mã sửa sai Ta xét ví dụ mã sửa lỗi (trường hợp

riêng đơn giản mã Hamming) Giả sử ta phải truyền từ nhị phân.x1; x2; x3; x4/ta bổ

sung ký tự kiểm trax5 Dx1Cx3Cx4; x6 Dx1Cx2Cx4; x7 D x1Cx2Cx3 (dấuC

ở phép cộng theo modulo 2;các ký tựx1; x2; x3; x4 gọi ký tự thơng tin) Quy

trình tính theo ký tự thông tin ký tự kiểm tra thiết lập từ mã (bản tin mã hóa) gọi mã hóa (và ánh xạ từ tin ban đầu thành từ mã gọi mã hóa) Trên ngơn ngữ ma trận ví dụ xét việc mã hóa quy việc nhân ma trận M với ma trận chuyển vị.x1; x2; x3; x4/T;tức vector cột

0 B B B B B B B B @

1 0 0 0 0 0 0 1 1 1 1 1

1 C C C C C C C C A B B @ x1 x2 x3 x4 C C AD B B B B B B B B @ x1 x2 x3 x4 x5 x6 x7 C C C C C C C C A :

Ta truyền tin mã hóa c D x1; x2; : : : ; x7/ thu tin nhiễu r D c C e với

e D.e1; e2; : : : ; e7/là vector lỗi Trong trường hợp có trọng bằng1;vì theo giả

thiết, lỗi xảy (nếu thực có lỗi) vị trí Giả sử, làe De3 D.0; 0; 1; 0; 0; 0; 0/:

Khi

r DcCeD.c1; c2; c3C1; c4; c5; c6; c7/D.c1; c2; c3; c4; c5; c6; c7/;

trong đó0D1; 1D0:Trong trường hợp xét, số3được gọi vị trí lỗi Để xác định vị trí lỗi, tính tổng kiểm tra

S1 Dr1Cr3Cr4Cr5;

S2 Dr1Cr2Cr4Cr6;

(160)

Một cách trực quan, tất tổng minh họa hình

Mỗi tổng chứa vịng trịn Trên ngơn ngữ ma trận, trình tương đương với việc nhân ma trận cho vector

0

@

1 1 0 1 1 1 0

1

A

B B B B B B B B @

r1

r2

r3

r4

r5

r6

r7

1

C C C C C C C C A

D

0

@

S1

S2

S3

1

A:

Ma trậnH gọi ma trận kiểm tra mã Trong cách viết gọn, phép nhân ma trậnH cho vectorrT được viết làS DH rT;trong đóS là vector cột.S

1; S2; S3/T

Chú ý ma trậnH chọn cho H cT D 0(trong đó0là vector0dạng cột) với mọi

vector mã c: Điều kiểm tra trực tiếp (và không cần phải sử dụng ngôn ngữ ma trận, cần tổng Si thay x5; x6; x7 giá trị chúng tính qua

.x1; x2; x3; x4/) Trên ngôn ngữ ma trận điều kiểm chứng dễ dàng Chú ý ma

trậnM viết dạng E A

!

;trong đóE ma trận đơn vị22/kích thước44vàAlà ma trận kích thước34:Ma trậnH tạo thành từ ma trận conAvà ma trận đơn vị kích thước33;vì

H cT DA.x1; x2; x3; x4/T CE.x5; x6; x7/T DA.x1; x2; x3; x4/T C.x5; x6; x7/T D0T;

vì từ đẳng thức

M x1; x2; x3; x4/T D.x1; x2; x3; x4; x5; x6; x7/T;

ta có

A.x1; x2; x3; x4/T D.x5; x6; x7/T:

Sử dụng tính chất dễ kiểm tra phép nhân cộng ma trận, ta S DH rT DH.cT CeT/DH eT DHi;

trong e vector với số 1duy vị trí thứi; Hi cột thứi ma trận H (đẳng

thức kiểm tra trực tiếp, thayrj tổng Si bởicj Cej D xj Cej/:Bây

(161)

xác định cách xác số thứ tự nó, có nghĩa vị trí lỗi Nếu khơng có lỗi, hiển nhiên làS D H rT D H cT D 0;và đẳng thức kiểm tra cách so sánhS với vector0dạng cột (nếu nhưS khác0thì tất nhiên xảy lỗi) Để xác định vị trí lỗi theo vector S tính (được gọi hội chứng) ta lập bảng có chứa vị trí lỗi, ví dụ cách viết nhị phân Bảng bao gồm dòng, đánh số số nhị phân độ dài3:Nếu lỗi số thứ tự cho bằng0:Nếu ma trận kiểm tra có dạng

0

@

1 1 0 1 0 1 1 1

1

A;

thì bảng khơng cần thiết, hội chứngS trường hợp trùng với thứ tự nhị phân vị trí lỗi Dĩ nhiên, ma trậnM phải thay đổi

Mã vừa xây dựng trường hợp riêng mã Hamming, nói đến phần Trong trường hợp chiều dài từ mã bằng31, ý tưởng xây dựng mã gần với toán 5; đề xuất nhà toán học Pháp Lucas từ kỷ XIX (điều trở nên rõ ràng sau đọc phần5) Và3bài toán gần với toán

Bài tốn 23 (Từ sở liệu toán Olympic toán Matxcova) Trong tập hợpn

phần tử ta chọn 5ntập phân biệt gồm 2phần tử Chứng minh cách hợp chúng

lại đơi một, ta thu khơng hơn45ntập con3phần tử.

Bài tốn 24 Hãy tìm đồng tiền giả từ63đồng tiền, sử dụng chỉ6lần cân, biết đồng

tiền giả nhẹ đồng tiền thật Mỗi lần cân cân nhiều đồng tiền lúc Kế hoạch cần phải thiết lập từ trước.

Bài toán 25 (MMO 1990, 8.5) Bảng điện bao gồm64bóng đèn, điều khiển bởi64chiếc

cơng tắc, bóng đèn cơng tắc Mỗi lần thực đồng thời nhấn một cơng tắc ghi lại bóng sáng Hỏi cần lần thực để có thể biết tất bóng đèn bảng: Bóng đèn điều khiển cơng tắc nào?

Lưu ý số tính chất mã xây dựng Nó có 24 D 16 từ mã, tổng hai từ mã theo modulo2lại từ mã (tức mã tuyến tính), khoảng cách mã bằng3:Khoảng cách khơng thể nhỏ hơn3;vì khơng khơng thể sửa lỗi, ta tính tường minh khoảng cách với ý d.a; b/D d.aCb; 0/:Vì mã chứa từ mã có trọng 0(từ 0) khoảng cách mã bằng3nên mã khơng có từ có trọng1hay2:Với từ mã ta xét hình cầu bán kính1với tâm từ Hình cầu chứa, ngồi tâm cịn7bộ nhị phân (đỉnh lập phương nhị phân7chiều), thu cách thay trung tâm 7ký tự thành ký tự đối

Các hình cầu với tâm từ mã khơng giao nhau24/ (khơng có đỉnh chung) đó

trong tổng thể chứa248 D27 đỉnh khác hình lập phương, tức tất đỉnh của

nó (hình cầu 7chiều có27đỉnh) Những phép phủ hình lập phương nhiều chiều hình cầu gọi hồn hảo, mã tương ứng gọi mã hoàn hảo

(162)

Hình 2:Hình lập phương chiều

Ở tìm phổ trọng lượng nó, số lượng từ trọng lượng cho Nó có dạng

a0 D1; a1 D0; a2 D0; a3 D7; a4D7; a5 D0; a6D0; a7 D1:

Ở giải thích từ mã trọng lượng xác định cấu hình tổ hợp thú vị -hệ thống ba Steiner26/;và chứng tỏ hệ thống ba đẳng cấu với cấu hình7đường

thẳng qua3điểm mặt phẳng xạ ảnh trên7điểm – gọi mặt phẳng Fano27/(hình 3)

Hình 3:Mặt phẳng Fano

(163)

7bộ4này tạo thành ví dụ sơ đồ khối, mà cặp phần tử thuộc hai bộ428/:

Các cấu hình nói (sơ đồ khối) lớp thứ ba thứ tư cịn có tính chất thú vị Chúng đối ngẫu với nhau, cụ thể là: Bộ 4của sơ đồ khối thứ hai phần bù ba sơ đồ khối thứ tập hợpf1; 2; 3; 4; 5; 6; 7g:

Ghi chú:

21) Claude Elwood Shannon 1916 2001/nhà toán học kỹ sư người Mỹ, cha đẻ lý thuyết thông tin

22) Ma trận vuông gọi đơn vị, đường chéo số1;cịn vị trí khác là0:Đường chéo đường chéo từ góc bên trái xuống góc bên phải Khi nhân ma trận đơn vị với vector kết thu vector 23) Điều xảy có8cột nhị phân khác có chiều cao3:

24) Nếu mặt cầu với tâma; bcó điểm chungcthìd.a; b/6d.a; c/Cd.c; b/62; điều xảy

25) Lớp k hình lập phương bao gồm đỉnh có trọng lượng k hiển nhiêu bao gồm

k

!

đỉnh

26) Jakov Steiner.1796 1863/nhà hình học tiếng người Thụy Sĩ 27) Gino Fano.1871 1951/nhà toán học tiếng người Ý

28) Về sơ đồ khối xem trongŒ4; 14; 16:

5 Mã Hamming

Hai ngày nghỉ liền đến phòng máy nhận tất liệu tải lên nhưng khơng có điều thực Tơi giận tím người tơi cần câu trả lời, hai ngày nghỉ đã trôi qua cách vơ ích Khi tơi nói với thân:“Quỷ thật, máy tính có thể phát lỗi điều ngăn cản xác định lỗi xảy chỗ sửa nó?”

Richard Hamming theo John MacCormick “Chín thuật tốn thay đổi giới” Chúng ta mã nhị phân Hamming Mã xây dựng sau XétnD2m 1; k D n m; giả sử x1; x2; : : : ; xk vector thông tin mà cần mã hóa Ta bổ

sung vào vector ký tự kiểm tra xkC1; : : : ; xn mà để tính chúng, ta nhân vector cột

X D x1; x2: : : ; xk/T với ma trận bậc.m; k/ Mn (vớimhàng kcột) mà cột

tất độ dàingồm số0và1;có chứa nhất2số1(có tất cả2m 1 mDn mDk

(164)

chúng tạo thành ma trận vuông với phần tử đường chéo bằng1), thu ma trận bậc.m; n/ Hn;có cột tất độ dàin;khác0gồm số0và1:Cũng

phần4;ta kiểm tra với từ mãx D x1; x2; : : : ; xn/T đẳng thức ma trận

sau thỏa mãnHnxT D0:

Ma trậnH (tiếp theo ta bỏ sốnđi) gọi ma trận kiểm tra mã Ma trận cho phép không kiểm tra từc cho có phải từ mã khơng, mà cịn cho phép tìm lỗi, có lỗi xảy

Thật vậy, 4;ta cóS D H rT D H.cT CeT/ D H eT D Hi;trong đóe vector

lỗi, có số1ở vị trí thứi;cịnHi cột ma trậnH với số thứ tựi:Vì tất cột

H khác khác0nên theo vector hội chứngS ta tìm cách xác vị trí lỗi

Cũng ở4;ta xét số tính chất mã xây dựng Mã có2kphần tử, tuyến tính, số chiều khơng gian vector trường hai phần tử bằngk(bởi sở khơng gian gồmkvector, khơng gian bao gồm tất tổng chúng, tức là2k tổng (tính tổng rỗng tương ứng với0theo định nghĩa)), khoảng cách mã bằng3(khoảng cách khơng thể nhỏ hơn3;vì khơng thể sửa lỗi) Vì mã chứa từ mã có trọng0(từ 0) khoảng cách mã bằng3nên mã khơng có từ có trọng1hay2:Với từ mã ta xét hình cầu bán kính1với tâm từ Hình cầu chứa, ngồi tâm cịnnbộ nhị phân (đỉnh lập phương nhị phân nchiều) Các hình cầu với tâm từ mã không giao tổng thể chứa2k.nC1/D2kCm D2nđỉnh khác hình lập phương, tức tất đỉnh Vì mã Hamming hồn hảo (và xếp kín) Cũng hiển nhiên mã cực đại với khoảng cách3sẽ có tham số giống với mã Hamming

Thật vậy, cận đánh giá m.n; 3/ n2Cn1 đạt thìnC1 D 2m (nếu khơng phân số cho khơng phải số ngun), từ đóm.n; 3/D 2n m:Như tốn3được giải hồn tồn, ta dễ dàng giải toán

Bài toán 26 Hỏi tìm số từ 1đến 2048 với 15 câu hỏi dạng (có/khơng) hay

khơng, ta có quyền trả lời sai câu hỏi đó? Các câu hỏi phải chọn trước.

Và toán theo chủ đề này:

Bài tốn 27 (19, trang 120) Ba nhà thơng thái có mũ màu đen màu trắng.

Người dẫn chương trình đội cho nhà thông thái mũ, cho người sẽ nhìn thấy mũ hai người khơng nhìn thấy mũ khơng biết màu mũ của mình Theo hiệu lệnh, nhà thơng thái đồng loạt đốn màu mũ Mỗi nhà thơng thái sẽ dự đoán màu mũ dựa vào màu mũ mà ơng ta nhìn thấy hai người cịn lại, họ được quyền bỏ qua, nghĩa từ chối không đốn Các nhà thơng thái thắng một người họ đoán màu mũ đồng thời khơng có họ đốn sai Trước chơi diễn nhà thông thái thông báo luật chơi cho phép họ thảo luận trước chiến thuật chơi Chiến thuật tối ưu chiến thuật mà với tất cách xếp mũ cho nhiều trường hợp chiến thắng nhất.

a) Hãy đề xuất chiến thuật nhà thông thái cho họ chiến thắng nhiều hơn một nửa trường hợp.

(165)

5.1 Mã Hamming q-phân

Tôi cô gái tội nghiệp, số học! Lớn toán cao cấp rồi

Dmitry Emetz, “Tanhia Grotter” Trong số trường hợp tồn mã q-phân hồn hảo Cận Hamming đạt chẳng hạn khi1C.q 1/n D qm:Khi số phần tử mã bằngqn m; đónlà độ dài khối Các mã xây dựng trường hợp tồn trường hữu hạnqphần tử29/:

Trường tập hợp mà định nghĩa hai phép toán cộng nhân cho phép toán thỏa mãn tính chất y phép tính cộng nhân cáo số hữu tỷ, cụ thể tính giao hốn:aCb DbCa; abDba;tính kết hợp:.aCb/Cc DaC.bCc/; ab/cDa.bc/; tính phân phối:a.bCc/D abCac thỏa mãn đẳng thứcaC0 D a; a1 D a; định nghĩa cách phép toán ngược phép trừa bvà phép chiaa=b thỏa mãn tính chất.a b/CaDa; a=b/bDa:

Từ đại số ta biết số phần tử qcủa trường hữu hạn phải lũy thừa số nguyên tốp; qDpn;hơn với sốqnhư tồn trường30/bậcq;gọi trường Galois31/và ký hiệu làGF q/:Ví dụGF 2/

D.f0; 1g;˚; /:

Ta xây dựng trườngGF q/mã, mở rộng mã Hamming Với vectorvkhác0độ dài mtrên trườngGF q/ta xét tập hợp vector song song với nóv;trong đó2GF q/n f0g: Tập hợp tạo thành đường thẳng không gianGF q/n;đi qua gốc tọa độ Ta chọn đường thẳng (số đường thẳng bằngnD.qm 1/=.q 1/;vì chúng có điểm chung gốc tọa độ) điểm khác với gốc tọa độ, ví dụ chọn điểm mà tọa độ cuối bằng1:Điều ln thực ngoại trừ trường hợp tất điểm đường thẳng có tọa độ cuối bằng0:Trong trường hợp chọn điểm mà tọa độ gần cuối 1:Nếu lại khơng có điểm tọa độ gần cuối bằng0thì ta lại chọn điểm có tọa độ thứ ba kể từ cuối bằng1; : : :Điểm đường thẳng (và vector bán kính tương ứng) xác định cách Tất vector khác thu cách nhân vector chọn cho phần tử trường (khi nhân với0ta vector0) Hiển nhiên vector khác0của không gianGF q/nsẽ thu từ vector cho v1; v2; : : : ; vnbằng cách nhân với phần tử khác0của trường, biểu diễn

duy

Xét ma trận m; n/ Hn trường GF q/; có cột vector v1; v2; : : : ; cho Định

nghĩa mã tập hợp vectorc;sao cho H cT D 0(tức không gian0của ma trận cho) Khi đóHn ma trận kiểm tra mã cho Mã sửa lỗi, vì, phần4;

ta có

S DH rT DH.cT CeT/DH eT DeiHi;

trong đóe vector lỗi, có ký tự khác0; ei vị trí thứi;cịnHi cột thứi ma

trận H:Bởi tất cột khác khác cột 0;và tích chúng với phần tử khác0của trường khác nên theo vectorS xác định cách vị trí i lỗi, giá trịei Vì ma trậnHn(sau đổi chỗ cột cách thích hợp) chứa

(166)

trườngGF(q)có lực lượngqn−m), nghĩa mã nằm xác cận xếp cầu đó

là mã hồn hảo Như phần d) toán4đã giải Lời giải phần b) độc giả có

thể xem ở[2,trang165−166]

Ghi chú:

29) Có giả thuyết cho với qkhơng phải lũy thừa số nguyên tố, mã hoàn hảo

như khơng tồn 30) Chính xác đến đẳng cấu trường

31) Evarist Galois(1811−1832)nhà toán học vĩ đại người Pháp

Tài liệu

[1] Берлекэмп Э Р Алгебраическая теория кодирования М.: Мир, 1971

[2] Васильев Н Б., Егоров А А Задачи Всесоюзных математических олимпиад М.: Наука, 1988

[3] Гальперин Г А., Толпыго А К Московские математические олимпиады М.: Про-свещение, 1986

[4] Гашков С Б Разностные множества, конечные геометрии, матрицы Заранкеви-ча и экстремальные графы // Математическое просвещение Сер Вып 21 М.: МЦНМО, 2017 С 145–185

[5] Гашков С Б Графы-расширители и их применения в теории кодирования // Математическое просвещение Сер Вып 13 М.: МЦНМО, 2009 С 104–126 [6] Левенштейн В И Элементы теории кодирования // Дискретная математика и математическая кибернетика Т М.: Наука, 1974

[7] Мак-Вильямс Ф Дж., Слоэн Н Дж А Теория кодов, исправляющих ошибки М.: Связь, 1979

[8] Прасолов В В и др Московские математические олимпиады 1935–1957 гг М.: МЦНМО, 2010 Коды и олимпиады 173

[9] Прасолов В В и др Московские математические олимпиады 1958–1967 гг М.: МЦНМО, 2013 [10] Бегунц А В и др Московские математические олимпиады 1981–1992 гг М.: МЦНМО, 2017 [11] Фёдоров Р М и др Московские математи-ческие олимпиады 1993–2005 гг / 3-е изд М.: МЦНМО, 2017

(167)

[14] Таранников Ю В Комбинаторные свойства дискретных структур и приложения к криптологии М.: МЦНМО, 2011

[15] Фейеш Тот Л Расположения на плоскости, на сфере и в пространстве М.: Физ-матгиз, 1958 [16] Холл М Комбинаторика М.: Мир, 1970

[17] Чашкин А В Дискретная математика М.: Академия, 2012

[18] Guruswami V., Sudan M Improved decoding of Reed — Solomon and algebraicgeometric codes // IEEE Trans Inform Theory 1999 Vol 45 P 1757–1767

(168)

KỲ THI TỐN BALTIC WAY 2019

Nguyễn Hùng Sơn

TĨM TẮT

Xin giới thiệu đến bạn đọc kỳ thi tốn mang tên Baltic Way Tác giả báo có vinh dự giao trọng trách làm chủ tịch hội đồng đề chấm thi thi Baltic Way 2019 Bài báo chia sẻ thi số quan sát trình chọn đề chấm thi

1 Giới thiệu thi

Baltic Way thi toán đồng đội tổ chức lần vào năm 1990 với tham gia nước cộng hịa Xơ Viết cũ Litva, Latvia Estonia

Tên gọi thi có xuất xứ từ kiện mang tên “Con đường Baltic” (Baltic Way) diễn vào ngày 23 tháng năm 1989 xấp xỉ hai triệu người nắm tay tạo thành chuỗi dài sáu trăm số trải qua ba nước vùng Baltic Latvia, Litva Estonia để đòi độc lập

Kể từ năm 1992, tình hình trị châu Âu thay đổi, thi mở rộng theo ý nghĩa tên gọi với tham gia nước cộng hòa xung quanh biển Baltic Ba Lan, Đan Mạch, Đức (các tỉnh phía bắc), Na Uy, Phần Lan, Thụy Điển Ngồi cịn có tham gia Ai-xơ-len, Ai-xơ-len (Iceland) nước công nhận độc lập nước Latvia, Litva Estonia, đội tuyển thành phố Saint Petersburg (Liên bang Nga) Để thay đổi khơng khí, ban tổ chức nước chủ nhà mời thêm quốc gia vùng Baltic tham gia thi

Cuộc thi Baltic Way tổ chức vào đầu tháng 11 hàng năm Mỗi quốc gia cử đội tuyển người gồm thí sinh học sinh tiểu học trung học thời điểm diễn thi Các đội phải giải thời gian 30 phút 20 toán (gồm đại số, hình, tổ hợp lý thuyết số) Mỗi có điểm số điểm nên đội đạt tối đa 100 điểm Trong lịch sử thi, có lần có đội đạt điểm tối đa 100/100 Đó thi Baltic Way tổ chức vào năm 2001 Hamburg (Đức), đội tuyển khách Israel lập nên kỳ tích

(169)

2 Kỳ thi Baltic Way lần thứ 30

Baltic Way lần thứ 30 tổ chức thành phố Szczecin, Balan từ 15 đến 19 tháng 11 năm 2019 Tơi vinh dự Ủy ban Olimpic Tốn Ba lan đề cử làm trưởng ban đề (chair of jury) Cũng nhiều kỳ thi quốc tế khác, nước tham gia gửi đề đến cho trưởng ban đề để chuẩn bịshortlist, tức danh sách có khả chọn làm đề thi Khi đến dự thi, trước ngày thi ngày, trưởng phó đồn nhận shortlist để chuẩn bị cho trình chọn Ngày 17/11, ban đề gồm trưởng ban đề trưởng đoàn họp để chọn cách biểu

Kỳ thi đồng đội nên ban đề phải chọn 20 thi thuộc lịch vực: Đại số, Số học, Hình học Tổ hợp Trong trình chọn đề, trưởng đoàn quyền biểu Trưởng ban đề có chức điều khiển họp, phân tích, đánh giá, so sánh lựa chọn khơng có quyền đưa lựa chọn Kỳ thi năm có 11 nước tham gia nên có 11 trưởng đồn quyền biểu Vì 11 số nguyên tố nên may mắn không xảy trường hợp lựa chọn có số phiếu Cũng cần lưu ý lúc chọn bài, trưởng đồn khơng biết tốn shortlist nước đề cử Trong trình chuẩn bị shortlist, trưởng ban đề thấy đề nghị lĩnh vực số học tổ hợp yếu nên mượn thêm ngân hàng đề bải Olimpic tốn Ba lan Rất tình cờ chọn vào danh sách 20 thi thức thi

Sau chọn 20 thi, ban đề lại tiếp tục họp để đưa phương án hoàn chỉnh phiên tiếng Anh Thời gian kéo dài khoảng tiếng đồng hồ có tranh luận liệt Sau đó, trưởng đồn phải dịch đề tiếng Anh sang tiếng nước Năm tồn trình từ chọn đến lúc in ấn đề xong kéo dài từ 10:00 sáng đến 22:30 Đây họp kéo dài ban đề Có năm họp kéo dài đến sáng

Trong thời gian ban đề chỉnh sửa đề thi ban chấm thi bắt đầu chỉnh sửa lời giải lên thang điểm (marking schemes) Ban chấm thi hoàn thiện marking scheme trước thi kết thúc tiếng đồng hồ đưa cho trưởng đồn Cuộc thi thức diễn hơm 18/11/2019 thí sinh làm thi từ 9:30 đến 15:00 Sau thi trưởng đồn chấm trước, sau ban giám khảo phối hợp cho điểm Toàn trình coordination kết thúc lúc 22:00 ngày

Kết quả, giải thi Baltic Way 2019 thuộc đội Saint Peterburg với kết 90 điểm (trên tổng số20×5 = 100điểm), giải nhì đội Ba lan (83 điểm) giải ba thuộc đội Estonia

(70 điểm)

3 Một kỳ thi giản dị, dân chủ trung thực

(170)

Công việc tổ chức giao cho sở giáo dục thành phố sở chọn trường cấp làm đơn vị đăng cai Năm 2019, thi tổ chức trường phổ thông trung học số XIII thành phố Szczecin phụ trách toàn thi thầy cô giáo trường Đây thi từ trước tới mà người biết đề không bị cách ly khỏi thí sinh

Sau in đề, Chúng tơi nghỉ khách sạn Các thí sinh nghỉ khách sạn với trưởng đoàn Sáng hơm sau, trưởng đồn, thí sinh ban giám khảo ăn sáng lên xe để đến nơi thi Đến nơi, trưởng đồn vào phịng riêng, ban chấm thi vào phịng khác cịn thí sinh vào phịng thi

Trong q trình chọn bài, hỏi có có ý kiến khơng số trưởng đồn đứng lên xin phát biểu:

• thưa quý vị, xin thông báo cách tuần, vừa dạy học sinh có cấu

trúc hình học tương đối giống lời giải G9 Nếu chọn lợi cho học sinh tơi q

• hoặc: Bài A6 giống toán quen thuộc học sinh Phần lan

Sự trung thực bạn thật đáng khâm phục Tuy nhiên sau nghe trình bày cụ thể, trưởng đồn khác cho lợi không đáng kể

Sau chọn xong Các trưởng đoàn phải dịch toàn đề sang tiếng nước Sau in ấn đề xong, 22:30 Chúng tơi nghỉ khách sạn Các thí sinh nghỉ khách sạn với trưởng đoàn

Sáng hơm thi, trưởng đồn, thí sinh ban giám khảo ăn sáng đến nơi thi Đến nơi, trưởng đoàn vào phòng riêng, ban chấm thi vào phòng khác cịn thí sinh vào phịng thi

Nếu muốn gian lận có nhiều cách để thí sinh biết đề trước thi Tuy quan sát năm trở lại đây, chưa nước chủ nhà đạt giải thi

Đúng ngày hội em học sinh tham gia kỳ thi

(171)

Time allowed: 4.5hours

During the first30minutes,questions may be asked

Tools for writing and drawing are the only ones allowed

Problem 1.For all non-negative real numbersx,y,zwithx≥y, prove the inequality

x3−y3+z3+1

6 ≥(x−y)√xyz

Problem 2.Let (Fn) be the sequence defined recursively byF1=F2 =1 andFn+1 =Fn+Fn−1forn≥2 Find

all pairs of positive integers (x,y) such that

5Fx−3Fy=1

Problem 3.Find all functions f :R→Rsuch that

f(x f(y)−y2)=(y+1)f(x−y)

holds for allx,y∈R

Problem 4.Determine all integersnfor which there exist an integerk≥2 and positive integersx1,x2, ,xk so

that

x1x2+x2x3+ .+xk−1xk =n and x1+x2+ .+xk=2019

Problem 5.The 2mnumbers

1·2, 2·3, 3·4, , 2m(2m+1)

are written on a blackboard, wherem ≥ is an integer Amoveconsists of choosing three numbersa,b, c, erasing them from the board and writing the single number

abc

ab+bc+ca

Afterm−1 such moves, only two numbers will remain on the blackboard Supposing one of these is 3, show

that the other is larger than

Problem 6.Alice and Bob play the following game They write the expressionsx+y, x−y,x2+xy+y2and

x2−xy+y2each on a separate card The four cards are shuffled and placed face down on a table One of the

cards is turned over, revealing the expression written on it, after which Alice chooses any two of the four cards, and gives the other two to Bob All cards are then revealed Now Alice picks one of the variablesxandy, assigns a real value to it, and tells Bob what value she assigned and to which variable Then Bob assigns a real value to the other variable

Finally, they both evaluate the product of the expressions on their two cards Whoever gets the larger result, wins Which player, if any, has a winning strategy?

Problem 7.Find the smallest integerk≥2 such that for every partition of the set{2,3, ,k}into two parts, at

least one of these parts contains (not necessarily distinct) numbersa,bandcwithab=c

(172)

Problem 9.For a positive integern, consider all nonincreasing functions f: {1, ,n} → {1, ,n} Some of

them have a fixed point (i.e acsuch that f(c)=c), some not Determine the difference between the sizes of

the two sets of functions

Remark.A function f isnonincreasingif f(x)≥ f(y) holds for allx≤y

Problem 10.There are 2019 points given in the plane A child wants to drawk(closed) discs in such a manner, that for any two distinct points there exists a disc that contains exactly one of these two points What is the minimalk, such that for any initial configuration of points it is possible to drawkdiscs with the above property?

Problem 11.LetABCbe a triangle withAB=AC LetMbe the midpoint ofBC Let the circles with diameters

ACandBMintersect at pointsMandP LetMPintersectABatQ LetRbe a point onAPsuch thatQRkBP Prove thatCPbisects∠RCB

Problem 12.LetABCbe a triangle andHits orthocenter LetDbe a point lying on the segmentACand letE

be the point on the lineBCsuch thatBC⊥DE Prove thatEH⊥BDif and only ifBDbisectsAE

Problem 13 Let ABCDEF be a convex hexagon in which AB = AF, BC = CD, DE = EF and∠ABC =

∠EFA=90◦.Prove thatAD⊥CE

Problem 14.LetABCbe a triangle with∠ABC=90◦, and letHbe the foot of the altitude fromB The points

MandN are the midpoints of the segmentsAHandCH, respectively LetPandQbe the second points of intersection of the circumcircle of the triangleABCwith the linesBMandBN, respectively The segmentsAQ

andCPintersect at the pointR Prove that the lineBRpasses through the midpoint of the segmentMN

Problem 15.Letn≥4, and consider a (not necessarily convex) polygonP1P2 .Pnin the plane Suppose that,

for eachPk, there is a unique vertexQk ,PkamongP1, ,Pnthat lies closest to it The polygon is then said to

behostileifQk, Pk±1for allk(whereP0= Pn,Pn+1 =P1)

(a) Prove that no hostile polygon is convex

(b) Find alln≥4 for which there exists a hostilen-gon

Problem 16 For a positive integerN, let f(N) be the number of ordered pairs of positive integers (a,b) such

that the number

ab

a+b

is a divisor ofN Prove that f(N) is always a perfect square

Problem 17.Letpbe an odd prime Show that for every integerc, there exists an integerasuch that

ap+21 +(a+c)p+21 ≡c (mod p).

Problem 18.Leta,b, andcbe odd positive integers such thatais not a perfect square and

a2+a+1=3(b2+b+1)(c2+c+1)

Prove that at least one of the numbersb2+b+1 andc2+c+1 is composite.

Problem 19.Prove that the equation 7x =1+y2+z2has no solutions over positive integers.

Problem 20.Let us consider a polynomialP(x) with integer coefficients satisfying

P(−1)=−4, P(−3)=−40, and P(−5)=−156

What is the largest possible number of integersxsatisfying

Ngày đăng: 08/02/2021, 07:40

Xem thêm:

w